final patho exam

Pataasin ang iyong marka sa homework at exams ngayon gamit ang Quizwiz!

what term should the nurse use when talking about the outermost membrane surrounding the brain? a. dura mater b. arachnoid mater c. pia mater d. falx cerebri

A

As the problem of childhood and adolescent obesity increases, an increase in the incidence of ____________ is occurring in this obese population. a. type 2 diabetes mellitus b. attention deficit disorder c. juvenile rheumatoid arthritis d. antibiotic-resistant bacterial infections

A

Both binge-eating and bulimia nervosa patients consume excessive amounts of foods secretively. A major difference is that binge-eaters: a. remain overweight. b. eat when not hungry. c. are substance abusers. d. experience depression.

A

A patient wants to know why ADH is important in the body. What is the nurses best response? ADH is important in: a. The bodys water balance and urine concentration b. Maintaining electrolyte levels and concentrations c. Follicular maturation d. Regulation of metabolic processes

A

A 12-year-old male is newly diagnosed with type 1 DM. Which of the following tests should the nurse prepare the patient to best confirm the diagnosis? a. Fasting plasma glucose levels b. Random serum glucose levels c. Genetic testing d. Glycosylated hemoglobin measurements

A

A 15-month-old child from Pennsylvania was brought to the ER for fever, seizure activity, cranial palsies, and paralysis. Which of the following diagnosis will be documented in the chart? a. Eastern equine encephalitis b. Venezuelan encephalitis c. St. Louis encephalitis d. West Nile encephalitis

A

A 15-year-old female presents with breast discharge, dysmenorrhea, and excessive excitability. Tests reveal that all her pituitary hormones are elevated. What does the nurse suspect as the most likely cause for these assessment findings? a. A pituitary adenoma b. Hypothalamic hyposecretion c. Hypothalamic inflammation d. Pheochromocytoma

A

A patient with an addiction to alcohol checked into a rehabilitation center. He experiences delirium, inability to concentrate, and is easily distracted. From which of the following is he most likely suffering? a. Acute confusional state b. Echolalia c. Dementia d. Dysphagia

A

A staff member asks a nurse how blood gets to the bone. How should the nurse reply? Blood vessels penetrate to the inner structures of the bone via: a. Volkmann canals b. Canaliculi c. Sharpey fibers d. Lamellae

A

An attorney spoke to the nurses regarding brain death. Which statement indicates the nurses understood brain death? For legal purposes, brain death is defined as: a. Cessation of brain function b. Lack of cortical function c. A VS d. Death of the brainstem

A

An isolated muscle is believed to be a type 1 fiber. Which of the following should be observed by the nurse when the muscle is stimulated? a. The muscle resists fatigue. b. The muscle tires easily. c. The muscle will not contract. d. The muscle has no fascia

A

In addition to facilitating bowel movements, a diet that is high in fiber confers which of the following benefits? a. Lowering cholesterol and blood glucose b. Removing toxins and metabolic byproducts c. Lowering blood pressure and resting heart rate d. Increasing intestinal absorption of vitamins and minerals

A

Myotonia is characterized by: a. Prolonged depolarization of muscle cell membranes b. Absence of adenosine triphosphate (ATP) for muscle contraction c. Delayed muscle contraction d. Hyperactive reflexes

A

Of the following groups, who are at highest risk for a cerebrovascular accident (CVA)? a. Blacks over 65 years of age b. Whites over 65 years of age c. Blacks under 65 years of age d. Whites under 65 years of age

A

The breathing pattern that reflects respirations based primarily on carbon dioxide (CO2) levels in the blood is: a. Cheyne-Stokes b. Ataxic c. Central neurogenic d. Normal

A

The patient is experiencing an increase in intracranial pressure. This increase results in: a. Brain tissue hypoxia b. Intracranial hypotension c. Ventricular swelling d. Expansion of the cranial vault

A

Transchondral fractures are most prevalent in: a. Adolescents b. Older adults c. Infants d. Premenopausal females

A

When a student asks what can cause dilated, fixed pupils, what is the nurses best response? Dilated fixed pupils can be caused by: a. Brainstem hypoxia b. Cerebral contusions c. Compression of the hypothalamus d. Spinal shock

A

Which information indicates the nurse understands osteocalcin? Osteocalcin is a: a. Glycoprotein b. Hormone c. Mineral d. Vitamin

A

a 12-year-old presents with hydrocephalus. blockage of which of the following would cause this condition? a. cerebral aqueduct b. inferior colliculi c. red nucleus d. tegmentum

A

a 45-year-old male was previously diagnosed with parkinson disease. he has impaired fine repetitive motor movements. which of the following areas does the nurse suspect in most likely damaged? a. basal ganglia b. prefrontal area c. hippocampus d. temporal lobe

A

a nurse is discussing the membrane that separates the cerebellum from the cerebrum. what term should the nurse use to describe this membrane? a. tentorium cerebelli b. falx cerebri c. arachnoid membrane d. temporal lobe

A

a student nurse asks the nurse what controls reflex activities concerned with heart rate and blood pressure. what is the nurse's best response? these reflex activities are controlled by the: a. medulla oblongata b. pons c. midbrain d. cerebrum

A

an aide asks a nurse what neurotransmitters interact with. which response is the nurses best answer? neurotransmitters interact with the postsynaptic membrane by binding to a: a. receptor b. nissl body c. glial cell d. neurofibril

A

cerebrospinal fluid (CSF) can accumulate around the brain when there is injury to the sites of CSF reabsorption, which are called the: a. arachnoid villi b. epidural foramina c. lateral apertures d. choroid plexuses

A

when a patient's vagus nerve is stimulated, what does the nurse expect to observe? a. increased gastrointestinal activity b. increased heart rate c. pupil constriction d. vasoconstriction

A

when a students asks in which region of the neuron do nerve impulses travel the fastest, how should the nurse respond? the: a. large axon b. axon hillock c. cell body d. dendrites

A

which cell type is the nurse discussing? a cell that is involved in forming the blood-brain barrier is the: a. microglia b. schwann cell c. oligodendrocyte d. astrocyte

D

A 20-year-old male is admitted to the neurological critical care unit with a severe closed head injury. When an intraventricular catheter is inserted, the ICP is recorded at 24 mm Hg. How should the nurse interpret this reading? This reading is: a. Higher than normal b. Lower than normal c. Normal d. Borderline

A

A 21-year-old female presents with low back pain and stiffness that is alleviated by physical activity. She was diagnosed with ankylosing spondylitis (AS). The first joint to be affected would be the: a. Sacroiliac b. Carpal c. Shoulder d. Knee

A

A 25-year-old male was in an automobile accident. At impact, his forehead struck the windshield. In this situation, a nurse recalls the coup injury would occur in the _____ region. a. Frontal b. Temporal c. Parietal d. Occipital

A

What problem should the nurse assess for in a patient with chronic hyperparathyroidism? a. Seizure disorders b. Vitamin D malabsorption c. Hyponatremia d. Osteoporosis and pathologic fractures

D

A patient with upper body obesity also has central fat distribution. This body fat configuration places the patient at greater risk for _____________ than a patient with lower body obesity. a. osteoporosis b. renal disease c. cardiometabolic disorders d. chronic anemia

C

which action will occur when a patient's-1 receptors are stimulated? a. dilation of the coronary arteries b. vasoconstriction of arteries c. increase in the strength of myocardial contaction d. decrease in the rate of myocardial contraction

C

which answer indicates a nurse understands regeneration of neurons? neurons that have the capacity for regeneration include: a. unmyelinated neurons in the brain b. myelinated neurons in the spinal cord c. myelinated peripheral neurons d. postganglionic motor neurons

C

which neurotransmitter is released when a patient's parasympathetic motor neurons are stimulated? a. epinephrine b. serotonin c. acetylcholine d. substance P

C

An 80-year-old male presents with skeletal pain and tenderness, especially in the hips. He is diagnosed with osteomalacia caused by: a. Collagen breakdown in the bone matrix b. Excessive bone resorption c. Crowding of bone marrow by excessive bone growth d. Inadequate bone mineralization

D

An adult has hydrocephalus. When the patient asks the nurse what caused this, how should the nurse respond? Hydrocephalus in adults is most often caused by: a. Overproduction of CSF b. Intercellular edema c. Elevated arterial blood pressure d. Defective CSF reabsorption

D

An important and useful clinical measure in the diagnosis of rhabdomyolysis is measurement of which laboratory value? a. White blood cell count b. Antinuclear antibodies c. Aspartate aminotransferase d. Creatine kinase (CK)

D

Fibromyalgia is a chronic musculoskeletal disorder characterized by: a. Pain resulting from joint and muscle inflammation b. Muscle pain in the back and gastrointestinal symptoms c. Neurologic pain in the skeletal muscles d. Diffuse pain, fatigue, and tender points

D

Natural appetite suppression mechanisms, necessary for food intake control, include : a. ketoacid deficiency. b. cholecystokinin storage. c. decreased blood glucose. d. leptin receptor stimulation.

D

The majority of intervertebral disk herniations occur between which vertebral levels (cervical, C; thoracic, T; lumbar, L; sacral, S)? a. C1 to C3 b. T1 to T4 c. T12 to L3 d. L4 to S1

D

The progress notes read: the cerebellar tonsil has shifted through the foramen magnum due to increased pressure within the posterior fossa. The nurse would identify this note as a description of _____ herniation. a. Supratentorial b. Central c. Cingulated gyrus d. Infratentorial

D

When the physical therapist is describing a freely movable joint, which term should be used? a. Synarthrosis b. Amphiarthrosis c. Fibrous d. Diarthrosis

D

Which of the following assessments should be prioritized in the care of a patient with anorexia nervosa? a. Serum electrolyte levels b. Chest auscultation c. White blood cell count with differential d. Blood pressure monitoring

D

1. Genetic disorders that involve a single gene trait are characterized by: A) multifactorial gene mutations. B) chromosome rearrangements. C) Mendelian patterns of transmission. D) abnormal numbers of chromosomes.

C

11. A woman who is a carrier for which of the following diseases possesses the greatest likelihood of passing the disease to her future children when heterozygous pairing exists? A) Phenylketonuria (PKU) B) Tay-Sachs disease C) Neurofibromatosis D) Cystic fibrosis

C

A nurse is preparing to teach the staff about bones. Which information should the nurse include? After puberty, the epiphyseal plate calcifies, and the epiphysis and _____ merge. a. Epiphyseal line b. Diaphyseal plate c. Metaphysis d. Articular cartilage

C

A nurse recalls direct stimulation of the insulin-secreting cells of the pancreas by the autonomic nervous system is an example of _____ control. a. Negative feedback b. Positive feedback c. Neural d. Substrate-level dependent

C

A patient has excessive movement. What disorder will the nurse see documented on the chart? a. Hypokinesia b. Akinesia c. Hyperkinesia d. Dyskinesia

C

A patient has high levels of hormones. To adapt to the high hormone concentrations, the patients target cells have the capacity for: a. Negative feedback b. Positive feedback c. Down-regulation d. Up-regulation

C

A patient has memory loss of events that occurred before a head injury. What cognitive disorder does the nurse suspect the patient is experiencing? a. Selective memory deficit b. Anterograde amnesia c. Retrograde amnesia d. Executive memory deficit

C

A 12-year-old female hurts her ankle while playing basketball. Tests reveal that she tore a ligament. This condition is known as a: a. Sprain b. Strain c. Disunion d. Subluxation

A

A 51-year-old male experienced severe acute gouty arthritis. Which of the following is the most common trigger for the symptoms? a. Trauma b. Anemia c. High-fat foods d. Lack of exercise

A

A nurse recalls fibrous joints generally are: a. Rotating b. Slightly movable c. Freely movable d. Calcified

B

a nurse remembers the brain receives approximately ________% of the cardiac output. a. 80 b. 40 c. 20 d. 10

C

. Diabetes insipidus, diabetes mellitus (DM), and SIADH share which of the following assessment manifestations? a. Polyuria b. Edema c. Vomiting and abdominal cramping d. Thirst

D

7. Aneuploidy of the X chromosome can result in a monosomy or polysomy disorder. The manifestations of monosomy X, Turner syndrome, differ from polysomy X disorders in numerous ways that include: A) short-stature female individual.. B) mental retardation. C) enlarged breasts. D) early onset puberty.

A

8. Wasting syndrome, an AIDS-defining illness, is characterized by involuntary weight loss of at least 10% of baseline body weight in the presence of: A) diarrhea. B) hypermetabolism. C) weakness and fever. D) glucose intolerance.

A

. Which information indicates the nursing student has a good understanding between compact bone and spongy bone? The major difference between compact bone and spongy bone is the: a. Organization of the structural elements b. Location within the body c. Activating chemicals in each d. Types of minerals in the bone matrix

A

. Which of the following people is at highest risk for the development of gout? a. Men aged 40 to 50 years b. Premenopausal women c. Male adolescents d. Female children

A

. While planning care for a patient with an extradural hematoma, which principle should the nurse remember? The main source of bleeding in extradural (epidural) hematomas is: a. Arterial b. Venous c. Capillary d. Sinus

A

1. The mediators involved in type I hypersensitivity allergic responses are released from: A) mast cells. B) plasma cells. C) monocytes. D) arachidonic acid.

A

14. Which of the following practitioners is most likely to be of assistance in the early care of an infant with a cleft lip? A) Lactation consultant B) Respiratory therapist C) Occupational therapist D) Social worker

A

17. Which of the following would constitute a normal assessment finding in a neonate? A) Minimal or absent levels of IgA and IgM B) Absence of plasma cells in the lymph nodes and spleen C) Undetectable levels of all immunoglobulins D) Absence of mature B cells with normal T-cell levels and function

A

18. A woman who has just learned that she is pregnant for the first time has sought advice from her healthcare provider about the safe use of alcohol during pregnancy. What advice should the clinician provide to the woman? A) Its likely best to eliminate alcohol from your diet while youre pregnant. B) Moderation in alcohol use is critical while you are pregnant. C) You should limit yourself to a maximum of one drink daily while youre pregnant. D) You should drink no alcohol until you are in your second trimester.

A

19. Which of the following health problems may be identified by a TORCH screening test? A) Rubella and herpes B) Tenovaginitis and human papillomavirus C) Rhinovirus and Ormond disease D) Chlamydia and rickets

A

2. A genetically determined hypersensitivity to common environmental allergens causes ___________ reactions, such as: A) atopic; urticaria. B) autoimmune; diarrhea. C) IgM-mediated; infections. D) delayed; poison ivy rash:

A

4. Type III hypersensitivity immune responses can be harmful when immune complex deposits in tissue activate ___________ that can directly damage area tissues. A) inflammation B) autoantibodies C) cytotoxic cells D) immunoglobulins

A

5. Multifactorial inheritance disorders, such as cleft palate, are often caused by____________ during fetal development. A) multiple gene mutations B) dominant gene expression C) X-linked crossover problem D) polyploidy of chromosomes

A

6. The newborn has the distinctive physical features of trisomy 21, Down syndrome, which includes: A) upward slanting of eyes. B) large, protruding ears. C) thin lips and small tongue. D) long fingers with extra creases.

A

A 3-year-old male was diagnosed with congenital hypothyroidism. The parents ask the nurse if left untreated what will happen. What is the nurses best response? If left untreated, the child would have: a. Mental retardation and stunted growth b. Increased risk of childhood thyroid cancer c. Hyperactivity and attention deficit disorder d. Liver, kidney, and pancreas failure

A

A 30-year-old male presents to his primary care provider reporting visual disturbances. CT prolactinoma. Which of the following treatments would the nurse help implement? Administering: a. Dopaminergic agonists b. Calcium c. Insulin d. Radiation

A

A 34-year-old female was recently diagnosed with RA. Physical examination revealed that inflammation started in the: a. Synovial membrane b. Articular cartilage c. Subchondral bone d. Surrounding ligaments

A

A 35-year-old female with Graves disease is admitted to a medical-surgical unit. While the nurse is reviewing the lab tests, which results would the nurse expect to find? a. High levels of circulating thyroid-stimulating antibodies b. Ectopic secretion of thyroid-stimulating hormone (TSH) c. Low circulating levels of thyroid hormones d. Increased circulation of iodine

A

A 50-year-old male patient presents with polyuria and extreme thirst. He was given exogenous ADH. For which of the following conditions would this treatment be effective? a. Neurogenic diabetes insipidus b. Psychogenic diabetes insipidus c. Nephrogenic diabetes insipidus d. SIADH

A

A 54-year-old patient with pulmonary tuberculosis (lung infection) is evaluated for syndrome of inappropriate ADH secretion (SIADH). Which of the following electrolyte imbalances would be expected in this patient? a. Hyponatremia b. Hyperkalemia c. Hypernatremia d. Hypokalemia

A

A 55-year-old female is admitted to the medical unit for complications of long-term, poorly controlled type 2 DM. Which of the following would the nurse expect to find in addition to elevated glucose? a. Atherosclerosis b. Metabolic alkalosis c. Elevated liver enzymes d. Anemia

A

A 60-year-old male presents with swelling and pain in the knee. CT reveals a tumor of spongy bone. His diagnosis would be: a. Chondrosarcoma b. Rhabdomyoma c. Rhabdomyosarcoma d. Fibrosarcoma

A

A 70-year-old male presents with back pain, fever, and weight loss. He reports that he had a recent respiratory infection from which he thought he recovered. Tests revealed increased white blood cell count, and a diagnosis of endogenous osteomyelitis was made. The primary organism causing this condition is: a. Staphylococcus aureus b. Salmonella c. Mycobacterium d. Haemophilus influenza

A

A cause of hydrocephalus in subarachnoid hemorrhage is: a. Scarring of meninges and impairment of CSF resorption b. Choroid plexus injury c. Impairment of CSF flow through the ventricles d. Vasoconstriction related to carbon dioxide (CO2) changes in the cerebral circulation

A

A frail, 87-year-old female patient has been admitted to a hospital after a fall and has been diagnosed with failure to thrive. Which of the following laboratory values would suggest that the patient may be experiencing malnutrition? a. Low prealbumin b. High C-reactive protein c. High bilirubin d. Low fasting blood sugar

A

A large, high-calorie meal has resulted in the intake of far more energy than a person requires. What will the individuals body do with the excess carbohydrates provided by this meal? a. Convert them into glucose and store them in the liver and muscles b. Excrete most of the excess polysaccharides through the kidneys c. Convert the carbohydrates into amino acids in preparation for long-term storage d. Create structural proteins from some of the carbohydrates and store the remainder as triglycerides

A

A neurologist is teaching about brain injuries. Which information should the neurologist include? The most severe diffuse brain injury caused by rotational acceleration is most likely to be located _____ the brainstem. a. More peripheral to b. In the central portion of c. Throughout d. Distal to

A

A nurse is discussing endocrine system dysfunction with a patient. Which statement indicates the patient understood? Endocrine system dysfunction can result from hyposecretion, hypersecretion, or from: a. Abnormal receptor activity b. Abnormal hormone levels c. Increased synthesis of second messengers d. Extracellular electrolyte alterations

A

A nurse is preparing to teach a patient about Addison disease. Which information should the nurse include? The most common cause of Addison disease is: a. An autoimmune reaction b. Dietary deficiency of sodium and potassium c. Cancer d. Viral infection of the pituitary gland

A

A nurse is preparing to teach about nerves. Which information should the nurse include? The axon leaves the cell body at the: a. axon hillock b. nissl body c. node of ranvier d. myelin sheath

A

A nurse is reviewing lab results. Which of the following lab results would slow down the rate of parathyroid hormone secretion? a. Increased serum calcium levels b. Decreased serum calcium levels c. Decreased levels of TSH d. Increased levels of TSH

A

A nurse notes that a patient walks with the leg extended and held stiff, causing a scraping over the floor surface. What type of gait is the patient experiencing? a. Spastic gait b. Cerebellar gait c. Basal ganglion gait d. Scissors gait

A

A nursing student wants to know during which stage actin binds to myosin. What is the nurses best response? During: a. Coupling b. Relaxation c. Discharging d. Excitation

A

A patient begins taking a new drug that causes pupil dilation, vasoconstriction, decreased gastrointestinal motility, and goosebumps. Which of the following receptors are activated? a. Alpha 1 b. Alpha 2 c. Beta 1 d. Beta 2

A

A patient has paralysis of both legs. What type of paralysis does the patient have? a. Paraplegia b. Quadriplegia c. Infraparaplegia d. Paresthesia

A

A patient has researched lipid-soluble hormones on the Internet. Which information indicates the patient has a good understanding? Lipid-soluble hormone receptors cross the plasma membrane by: a. Diffusion b. Osmosis c. Active transport d. Endocytosis

A

A nurse recalls neural systems basic to cognitive functions include _____ systems. (Select all that apply.) a. Attentional b. Memory and language c. Affective d. Sensory and motor e. Tactile

A, B, C

The pattern of bone destruction indicative of an aggressive malignant tumor is known as J_____ pattern. (Select all that apply.) a. Moth-eaten b. Permeative c. Geographic d. Radial e. Selective

A, B, C

A 35-year-old female presents with impaired motor function and visual disturbances. The diagnosis is Paget disease. What additional symptoms would be expected? (Select all that apply.) a. Skull thickness b. Dementia c. Deafness d. Headache e. Hypertension

A, B, C, D

The nursing student has a good understanding of glycoproteins when he states that the following glycoproteins are present in the bone? (Select all that apply.) a. Osteocalcin b. Sialoprotein c. Bone albumin d. Alpha-glycoprotein e. Collagen

A, B, C, D

Which information indicates the staff member has a good understanding of bone classifications? The shapes that represent bone classifications include (select all that apply): a. Long b. Flat c. Short d. Irregular e. Round

A, B, C, D

The nurse has a good understanding of bone matrix when he comments that the following molecules are part of the bone matrix? (Select all that apply.) a. Collagen fibers b. Elastin fibers c. Proteoglycans d. Glycoproteins e. Epithelial cells

A, B, D

Which of the following is a neuroglial cell? (Select all that apply.) a. Astrocyte b. Oligodendrocyte c. Neuron d. Ependymal cell e. Melanocyte

A, B, D

. Signs and symptoms of subluxation include (select all that apply): a. Pain b. Swelling c. Hemorrhage d. Limitation of movement e. Joint deformity

A, B, D, E

A 29-year-old male was recently diagnosed with AS. He is interested in obtaining more information about his disease. Patient teaching would include which of the following? (Select all that apply.) a. A diagnosis is made from history, physical examination, x-rays, and genetic analysis. b. Inflammation of the fibrocartilage in cartilaginous joints results in the erosion of bone structure, scar tissue formation, and joint fusion. c. The more common signs and symptoms of early disease include restricted joint movement and increased pain after physical activity. d. The usual treatment includes anti-inflammatory and analgesic medications, exercises, and physical therapy. e. The spine becomes bent forward as the normal convex curve of the lower spine diminishes.

A, B, D, E

The nurse is explaining clinical manifestations of alterations in the extrapyramidal system. The nurse would correctly include (select all that apply): a. Little or no paralysis of voluntary movement b. Normal or slightly increased tendon reflexes c. Positive (present) Babinski d. Presence of tremor e. Rigidity in muscle tone

A, B, D, E

A patient has researched compact bones on the Internet. Which information indicates the patient has a good understanding of compact bone? The structures present in compact bone include the (select all that apply): a. Haversian canals b. Trabeculae c. Canaliculi d. Lamellae e. Lacunae

A, C, D, E

When thought content and arousal level are intact but a patient cannot communicate, the patient has: a. Cerebral death b. Locked-in syndrome c. Dysphagia d. Cerebellar motor syndrome

B

19. A kidney has a glomerular capillary hydrostatic pressure of 50 mm Hg, a Bowman capsule hydrostatic pressure of 15 mm Hg, and a glomerular capillary oncotic pressure of 12 mm Hg. The net filtration pressure is ____ mm Hg. a. 23 b. 27 c. 35 d. 38

ANS: A /aAdd the opposing factors: a Bowman capsule hydrostatic pressure of 15 mm Hg, and a glomerular capillary oncotic pressure of 12 mm Hg equals 27 and subtract it from the promoting factor of 50 mm Hg, for a total of 23. Cognitive Level: Comprehension 732

2. A nurse is explaining the function of the heart. Which is a correct response by the nurse? A function of the pericardium is to: a. Provide a barrier against extra cardial infections. b. Improve blood flow through the heart. c. Play a role in cardiac conduction. d. Assist in cardiac contraction.

ANS: A A function of the pericardium is to provide a barrier against extra cardial infections. The pericardium does not improve blood flow through the heart as it is on the outside. The pericardium does not have a role in cardiac conduction. The inner portions of the heart control this. The pericardium does not assist in contraction. The muscular layers assist with this. Cognitive Level: Comprehension 552

21. An 8-year-old female presents with edema of the cutaneous and mucosal tissue layers. Her mother reports that the condition is recurrent and seems to occur more often during stressful situations. The child is diagnosed with hereditary angioedema. Which of the following is deficient in this child? a. C1 esterase inhibitor b. Carboxypeptidase c. Neutrophils d. Plasmin

ANS: A A genetic defect in C1 esterase inhibitor (C1 INH deficiency) results in hereditary angioedema. Hereditary angioedema is due to C1 esterase inhibitor. Carboxypeptidase degrades kinins. Hereditary angioedema is due to C1 esterase inhibitor, not a disorder of neutrophils. Plasmin is not associated with hereditary angioedema, but is associated with clots. Cognitive Level: Comprehension 124

1. A patient has a tissue growth that was diagnosed as cancer. Which of the following terms best describes this growth? a. Neoplasm b. Lipoma c. Meningioma d. Hypertrophy

ANS: A A neoplasm is a cancerous growth. Lipomas are benign growths. A meningioma is a benign tumor. Hypertrophy refers to tissue overgrowth, but not cancer. Cognitive Level: Comprehension 222

2. A 20-year-old male was recently diagnosed with lactose intolerance. He eats an ice cream cone and develops diarrhea. His diarrhea can be classified as _____ diarrhea. a. Osmotic b. Secretory c. Hypotonic d. Motility

ANS: A A nonabsorbable substance in the intestine leads to osmotic diarrhea. Infections lead to secretory diarrhea. Hypotonic diarrhea is not a form of diarrhea. Food is not mixed properly, digestion and absorption is impaired, and motility is increased leading to motility diarrhea. Cognitive Level: Comprehension 896

18. A cardiologist is teaching about the period that follows depolarization of the myocardium and represents a period during which no new cardiac potential can be propagated. What is the cardiologist describing? a. Absolute refractory b. Hyperpolarization c. Resting d. Threshold

ANS: A A refractory period is the time during which no new cardiac action potential can be initiated by a stimulus. It follows depolarization. A refractory period is the time, not hyperpolarization, during which no new cardiac action potential can be initiated by a stimulus. It follows depolarization. Cardiac potential does rest, but the period is termed the refractory period. Threshold is related to depolarization. Cognitive Level: Comprehension 559

26. Self-antigens do not meet the requirements for immunogenicity because they lack which of the following? a. Foreignness b. Size c. Quantity d. Virulence

ANS: A A self-antigen that fulfills all the criteria listed above except foreignness does not normally elicit an immune response. A self-antigen fulfills all the criteria listed above except foreignness. They possess size. A self-antigen fulfills all the criteria listed above except foreignness. They possess quantity. Virulence is not a factor. Cognitive Level: Comprehension 145

16. A 32-year-old female presents with lower leg pain, with swelling and redness. While obtaining the patients history, which finding could have caused her condition? a. Venous thrombus b. Heart valve damage c. Bacterial infection d. Atherosclerosis

ANS: A A thrombus formation in the vein leads to inflammation that may cause pain and redness with obstruction. Increased pressure in the vein behind the clot may lead to edema of the extremity. Heart valve damage may lead to thrombus, but it is not the cause of the patients symptoms. The patient is experiencing a venous thrombus that leads to the swelling, redness, and pain. A bacterial infection would not cause these localized specific symptoms. Atherosclerosis causes narrowing, primarily of arteries, but this is a venous thrombus that is causing the patients symptoms. REF: pp. 592-593

22. A nurse is preparing to teach about the collecting ducts. Reabsorption of water in the collecting ducts requires which of these hormones? a. Antidiuretic hormone (ADH) b. Atrial natriuretic factor (ANP) c. Renin d. Aldosterone

ANS: A ADH increases water permeability and reabsorption in the last segment of the distal tubule and along the entire length of the collecting ducts. ANP inhibits secretion of renin, inhibits angiotensin-induced secretion of aldosterone, relaxes vascular smooth muscle, and inhibits sodium and water absorption by kidney tubules. Renin is secreted to raise blood pressure. Aldosterone regulates water and sodium balance. Cognitive Level: Comprehension 734

18. A 39-year-old female with chronic intermittent pain in the epigastric area 2 to 3 hours after eating is diagnosed with a duodenal ulcer. Which of the following behaviors may have contributed to the development of the ulcer? a. Cigarette smoking b. Drinking caffeinated beverages c. Consuming limited fiber d. Antacid consumption

ANS: A Acid production is stimulated by cigarette smoking. Caffeinated beverages do not contribute to ulcer formation. Fiber is important, but consuming limited fiber will not contribute to ulcer formation. Antacids may relieve pain, but they do not contribute to ulcer formation. Cognitive Level: Comprehension 904

24. Which patient will develop active immunity? A patient who: a. Has natural exposure to an antigen or receives an immunization b. Receives preformed antibodies or T cells from a donor c. Has T cells that become B cells d. Receives immunoglobulin

ANS: A Active immunity occurs after either natural exposure to an antigen or after immunization. Active immunity occurs after either natural exposure to an antigen or after immunization, not with preformed antibodies. Active immunity occurs after either natural exposure to an antigen or after immunization, not when T cells become B cells. Active immunity occurs after either natural exposure to an antigen or after immunization, not when the patient receives immunoglobulins. Cognitive Level: Comprehension 144

40. In alcoholic cirrhosis, hepatocellular damage is caused by: a. Acetaldehyde accumulation b. Bile toxicity c. Acidosis d. Fatty infiltrations

ANS: A Alcoholic cirrhosis is caused by the toxic effects of alcohol metabolism on the liver. Alcohol is transformed to acetaldehyde, and excessive amounts significantly alter hepatocyte function and activate hepatic stellate cells, a primary cell involved in liver fibrosis. Bile toxicity does not cause alcoholic cirrhosis. Acidosis does not cause alcoholic cirrhosis. Fatty infiltrations do not cause alcoholic cirrhosis.

29. A patient has searched on the internet for muscle contractions. Which information indicates a good understanding? Muscle contractions occur when there is a decreased: a. Distance between Z lines b. A band length c. Z line length d. H zone distance

ANS: A Anatomically, contraction occurs when the sarcomere shortens, so adjacent Z lines move closer together. Anatomically, contraction occurs when the sarcomere shortens, so adjacent Z lines move closer together, not a decrease in A band length. Anatomically, contraction occurs when the sarcomere shortens, so adjacent Z lines move closer together, not a decrease in Z line length. Anatomically, contraction occurs when the sarcomere shortens, so adjacent Z lines move closer together, not a decrease in H zone distance. Cognitive Level: Comprehension 561

1. A primary care provider is talking about plasma proteins synthesized by lymphocytes in the lymph nodes. What is the primary care provider describing? a. Antibodies b. Albumins c. Clotting factors d. Complement proteins

ANS: A Antibodies are produced by plasma cells in the lymph nodes and other lymphoid tissues. Albumins are specific types of plasma proteins. Clotting factors describe a function of plasma proteins. Complement proteins are a type of plasma proteins. Cognitive Level: Comprehension 480

3. A 6-year-old female is diagnosed with a bacterial infection of the respiratory system. Which of the following will most likely try to fight the antigen? a. Antibodies b. Cytotoxic T cells c. Self-antigens d. Helper T cells

ANS: A Antibodies are produced by plasma cells that mature from lymphocytes, called B lymphocytes (B cells), in response to an antigen. Antibodies are produced by plasma cells that mature from lymphocytes, called B lymphocytes (B cells), in response to an antigen. Cytotoxic T cells do not respond to antigens. Antibodies are produced by plasma cells that mature from lymphocytes, called B lymphocytes (B cells), in response to an antigen. Self-antigens do not respond to antigens. Antibodies are produced by plasma cells that mature from lymphocytes, called B lymphocytes (B cells), in response to an antigen. Helper T cells do not respond to antigens. Cognitive Level: Comprehension 147 | p. 149

5. An immunologist has isolated a molecule from the human body to study. It is found to react with antibodies on B cells. Further testing reveals that the molecule is large and chemically diverse. What type of molecule is the immunologist studying? a. Antigen b. Immunogen c. Epitope d. Paratope

ANS: A Antibodies react with antigens. Antibodies react with antigens, not immunogen. Antibodies react with antigens, not epitopes. Antibodies react with antigens, not paratopes. Cognitive Level: Comprehension 147

6. A 10-year-old female is arriving at a national spelling bee contest. Her heart starts beating faster and harder, and she begins to sweat. Which of the following is she experiencing? a. Anticipatory response b. Homeostasis c. Reactive response d. Exhaustion stage

ANS: A Anticipation of experiencing these events produces a physiologic stress response. Homeostasis is a steady state. Reactive response occurs following a stressful event; the student is anticipating a stressful event. Exhaustion stage is the final stage of the stressful response. Cognitive Level: Comprehension 207

16. A staff member wants to know where the greatest proportion of iron is located. How should the nurse respond? The greatest proportion of total body iron is located in the: a. Erythrocytes b. Spleen pulp c. Bone marrow d. Liver tissue

ANS: A Approximately 67% of total body iron is bound to heme in erythrocytes (hemoglobin) and muscle cells (myoglobin). Approximately 67% of total body iron is bound to heme in erythrocytes (hemoglobin) and muscle cells (myoglobin), not the spleen pulp. Approximately 67% of total body iron is bound to heme in erythrocytes (hemoglobin) and muscle cells (myoglobin), not the bone marrow. Approximately 67% of total body iron is bound to heme in erythrocytes (hemoglobin) and muscle cells (myoglobin), not the liver tissue. Cognitive Level: Comprehension 487

34. A 60-year-old female with a history of alcoholism complains of recent weight gain and right flank pain. Physical examination reveals severe ascites. This condition is caused by decreased: a. Albumin and lack of cellular integrity b. Capillary filtration pressure c. Capillary permeability d. Antidiuretic hormone secretion

ANS: A Ascites is due to decreased albumin and lack of cellular integrity. Ascites is due to decreased albumin and lack of cellular integrity, not capillary filtration pressure. Ascites is due to decreased albumin and lack of cellular integrity, not decreased capillary permeability. Ascites is due to decreased albumin and lack of cellular integrity, not antidiuretic hormone secretion. Cognitive Level: Comprehension 916

25. A person is given an attenuated antigen as a vaccine. When the person asks what was given in the vaccine, how should the nurse respond? The antigen is: a. Alive, but less infectious b. Mutated, but highly infectious c. Normal, but not infectious d. Inactive, but infectious

ANS: A Attenuated vaccines are alive, but less infectious. Attenuated vaccines are alive, not mutated or highly infectious. Attenuated vaccines are normal, but mutated doesnt relate to the answer. Attenuated vaccines are alive, but less infectious; they are not inactive. Cognitive Level: Comprehension 177

27. A nurse recalls the glomerular filtration rate (GFR) and plasma creatinine (Pcr) concentration are _____ related. a. Directly b. Indirectly c. Inversely d. Not

ANS: C The GFR and Pcr are inversely related. The and Pcr are inversely, not directly, related. The GFR and Pcr are inversely, not indirectly, related. The GFR and Pcr are inversely related. Cognitive Level: Comprehension 737

2. After teaching the students about B cells, which statement indicates teaching was successful? B cells are originally derived from cells of the: a. Bone marrow b. Lymph nodes c. Gut-associated lymphoid tissue d. Thymus

ANS: A B cells are derived from the bone marrow. B cells are derived from the bone marrow, not the lymph nodes. B cells are derived from the bone marrow, not the gut-associated lymphoid tissue. B cells are derived from the bone marrow, not the thymus. Cognitive Level: Comprehension 147

24. When bacterial pathogens enter a patients body, they can defend themselves from an immune response by: a. Producing capsules b. Phagocytosis c. Retreating d. Developing antibodies

ANS: A Bacterial survival and growth depend on the effectiveness of the bodys defense mechanisms and on the bacteriums ability to resist these defenses. Many pathogens devise ways of preventing destruction by the inflammatory and immune systems by producing thick capsules of carbohydrate or protein. Bacteria do not produce phagocytosis. Bacteria do not retreat. Bacteria do not develop antibodies. Cognitive Level: Comprehension 168

10. A 19-year-old female was involved in a motor vehicle accident during which she sustained a closed head injury. She is now experiencing detrusor sphincter dyssynergia. Which of the following is the most beneficial medication treatment? a. Alpha blocker b. Beta blocker c. Vasodilator d. Diuretic

ANS: A Because the bladder neck consists of circular smooth muscle with adrenergic innervation, detrusor sphincter dyssynergia may be managed by alpha-adrenergic blocking (antimuscarinic) medications. Because the bladder neck consists of circular smooth muscle with adrenergic innervation, detrusor sphincter dyssynergia may be managed by alpha-adrenergic blocking (antimuscarinic) medications, not beta blockers. Because the bladder neck consists of circular smooth muscle with adrenergic innervation, detrusor sphincter dyssynergia may be managed by alpha-adrenergic blocking (antimuscarinic) medications, not a vasodilator. Because the bladder neck consists of circular smooth muscle with adrenergic innervation, detrusor sphincter dyssynergia may be managed by alpha-adrenergic blocking (antimuscarinic) medications, not a diuretic. Cognitive Level: Comprehension 746

25. When phagocytes begin to stick avidly to capillary walls, which process is occurring? a. Margination b. Exudation c. Integration d. Emigration

ANS: A Both leukocytes and endothelial cells secrete substances that increase adhesion, or stickiness, causing the leukocytes to adhere more avidly to the endothelial cells in the walls of the capillaries and venules in a process called margination. Exudation is the process of pus formation and does not result in stickiness. Integration occurs in cells but is not a major function and does not lead to stickiness. Emigration is similar to diapedesis and is not associated with increased stickiness. Cognitive Level: Comprehension 131

18. A patient has researched bradykinin on the Internet. Which information indicates the patient understands the functions of bradykinin? Bradykinin is involved in: a. Increasing vascular permeability b. Vasoconstricting blood vessels c. Stimulating the clotting system d. Increasing degradation of prostaglandins

ANS: A Bradykinin increases vascular permeability. Bradykinin increases vascular permeability; it does not promote vasoconstriction. Bradykinin increases vascular permeability; it does not stimulate clotting. Bradykinin promotes pain; thus, it does not degrade prostaglandins but supports them. Cognitive Level: Comprehension 124

21. A cell that is produced in the thymus and interacts with MHC class II molecules would have which of the following surface proteins? a. CD4 b. CD8 c. Th1 d. Superantigen

ANS: A CD4-positive Th cells are restricted to interactions with cells presenting antigen by MHC class II molecules. CD4-positive Th, not CD8, cells are restricted to interactions with cells presenting antigen by MHC class II molecules. CD4-positive Th, not Th1, cells are restricted to interactions with cells presenting antigen by MHC class II molecules. CD4-positive Th, not superantigen, cells are restricted to interactions with cells presenting antigen by MHC class II molecules. Cognitive Level: Comprehension 158

25. To help a nursing student differentiate cardiac muscle from skeletal muscle, which characteristic should the nurse use? One difference between cardiac muscle and skeletal muscle is that: a. Cardiac muscle cells are arranged in branching networks. b. Skeletal muscle cells have only one nucleus. c. Cardiac muscle cells appear striped. d. Skeletal muscle cells contain sarcomeres.

ANS: A Cardiac cells are arranged in branching networks throughout the myocardium, whereas skeletal muscle cells tend to be arranged in parallel units throughout the length of the muscle. Cardiac muscle cells have only one nucleus, whereas skeletal muscle cells have many nuclei. Both cardiac and muscle cells appear striped. Both cardiac and muscle cells contain sarcomeres. Cognitive Level: Comprehension 560

19. A 45-year-old male presents with oliguria. He is diagnosed with chronic glomerulonephritis. The nurse knows oliguria is related to: a. Thickening of the glomerular membrane and decreased renal blood flow b. Increased glomerular capillary oncotic pressure and tubular obstruction c. Activation of renin-angiotensin from decreased blood volume d. Vasoconstriction of the efferent arterioles

ANS: A Changes in the glomerulus are characterized by progressive thickening and fibrosis of the glomerular basement membrane. Changes in the glomerulus are characterized by progressive thickening and fibrosis of the glomerular basement membrane; there is not tubular obstruction. The renin-angiotensin system is not activated in glomerulonephritis. The efferent arterioles are not affected in glomerulonephritis. Cognitive Level: Comprehension 751

10. The directional migration of leukocytes along a chemical gradient is termed: a. Chemotaxis b. Endocytosis c. Margination d. Diapedesis

ANS: A Chemotaxis is the process by which leukocytes undergo directed migration. Endocytosis is a form of engulfment and a part of phagocytosis. Margination occurs when leukocytes adhere to endothelial cells in the walls of vessels. Diapedesis is the emigration of the cells through cell junctions that have loosened in response to inflammatory mediators. Cognitive Level: Comprehension 131

22. Clinical manifestations of bile salt deficiencies are related to poor absorption of: a. Fats and fat-soluble vitamins b. Water-soluble vitamins c. Proteins d. Minerals

ANS: A Clinical manifestations of bile salt deficiency are related to poor intestinal absorption of fat and fat-soluble vitamins (A, D, E, K). Water-soluble vitamins do not require bile salts for absorption; thus, they are not affected. Protein breakdown is facilitated by bile, but its absorption is not dependent upon it; thus, the correct answer is the fat soluble vitamins. Absorption of minerals do not require bile salts for absorption; thus, they are not affected. Cognitive Level: Comprehension 908

22. Which of the following nutritional components will the nurse encourage a patient to consume as it is needed for erythropoiesis? a. Cobalamin b. Vitamin B1 c. Vitamin D d. Zinc

ANS: A Cobalamin is necessary for erythropoiesis. Cobalamin or vitamin B12 is necessary for erythropoiesis, not B1. Cobalamin is necessary for erythropoiesis, not vitamin D. Cobalamin is necessary for erythropoiesis, not zinc. Cognitive Level: Comprehension 487

13. A 30-year-old male develops poison ivy after mowing the lawn near the woods. His primary care provider prescribes a synthetic cortisol. How will the cortisol relieve his symptoms? a. Suppressing Th1 cell activity b. Increasing cytokine secretion c. Suppressing natural killer cell activity d. Increasing Th2 activity

ANS: A Cortisol suppresses Th1 responses. Cortisol does not increase cytokine secretion. Cortisol does not suppress natural killer cell activity. Cortisol suppresses Th2 activity. Cognitive Level: Comprehension 213

24. A 60-year-old male is diagnosed with renal failure. While the nurse is reviewing lab results, which of the following lab values would be most consistent with this diagnosis? a. Elevated plasma creatinine level b. Decreased plasma potassium level c. Metabolic alkalosis d. Increased urea clearance

ANS: A Creatinine is constantly released from muscle and excreted primarily by glomerular filtration. In chronic kidney disease (CKD), as glomerular filtration rate (GFR) declines, the plasma creatinine level increases by a reciprocal amount to maintain a constant rate of excretion. As GFR continues to decline, plasma creatinine concentration increases. Creatinine is elevated; potassium is also elevated. Metabolic acidosis develops. Decreased urea clearance occurs. Cognitive Level: Comprehension 756

13. Digested fats are primarily absorbed into the: a. Lacteals b. Central arterioles c. Lamina propria d. Villus capillaries

ANS: A Digested fats move into the lacteals and eventually reach the liver through the systemic circulation. Digested fats move into the lacteals and eventually reach the liver through the systemic circulation. They are not absorbed by central arterioles. Digested fats move into the lacteals and eventually reach the liver through the systemic circulation. They are not absorbed by the lamina propria. Digested fats move into the lacteals and eventually reach the liver through the systemic circulation. They are not absorbed by the villus capillaries. Cognitive Level: Comprehension 878

11. During an infection, the nurse assesses the lymph nodes. Lymph nodes enlarge and become tender because: a. Lymphocytes are rapidly dividing. b. Edema accumulates within the fibrous capsule. c. Microorganisms are accumulating. d. The nodes are not functioning properly.

ANS: A During an infection, the rate of proliferation of lymphocytes within the nodes is so great that the nodes enlarge and become tender. Edema may be present, but the tenderness is due to proliferation of lymphocytes. The tenderness is not due to the microorganisms, but to the proliferation of lymphocytes. The tenderness is due to the normal function of the gland in response to the proliferation of lymphocytes. Cognitive Level: Comprehension 482

26. An immunologist is discussing endotoxin production. Which information should the immunologist include? Endotoxins are produced by: a. Gram-negative bacteria b. Gram-positive bacteria c. Gram-negative fungi d. Gram-positive fungi

ANS: A Endotoxins are produces by gram-negative bacteria. Endotoxins are produced primarily by gram-negative bacteria, not gram-positive. Endotoxins are produced primarily by gram-negative bacteria, not gram-negative fungi. Endotoxins are produced primarily by gram-negative bacteria, not gram-positive fungi. Cognitive Level: Comprehension 171

14. A 30-year-old male with HIV is diagnosed with Epstein-Barr virus. After 2 months, the virus is still active. Based upon the Epstein-Barr virus, which of the following cancers is most likely to develop in this patient? a. B cell lymphoma b. Kaposi sarcoma c. T cell leukemia d. T cell lipoma

ANS: A Epstein-Barr virus is associated with B cell lymphoma. Kaposi sarcoma is associated with HIV. Retroviruses are associated with leukemia. Lipomas are not associated with HIV. Cognitive Level: Comprehension 241

8. A 25-year-old female was diagnosed with cervical cancer. History reveals she had many sexual partners, which she indicates is too many to count. Which of the following is most likely to have caused her cancer? a. HPV-16 b. HPV-18 c. HPV-31 d. HPV-45

ANS: A HPV-16, in most countries, accounts for 50% to 60% of cervical cancer cases. HPV-18 accounts for 10% to12% of cervical cancer cases. HPV-31 accounts for 4% to 5% of cervical cancer cases. HPV-45 accounts for 4% to 5% of cervical cancer cases. Cognitive Level: Comprehension 282

24. A 35-year-old hypertensive male begins taking a diuretic. Which of the following common side effects of this medication should the nurse monitor? a. Hypokalemia b. Hyponatremia c. Increased uric acid secretion d. Hypermagnesemia

ANS: A Hypokalemia is a side effect of diuretics. Hypokalemia, not hyponatremia, is a side effect of diuretic therapy. Diuretics promote uric acid retention, not excretion. Hypokalemia is a side effect of diuretics. Hypermagnesemia is not. Cognitive Level: Comprehension 736

8. An immunology nurse is caring for a patient. While planning care, which principle will the nurse remember? The primary role of IgA1 is to prevent infections in the: a. Blood b. Kidneys c. Lungs d. Mucous membranes

ANS: A IgA1 is found predominantly in the blood. IgA1 is found predominantly in the blood, not the kidneys. IgA1 is found predominantly in the blood, not the lungs. IgA1 is found predominantly in the blood, not the mucous membranes. Cognitive Level: Comprehension 147

6. Which statement indicates a correct understanding of antibodies? The most abundant class of antibody in the serum is: a. IgG b. IgM c. IgA d. IgE

ANS: A IgG is the most abundant class of immunoglobulins, constituting 80% to 85% of the immunoglobulins in the blood. IgG is the most abundant class of immunoglobulins, not IgM. IgG is the most abundant class of immunoglobulins, not IgA. IgG is the most abundant class of immunoglobulins, not IgE. Cognitive Level: Comprehension 147

11. Which of the following indicates a nurse understands a proto-oncogene? A proto-oncogene is best defined as a(n) _____ gene. a. Normal b. Altered c. Inactive d. Tumor-suppressor

ANS: A In its normal, nonmutant state, an oncogene is referred to as a proto-oncogene. A proto-oncogene is a normal gene, not an altered gene. A proto-oncogene is a normal gene, not an inactive gene. A proto-oncogene is a normal gene, not a tumor-suppressor gene. Cognitive Level: Comprehension 229

30. A 3-year-old is making play cakes in a sandbox and is eating the play cakes. The sand was also being used by cats as a litter box and was contaminated with toxoplasmosis. Which of the following would most likely also be present? a. Granuloma formation b. Degranulation c. Blood clots d. Exudate production

ANS: A Infections caused by bacteria such as toxoplasmosis can result in granuloma formation. Degranulation is a part of mast cell destruction. Blood clots are not expected with chronic inflammation. Exudate production is pus formation. Cognitive Level: Comprehension 134

26. An infant develops a fever secondary to a bacterial infection. Which of the following most likely triggered the fever? a. Interleukin-1 b. Interleukin-6 c. Interleukin-10 d. Interferons (INFs)

ANS: A Interleukin-1 is responsible for fever production. Interleukin-6 stimulates growth and differentiation of blood cells. Interleukin-10 helps decrease the immune response. INFs are members of a family of cytokines that protect against viral infections. Cognitive Level: Comprehension 126

22. A 21-year-old woman was recently diagnosed with iron deficiency anemia. Her hematocrit is 32%. Which of the following treatments would the nurse expect to be prescribed for her? a. Iron replacement b. Splenectomy c. A bone marrow transplant d. No treatment is necessary

ANS: A Iron replacement therapy is required and very effective. Initial doses are 150 mg to 200 mg/day and are continued until the serum ferritin level reaches 50 mg/L. Splenectomy is not indicated. Iron replacement therapy is required and very effective. Initial doses are 150 mg to 200 mg/day and are continued until the serum ferritin level reaches 50 mg/L. A bone marrow transplant is not indicated. Iron replacement therapy is required and very effective. Initial doses are 150 mg to 200 mg/day and are continued until the serum ferritin level reaches 50 mg/L. Treatment is indicated. Iron replacement therapy is required and very effective. Initial doses are 150 mg to 200 mg/day and are continued until the serum ferritin level reaches 50 mg/L. Cognitive Level: Comprehension 504

10. A 45-year-old female undergoes a splenectomy to remove a tumor. Which of the following assessment finding is most likely to occur following surgery? a. Leukocytosis b. Hypoglycemia c. Decreased red blood cell count d. Decreased platelets

ANS: A Leukocytosis often occurs after splenectomy. Elevated levels of leukocytes, not hypoglycemia, occur after splenectomy. Elevated levels of leukocytes, not decreased levels of red blood cells, occur after splenectomy. Elevated levels of leukocytes, not decreased platelets, occur after splenectomy. Cognitive Level: Comprehension 482

8. A 52-year-old male with hepatitis C recently developed hepatic cancer. Which of the following markers should be increased? a. Alpha-fetoprotein (AFP) b. Catecholamines c. Prostate-specific antigen d. Homovanillic acid

ANS: A Liver and germ cell tumors secrete a protein known as AFP. Liver and germ cell tumors secrete a protein known as AFP, not catecholamines. Prostate tumors secrete prostate-specific antigen. Homovanillic acid is a catecholamine marker. Cognitive Level: Comprehension 225

7. A nurse recalls that monocytes are blood cells that mature (differentiate) into: a. Macrophages b. Neutrophils c. Eosinophils d. Mast cells

ANS: A Macrophages are mature monocytes. Monocytes mature into macrophages, not neutrophils. Monocytes mature into macrophages, not eosinophils. Monocytes mature into macrophages, not mast cells. Cognitive Level: Comprehension 481

5. Most cases of combined systolic and diastolic hypertension have no known cause and are documented on the chart as _____ hypertension. a. Primary b. Secondary c. Congenital d. Acquired

ANS: A Most cases of hypertension are diagnosed as primary hypertension. Most cases of hypertension are diagnosed as primary hypertension, not secondary, which is due to a known cause. Most cases of hypertension are diagnosed as primary hypertension, not congenital. Most cases of hypertension are diagnosed as primary hypertension, not acquired. Cognitive Level: Comprehension 587

3. An adult patients blood sample is analyzed in a laboratory. Assuming a normal sample, which type of white blood cell accounts for the highest percentage? a. Neutrophils b. Eosinophils c. Basophils d. Lymphocytes

ANS: A Neutrophils constitute about 55% of the total leukocyte count in adults. The eosinophils, which have large, coarse granules, constitute only 1% to 4% of the normal leukocyte count in adults. Basophils make up less than 1% of the leukocytes. Lymphocytes constitute approximately 36% of the total leukocyte count. Cognitive Level: Comprehension 480

28. Which of the following characteristics is associated with an acute occlusion of mesenteric blood flow to the small intestine? a. Often precipitated by an embolism b. Commonly associated with disease such as pancreatitis and gallstones c. Caused by chronic malnutrition and mucosal atrophy d. Often a complication of hypovolemic shock

ANS: A Occlusion of blood flow is often precipitated by embolism. Occlusion of blood flow is often precipitated by embolism; it is not associated with pancreatitis. Occlusion of blood flow is often precipitated by embolism, even if chronic malnutrition is present. Occlusion of blood flow is often precipitated by embolism; it is not a complication of hypovolemic shock. Cognitive Level: Comprehension 911

10. A 65-year-old male was recently diagnosed with cancer. He is retired from construction work. Which of the following cancers is he likely to develop secondary to occupational hazards? a. Mesothelioma b. Bladder cancer c. Prostate cancer d. Bone cancer

ANS: A One notable occupational factor is asbestos, which increases the risk of mesothelioma, lung cancer, and possibly others. The cancer secondary to occupational hazards is mesothelioma, not bladder cancer. The cancer secondary to occupational hazards is mesothelioma, not prostate cancer. The cancer secondary to occupational hazards is mesothelioma, not bone cancer. Cognitive Level: Comprehension 278

27. A 57-year-old male presents to his primary care provider for red face, hands, feet, ears, and headache and drowsiness. A blood smear reveals an increased number of erythrocytes, indicating: a. Polycythemia vera (PV) b. Leukemia c. Sideroblastic anemia d. Hemosiderosis

ANS: A PV is manifested by increased numbers of erythrocytes. PV is characterized by excessive proliferation of erythrocyte precursors in the bone marrow. Leukemia is not manifested by changes in erythrocytes. PV is characterized by excessive proliferation of erythrocyte precursors in the bone marrow. Sideroblastic anemia is characterized by sideroblastic rings. PV is characterized by excessive proliferation of erythrocyte precursors in the bone marrow. Hemosiderosis is iron overload. Cognitive Level: Comprehension 506

Which of the following clinical findings would be expected in the patient with rhabdomyolysis? a. Sweating b. Dark urine c. Yellow color to the skin d. Lower extremity swelling

B

34. A 20-year-old female has an increase in eosinophils. When the patient wants to know the most likely cause of the eosinophilia. What is the nurses best response? a. Parasitic invasion and allergic reactions b. Viral and bacterial infections c. Stress and anxiety reactions d. Fungal infections and delayed hypersensitivity

ANS: A Parasitic invasion can lead to eosinophilia. Viral infections do not lead to eosinophilia; parasitic invasions do. Stress and anxiety reactions do not lead to eosinophilia; parasitic invasions do. Fungal infections do not lead to eosinophilia; parasitic invasions do. Cognitive Level: Comprehension 509

15. A 42-year-old female presents with abdominal discomfort, epigastric tenderness, and bleeding. Gastroscopy reveals degeneration of the gastric mucosa in the body and fundus of the stomach. Which of the following would most likely follow? a. Pernicious anemia b. Osmotic diarrhea c. Increased acid secretion d. Decreased gastrin secretion

ANS: A Pernicious anemia can develop because the damage to the mucosa makes the intrinsic factor less available to facilitate vitamin B12 absorption in the ileum. Osmotic diarrhea would not occur as a portion of the damage to the mucosa, but pernicious anemia could. Increased acid secretion would not occur as a portion of the damage to the mucosa, but pernicious anemia could. Decreased gastrin secretion would not occur as a portion of the damage to the mucosa, but pernicious anemia could. Cognitive Level: Comprehension 903

16. When a patients renal system secretes rennin, what effect will that cause in the body? It causes the direct activation of: a. Angiotensin I b. Angiotensin II c. Antidiuretic hormone d. Aldosterone

ANS: A Renin secretion activates angiotensin I. Renin secretion activates angiotensin I; ACE activates angiotensin II. Renin secretion activates angiotensin I; it does not activate antidiuretic hormone. Renin secretion activates angiotensin I; it does not activate aldosterone. Cognitive Level: Comprehension 730

15. When a nurse cares for a patient with systemic lupus erythematosus (SLE), the nurse remembers this disease is an example of: a. Autoimmunity b. Alloimmunity c. Homoimmunity d. Alleimmunity

ANS: A SLE is the most common, complex, and serious of the autoimmune disorders. SLE is an autoimmune disorder, not alloimmune. SLE is an autoimmune disorder not homoimmune. SLE is an autoimmune disorder not alleimmune. Cognitive Level: Comprehension 197

4. While planning care, a nurse recalls seasonal allergic rhinitis is expressed through: a. IgE-mediated reactions b. Tissue-specific reactions c. Antigen-antibody complexes d. Type II hypersensitivity reactions

ANS: A Seasonal allergic rhinitis is expressed through IgE-mediated reactions. Tissue-specific reaction is an autoimmune reaction. Antigen-antibody reactions are type III and are organ specific. Type II hypersensitivity reactions are tissue specific. Cognitive Level: Comprehension 190

14. A hematologist is discussing hematopoiesis. Which information should be included? ________ participate in hematopoiesis. a. Colony-stimulating factors (CSFs) b. Eosinophils c. Basophils d. Neutrophils

ANS: A Several cytokines participate in hematopoiesis, particularly CSFs (or hematopoietic growth factors). Several cytokines participate in hematopoiesis, particularly CSFs (or hematopoietic growth factors); eosinophils do not play a role in hematopoiesis. Several cytokines participate in hematopoiesis, particularly CSFs (or hematopoietic growth factors); basophils do not play a role in hematopoiesis. Several cytokines participate in hematopoiesis, particularly CSFs (or hematopoietic growth factors); neutrophils do not play a role in hematopoiesis. Cognitive Level: Comprehension 485

23. A 45-year-old male is diagnosed with sideroblastic anemia. When he asks what the most likely cause of this disease is, what is the nurses best response? a. Ineffective iron uptake and abnormal hemoglobin production b. Misshapen erythrocytes with low hemoglobin c. Decreased levels of tissue iron with megaloblastic erythrocytes d. Premature erythrocyte destruction and erythropoietin deficiency

ANS: A Sideroblastic anemia is due to ineffective iron uptake and hemoglobin production. Erythrocytes are not misshapen in sideroblastic anemia; iron uptake is altered. Iron is increased in sideroblastic anemia. Erythrocytes are not destroyed in sideroblastic anemia; alteration in iron uptake occurs. Cognitive Level: Comprehension 505

14. A 25-year-old female has sexual relations with her boyfriend. Later she is told that the boyfriend is infected with Neisseria gonorrhoeae. Testing reveals that she does not have the disease. How is it possible that she did not contract the disease? a. Antibodies covered sites of attachment. b. She was vaccinated against it. c. Platelets provided protection. d. IgE was released.

ANS: A Some bacteria, such as Neisseria gonorrhoeae that causes gonorrhea, must attach to specific sites on urogenital epithelial cells. Antibodies may protect the host by covering sites on the microorganism that are needed for attachment, thereby preventing infection. The patient did not contract the disease because antibodies covered sites of attachment, not because of vaccination. The patient did not contract the disease because antibodies covered sites of attachment, not because of platelets. The patient did not contract the disease because antibodies covered sites of attachment, not because of IgE release. Cognitive Level: Comprehension 150

29. A 51-year-old male is at the health clinic for an annual physical exam. After walking from the car to the clinic, he developed substernal pain. He also reported discomfort in his left shoulder and his jaw, lasting 2 to 3 minutes and then subsiding with rest. He indicates that this has occurred frequently over the past few months with similar exertion. The nurse suspects he is most likely experiencing: a. Stable angina b. Unstable angina c. Prinzmetal angina d. Myocardial infarction (MI)

ANS: A Stable angina is associated with activity and subsides with rest. Unstable angina is a form of acute coronary syndrome that results from reversible myocardial ischemia and occurs at rest. Chest pain that occurs at rest and at night is descriptive of Prinzmetal angina. MI pain does not subside with rest. Cognitive Level: Comprehension 601

4. The nursing student is teaching a class to second graders. The student would correctly identify that food moves down the esophagus via the process termed: a. Peristalsis b. Retropulsion c. Haustral segmentation d. Defecation

ANS: A Swallowed food is moved from the esophagus to the stomach by peristalsis. Swallowed food is moved by peristalsis, not retropulsion. Swallowed food is moved by peristalsis; haustral segmentation occurs in the small intestine. Swallowed food is moved by peristalsis; defecation is the process of eliminating stool from the rectum. Cognitive Level: Comprehension 873

Which of the following patients has the best chance of recovery from nerve injury? A patient with a(n): a. Torn nerve b. Crushed nerve c. Injury located closer to the cell body of the nerve d. Injury located closer to the synapse

B

26. If a nurse wants to obtain the best estimate of renal function, which test should the nurse monitor? a. Glomerular filtration rate (GFR) b. Circulating antidiuretic hormone (ADH) levels c. Volume of urine output d. Urine-specific gravity

ANS: A The GFR provides the best estimate of functioning renal tissue. The GFR provides the best estimate of functioning renal tissue. Circulating ADH levels is not the best indicator. The GFR provides the best estimate of functioning renal tissue, not urine output. The GFR provides the best estimate of functioning renal tissue. Specific gravity is not the best indicator. Cognitive Level: Comprehension 736

11. A 65-year-old male is transported to the ER for chest pain. An electrocardiogram reveals a prolonged QRS interval. What is the nurse monitoring when the nurse observes the QRS complex on the electrocardiogram? The QRS complex reflects: a. Ventricular activity b. Pulmonary artery closure c. Mitral valve opening d. Aortic valve closing

ANS: A The QRS complex represents the sum of all ventricular muscle cell depolarizations. The QRS complex represents the sum of all ventricular muscle cell depolarizations, not the pulmonary artery. The QRS complex represents the sum of all ventricular muscle cell depolarizations; it does not reflect mitral opening. The QRS complex represents the sum of all ventricular muscle cell depolarizations; it does not reflect activity of the aortic valve. Cognitive Level: Comprehension 559

16. How should the nurse prepare a patient who is to receive a Schilling test for pernicious anemia? a. Administer radioactive cobalamin and measure its excretion in the urine. b. Measure antigen-antibody immune complexes. c. Measure serum ferritin and total iron-binding capacity. d. Administer folate and evaluate folate content in a blood serum sample.

ANS: A The Schilling test is performed by administering radioactive cobalamin and then measuring its excretion in the urine. Low urinary excretion is significant for pernicious anemia (PA). The Schilling test is performed by administering radioactive cobalamin and then measuring its excretion in the urine. Low urinary excretion is significant for PA. It does not involve the measurement of antigen-antibody complexes. The Schilling test is performed by administering radioactive cobalamin and then measuring its excretion in the urine. Low urinary excretion is significant for PA. It does not involve the measurement of serum ferritin and total iron-binding capacity. The Schilling test is performed by administering radioactive cobalamin and then measuring its excretion in the urine. Low urinary excretion is significant for PA. It does not involve the administration of folate. Cognitive Level: Comprehension 503

3. A female student is driving to school when another driver nearly hits her. Her heart begins beating harder and faster as she becomes aroused and scared. Which of the following stages of the general adaptation syndrome is she experiencing? a. Alarm stage b. Stage of resistance c. Adaptation d. Exhaustion

ANS: A The alarm stage is manifested by arousal of the bodys defenses. The stage of resistance is the second step. Adaptation is a part of resistance. Exhaustion marks the breakdown of compensatory mechanisms. Cognitive Level: Comprehension 204

8. A cardiologist is discussing valves with the staff. Which information should the cardiologist include? _____ are the anchors of the atrioventricular valves. a. Chordae tendineae cordis b. Great vessels c. Coronary ostia d. Trabeculae carneae

ANS: A The atrioventricular valve openings are attached to the papillary muscles by the chordae tendineae cordis. The atrioventricular valve openings are attached to the papillary muscles by the chordae tendineae cordis. The great vessels are the vessels that bring blood to and out of the heart and are not attached to the chordae tendineae. The atrioventricular valve openings are attached to the papillary muscles by the chordae tendineae cordis, not the coronary ostia, which are openings in the aorta for the coronary arteries. The atrioventricular valve openings are attached to the papillary muscles by the chordae tendineae cordis, not the trabeculae carneae, which are a portion of the myocardium. Cognitive Level: Comprehension 554

9. A young male student becomes very hungry during class. He begins thinking of a cheeseburger and fries, his favorite meal. These thoughts will trigger which phase of gastric secretion? a. Cephalic b. Caudal c. Enteral d. Intestinal

ANS: A The cephalic phase is stimulated by the thought, smell, and taste of food. The gastric phase is stimulated by distention of the stomach. The enteral phase is not a phase of gastric secretion. The intestinal phase is stimulated by histamine and digested protein. Cognitive Level: Comprehension 876

16. The complement, clotting, and kinin systems share which of the following characteristics? a. Activation of a series of proenzymes b. Phagocytosis initiation c. Granulocyte production d. Activated by interferon

ANS: A The complement system, the clotting system, and the kinin system are normally in inactive forms, but can activate in a series as proenzymes and are involved in the inflammatory process. The complement system, the clotting system, and the kinin system do not play a role in phagocytosis, but do play a role in the inflammatory response as proenzymes. The complement system, the clotting system, and tje kinin system do not play a role in granulocyte production, but they function as proenzymes in the inflammatory response. The complement system, the clotting system, and the kinin system are not activated by interferon, but are activated by enzymatic action. Cognitive Level: Comprehension 122

4. Chromosome aberrations and mutations in cells that were not directly irradiated are referred to as: a. Bystander effects b. Lethal mutation c. Delayed reproductive death d. Genetic instability

ANS: A The directly irradiated cells also can lead to genetic effects in so-called bystander cells or innocent cells. This is termed bystander effects. Lethal mutations occur when cells cannot reproduce, and this is not related to bystander effects. Lethal mutation and delayed reproductive death are similar phenomenon and are not related to bystander effects. Genetic instability is related to chromosomal instability. Cognitive Level: Comprehension 274

3. A 20-year-old male received a knife wound to the arm during an altercation. Which of the following types of immunity was compromised? a. Innate immunity b. Inflammatory response c. Adaptive immunity d. Specific immunity

ANS: A The epithelial cells of the skin are a part of innate immunity. The inflammatory response is not a type of immunity. Adaptive immunity is represented by the normal flora of the bowel. Specific immunity is a type of adaptive immunity and is not associated with a break in skin integrity. Cognitive Level: Comprehension 120

13. While reviewing urine lab results, the nurse remembers the glomerular filtration rate (GFR) is directly related to the: a. Perfusion pressure in the glomerular capillaries b. Oncotic pressure in the glomerular capillaries c. Vascular resistance in the glomerular arterioles d. Hydrostatic pressure in the Bowman capsule

ANS: A The filtration of the plasma per unit of time is known as the GFR, which is directly related to the perfusion pressure of the glomerular capillaries. The filtration of the plasma per unit of time is known as the GFR, which is directly related to the perfusion pressure, not the oncotic pressure of the glomerular capillaries. The filtration of the plasma per unit of time is known as the GFR, which is directly related to the perfusion pressure in the glomerular capillaries, not the vascular resistance in the glomerular arterioles. The filtration of the plasma per unit of time is known as the GFR, which is directly related to the perfusion pressure in the glomerular capillaries, not the hydrostatic pressure in the Bowman capsule. Cognitive Level: Comprehension 729

17. The _____ reflex initiates propulsion in the entire colon, usually during or immediately after eating. a. Gastrocolic b. Ileocolic c. Duodenocolic d. Cephalocolic

ANS: A The gastrocolic reflex initiates propulsion in the entire colon. The gastrocolic reflex, not the ileocolic, initiates propulsion in the entire colon. The gastrocolic reflex, not the duodenocolic, initiates propulsion in the entire colon. The gastrocolic reflex, not the cephalocolic, initiates propulsion in the entire colon. Cognitive Level: Comprehension 882

33. When trying to distinguish between an infectious disease and noninfectious disease, what is the hallmark symptom for most infectious diseases? a. Fever b. Jaundice c. Vomiting d. Pain

ANS: A The hallmark of most infectious diseases is fever. The hallmark of most infectious diseases is fever, not jaundice. The hallmark of most infectious diseases is fever, not vomiting. The hallmark of most infectious diseases is fever, not pain. Cognitive Level: Comprehension 175

14. Which reflex inhibits gastric motility when the ileum becomes overdistended? a. Ileogastric b. Gastroduodenal c. Gastroileal d. Colonointestinal

ANS: A The ileogastric reflex inhibits gastric motility when the ileum becomes distended. The ileogastric reflex, not the gastroduodenal, inhibits gastric motility when the ileum becomes distended. The ileogastric reflex, not he gastroileal, inhibits gastric motility when the ileum becomes distended. The ileogastric reflex, not the colonointestinal, inhibits gastric motility when the ileum becomes distended. Cognitive Level: Comprehension 881

3. The urologist is teaching about the nephrons that determine the concentration of the urine. The urologist is discussing the _____ nephrons. a. Juxtamedullary b. Midcortical c. Cortical d. Medullary

ANS: A The juxtamedullary nephrons lie close to and extend deep into the medulla and are important for the concentration of urine. The juxtamedullary nephrons, not the midcortical nephrons, are important for the concentration of urine. The juxtamedullary nephrons, not the cortical nephrons, are important for the concentration of urine. The juxtamedullary nephrons, not the medullary nephrons, are important for the concentration of urine. Cognitive Level: Comprehension 726

34. For a patient with respiratory acidosis, chronic compensation by the body will include: a. Kidney excretion of H+ b. Kidney excretion of HCO3 c. Prolonged exhalations to blow off CO2 d. Protein buffering

ANS: A The kidneys excrete H+ to compensate for respiratory acidosis. The kidneys do not excrete HCO3 to compensate; this would increase acidosis. Prolonged exhalations would not be effective for compensation, especially in a chronic state. Protein buffering is intracellular and will not be effective enough to compensate for respiratory acidosis. Cognitive Level: Comprehension 114

8. A 2-year-old malnourished child has vitamin B12 and folate deficiencies. A blood smear suggests the deficiency is macrocytic and normochromic. The nurse would expect the hemoglobin to be: a. Normal b. Sporadic c. Low d. High

ANS: A The macrocytic (megaloblastic) anemias are characterized by unusually large stem cells (megaloblasts) in the marrow that mature into erythrocytes that are unusually large in size (macrocytic), thickness, and volume. The hemoglobin content is normal, thus allowing them to be classified as normochromic. The hemoglobin content is normal, not sporadic. The hemoglobin content is normal, not low. The hemoglobin content is normal, not high. Cognitive Level: Comprehension 501

18. When teaching about sodium reabsorption, which information should the nurse include? The majority of sodium reabsorption takes place in the: a. Proximal tubule b. Loop of Henle c. Distal tubule d. Collecting duct

ANS: A The majority of sodium reabsorption takes place in the proximal tubule. The majority of sodium reabsorption takes place in the proximal tubule, not the loop of Henle. The majority of sodium reabsorption takes place in the proximal tubule, not the distal tubule. The majority of sodium reabsorption takes place in the proximal tubule, not the collecting duct. Cognitive Level: Comprehension 733

5. While planning care for a patient with renal calculi, the nurse remembers the most important factor in renal calculus formation is: a. urine pH. b. body temperature. c. gender. d. serum mineral concentrations.

ANS: A The most important factor in renal calculus formation is urine pH. The most important factor in renal calculus formation is urine pH, not body temperature. The most important factor in renal calculus formation is urine pH, not gender, although calculi form more often in men. The most important factor in renal calculus formation is urine pH, not serum mineral concentrations. Cognitive Level: Comprehension 743

6. A 25-year-old male nursing student recently learned how diet can alter the chances of developing cancer. He tries to minimize his risks of developing the disease by ordering his steak: a. Rare b. Medium c. Medium-well d. Well done

ANS: A The nursing student should order the steak rare, as the most relevant carcinogens produced by cooking are found in well-done charbroiled beef. The nursing student should order the steak rare, not medium, as the most relevant carcinogens produced by cooking are found in well-done charbroiled beef. The nursing student should order the steak rare, not medium-well, as the most relevant carcinogens produced by cooking are found in well-done charbroiled beef. The nursing student should order the steak rare, not well-done, as the most relevant carcinogens produced by cooking are found in well-done charbroiled beef. Cognitive Level: Comprehension 262

A 51-year-old male is admitted to the neurological critical care unit with a severe closed head injury. All four extremities are in rigid extension, his forearm is hyperpronated, and his legs are in plantar extension. How should the nurse chart this condition? a. Decorticate posturing b. Decerebrate posturing c. Caloric posturing d. Excitation posturing

B

14. A 29-year-old female presents with cloudy urine, flank pain, hematuria, and fever. Which of the following does the nurse suspect the patient is most likely experiencing? a. Acute cystitis b. Renal calculi c. Chronic renal failure d. Postrenal renal failure

ANS: A The patient is demonstrating symptoms of acute cystitis. Although renal calculi can cause pain and hematuria, it is not manifested by fever and cloudy urine. Chronic renal failure is not manifested by cloudy urine. Postrenal renal failure is not manifested by hematuria and cloudy urine. Cognitive Level: Comprehension 747

15. A 45-year-old male presents with persistent, severe stomach pain. Testing reveals a peptic ulcer. Further laboratory tests reveal the presence of Helicobacter pylori. Which of the following is of concern for this patient? a. Gastric cancer b. Leukemia c. Lung cancer d. Adenocarcinoma of the colon

ANS: A The presence of Helicobacter pylori is associated with gastric cancer. The presence of Helicobacter pylori is associated with gastric cancer, not leukemia. The presence of Helicobacter pylori is associated with gastric cancer, not lung cancer. The presence of Helicobacter pylori is associated with gastric cancer, not colon cancer. Cognitive Level: Comprehension 241

10. A patient has been researching telomere caps on the Internet. Which statement indicates the patient has a good understanding? Presence of telomere caps gives cancer cells: a. The ability to divide over and over again b. Clonal distinction c. Limited mitosis d. Mutation abilities

ANS: A The presence of telomere caps gives cancer cells the ability to divide over and over. The presence of telomere caps does not give cells clonal distinction. Telomeres allow division over and over; thus, cancer cells have unlimited mitosis. Mutation capability is a characteristic of cancer cells, but this property is not related to telomeres. REF: pp. 235-236

5. A nurse recalls the chamber that receives blood from the systemic circulation is the: a. Right atrium b. Right ventricle c. Left atrium d. Left ventricle

ANS: A The right atrium receives the blood from the systemic circulation. The right atrium receives the blood from the systemic circulation; the right ventricle receives blood from the right atrium. The right atrium receives the blood from the systemic circulation; the left atrium receives blood from the lungs. The right atrium receives the blood from the systemic circulation; the left ventricle receives blood from the left atrium. Cognitive Level: Comprehension 554

24. A 50-year-old female was diagnosed with sideroblastic anemia. Which of the following assessment findings would most likely occur? a. Bronze colored skin b. Decreased iron c. Normochromic erythrocytes d. Aplastic bone marrow

ANS: A The skin of the patient with sideroblastic anemia is bronze in color. The patient with sideroblastic anemia shows signs of iron overload. The erythrocytes of individuals with sideroblastic anemia are hypochromic. The bone marrow is not aplastic, but contains ringed sideroblasts. Cognitive Level: Comprehension 505

39. A 10-year-old male is diagnosed with leukemia. The nurse assesses for which other condition that could be associated with his disease? a. Down syndrome b. Hemophilia c. Hyperthyroidism d. Pheochromocytoma

ANS: A There is also an increased incidence of leukemia in association with other hereditary abnormalities such as Down syndrome. There is also an increased incidence of leukemia in association with other hereditary abnormalities such as Down syndrome, not hemophilia. There is also an increased incidence of leukemia in association with other hereditary abnormalities such as Down syndrome, not hyperthyroidism. There is also an increased incidence of leukemia in association with other hereditary abnormalities such as Down syndrome, not pheochromocytoma. Cognitive Level: Comprehension 513

9. A 45-year-old male is diagnosed with macrocytic, normochromic anemia. The nurse suspects the most likely cause of this condition is: a. Defective DNA synthesis b. Abnormal synthesis of hemoglobin c. Defective use of vitamin C d. Blocked protein synthesis

ANS: A These anemias are the result of ineffective erythrocyte deoxyribonucleic acid (DNA) synthesis. These anemias are the result of ineffective erythrocyte DNA synthesis; hemoglobin is normal. These anemias are the result of ineffective erythrocyte DNA synthesis; it is not related to use of vitamin C. These anemias are the result of ineffective erythrocyte DNA synthesis; it is not due to blocked protein synthesis. Cognitive Level: Comprehension 501

7. Which of the following compounds has been shown to increase the risk of cancer when used in combination with smoking? a. Alcohol b. Steroids c. Antihistamines d. Hypnotics

ANS: A Tobacco use and alcohol use are known etiologic factors in head and neck cancers. It is alcohol, not steroids, and tobacco that have been identified as etiologic factors in cancer. It is alcohol, not antihistamines, and tobacco that have been identified as etiologic factors in cancer. It is alcohol, not hypnotics, and tobacco that have been identified as etiologic factors in cancer. Cognitive Level: Comprehension 261

3. When a patient asks what types of cancers are associated with tobacco use, how should the nurse respond? a. Squamous and small cell adenocarcinomas b. Sarcoma and adenoma c. Melanoma and lymphoma d. Basal cell and lipoma

ANS: A Tobacco use is associated with squamous and small cell adenocarcinomas. Tobacco use is associated with squamous and small cell adenocarcinomas, not sarcoma and adenoma. Tobacco use is associated with squamous and small cell adenocarcinomas, not melanoma and lymphoma. Tobacco use is associated with squamous and small cell adenocarcinomas, not basal cell and lipoma. Cognitive Level: Comprehension 261

5. When the nurse discusses the glomerulus and Bowman capsule together, it is referred to as the renal: a. Corpuscle b. Capsule c. Medulla d. Pyramid

ANS: A Together, the glomerulus and Bowman capsule are called the renal corpuscle. Together, the glomerulus and Bowman capsule are called the renal corpuscle, not capsule. Together, the glomerulus and Bowman capsule are called the renal corpuscle, not medulla. Together, the glomerulus and Bowman capsule are called the renal corpuscle, not pyramid. Cognitive Level: Comprehension 726

30. A 67-year-old female is admitted to the emergency department with a diagnosis of polycythemia vera (PV). Which treatment should the nurse discuss with the patient? a. Therapeutic phlebotomy b. Restoration of blood volume by plasma expanders c. Administration of packed red blood cells d. Iron replacement therapy

ANS: A Treatment of PV is phlebotomy to reduce red cell mass and blood volume. Treatment of PV is phlebotomy to reduce red cell mass and blood volume; increasing blood volume will increase symptoms. Treatment of PV is phlebotomy to reduce red cell mass and blood volume; administration of additional cells will increase the problems. Treatment of PV is phlebotomy to reduce red cell mass and blood volume; iron replacement will increase the symptoms. Cognitive Level: Comprehension 507

24. A 50-year-old male complains of frequently recurring abdominal pain, diarrhea, and bloody stools. A possible diagnosis would be: a. Ulcerative colitis b. Hiatal hernia c. Pyloric obstruction d. Achalasia

ANS: A Ulcerative colitis is manifested by fever, elevated pulse rate, frequent diarrhea (10 to 20 stools/day), urgency, obviously bloody stools, and continuous lesions present in the colon. Hiatal hernia is most often asymptomatic and would not be manifested by abdominal pain. Pyloric obstruction would be manifested by forceful or projectile vomiting. Achalasia would be manifested by difficulty or uncomfortable swallowing. Cognitive Level: Comprehension 908

3. A 40-year-old female presents complaining of pain near the midline in the epigastrium. Assuming the pain is caused by a stimulus acting on an abdominal organ, the pain felt is classified as: a. Visceral b. Somatic c. Parietal d. Referred

ANS: A Visceral pain arises from a stimulus (distention, inflammation, ischemia) acting on an abdominal organ. Somatic is a form of parietal pain. Parietal pain, from the parietal peritoneum, is more localized and intense than visceral pain, which arises from the organs themselves. Referred pain is visceral pain felt at some distance from a diseased or affected organ. Cognitive Level: Comprehension 896

35. A 55-year-old female presents to her primary care provider and reports dizziness, confusion, and tingling in the extremities. Blood tests reveal an elevated pH, decreased PCO2, and slightly decreased HCO3. Which of the following is the most likely diagnosis? a. Respiratory alkalosis with renal compensation b. Respiratory acidosis with renal compensation c. Metabolic alkalosis with respiratory compensation d. Metabolic acidosis with respiratory compensation

ANS: A With an elevated pH, the diagnosis must be alkalosis. Since the PCO2 is low, it is likely respiratory with a slight decrease in HCO3 indicating renal compensation. With an elevated pH, the diagnosis must be alkalosis, not acidosis. With an elevated pH, the diagnosis must be alkalosis. Since the PCO2 is low, it is likely respiratory since the HCO3 is decreased. With an elevated pH, the diagnosis must be alkalosis. Cognitive Level: Comprehension 115

2. During childhood, the thymus decreases in size, and this is referred to as _____ atrophy. a. Physiologic b. Pathologic c. Disuse d. Neurogenic

ANS: A A normal decrease in cell size is physiologic atrophy. When cell size decrease due to disease, it is termed pathologic atrophy. When cells undergo disuse, the cells shrink in size, and this is disuse atrophy. Neurogenic atrophy is injury or disease affecting the nerves that connect to the muscle.

11. Sodium and water accumulation in an injured cell are a direct result of: a. Decreased ATP production b. Karyorrhexis c. Ribosome detachment d. Dehydration

ANS: A A reduction in ATP levels causes the plasma membranes sodium-potassium (Na+-K+) pump and sodium-calcium exchange to fail. Sodium and water can then enter the cell freely. Karyorrhexis means fragmentation of the nucleus into smaller particles or nuclear dust. Ribosome detachment reduces protein synthesis. Dehydration leads to loss of sodium and water.

16. A 50-year-old male sustained a closed head injury as a result of a motor vehicle accident. CT scan revealed a collection of blood between the inner surface of the dura mater and the surface of the brain. Which type of injury will the nurse be caring for? a. Subdural hematoma b. Epidural hematoma c. Contusion d. Abrasion

ANS: A A subdural hematoma occurs when blood is between the inner surface of the dura mater and the surface of the brain; it can result from blows, falls, or sudden acceleration/deceleration of the head. An epidural hematoma is a collection of blood between the inner surface of the skull and the dura; it is most often associated with a skull fracture. A contusion is bleeding into the skin or underlying tissues. An abrasion (scrape) results from removal of the superficial layers of the skin caused by friction between the skin and injuring object.

9. A 10-year-old male is brought to the emergency room (ER) because he is incoherent and semiconscious. CT scan reveals that he is suffering from cerebral edema. This type of edema is referred to as: a. Localized edema b. Generalized edema c. Pitting edema d. Lymphedema

ANS: A Cerebral edema is a form of localized edema. Generalized edema is manifested by a more uniform distribution of fluid in interstitial spaces. Pitting edema is due to a pit left in the skin. Lymphedema is due to swelling in interstitial spaces, primarily in the extremities. Cognitive Level: Comprehension 102

16. A couple has three offspring: one child with an autosomal dominant disease trait and two who are normal. The father is affected by the autosomal dominant disease, but the mother does not have the disease gene. What is the recurrence risk of this autosomal dominant disease for their next child? a. 50% b. 33% c. 25% d. Impossible to determine

ANS: A For each child with an autosomal dominant disease parent there is a 1 in 2, or 50%, risk. The risk is 50%. The risk is 50%. The risk can be determined and the risk is 50%.

33. An 86-year-old female patient has the wasting syndrome of aging, making her vulnerable to falls, functional decline, disease, and death. The nurse knows this patient is experiencing: a. Frailty b. Sarcopenia c. Somatic death d. Cellular aging

ANS: A Frailty leaves the individual vulnerable to falls, disease, and death. Sarcopenia is loss of muscle mass. It is associated with aging, but is not as severe as frailty. Somatic death is death of the whole person. Cellular aging occurs at the cellular level and is characteristic of aging, including cell atrophy, decreased function, and loss of cells.

19. When taking care of a patient with hyperkalemia, which principle is priority? Hyperkalemia causes a(n) _____ in resting membrane potential with _____ excitability of cardiac muscle. a. Increase; increased b. Decrease; increased c. Increase; decreased d. Decrease; decreased

ANS: A Hyperkalemia causes an increase in resting membrane potential and increased excitability of cardiac muscle. Hyperkalemia does cause an increased excitability of cardiac muscle, but the result is an increase in resting membrane potential. Hyperkalemia does cause an increase in resting membrane potential, but the result is an increase in excitability of cardiac muscle. Hyperkalemia causes an increase in resting membrane potential and increased excitability of cardiac muscle. Cognitive Level: Comprehension 110

23. A 52-year-old female is diagnosed with coronary artery disease. The nurse assesses for myocardial: a. Hypertrophy b. Ischemia c. Necrosis d. Inflammation

ANS: B Coronary artery disease leads to myocardial ischemia. Coronary artery disease would not lead to hypertrophy, but to ischemia. Coronary artery disease can contribute to necrosis, but obstruction by a clot would more likely lead to necrosis. Coronary artery disease would lead to myocardial ischemia, not inflammation. Cognitive Level: Comprehension 587

30. What principle should the nurse remember when trying to distinguish aging from diseases? a. It is difficult to tell the difference because both processes are believed to result from cell injury. b. It is easy to tell normal processes from abnormal processes. c. Disease, unlike aging, has a genetic component. d. Aging is defined as exceeding life expectancy, but not maximal life span.

ANS: A It is difficult to differentiate between aging and disease because both occur secondary to cellular aging. It is not easy to differentiate normal processes from abnormal because aging appears as a normal process. Disease and aging have a possible genetic component. Aging is a time-dependent loss of structure and function that proceeds slowly and in such small increments that it appears to be the result of the accumulation of small, imperceptible injuries. It is not a time period outside of life expectancy.

28. Physiologic pH is maintained around 7.4 because carbonic acid and bicarbonate exist in a ratio of: a. 20:1 b. 1:20 c. 10:2 d. 2:10

ANS: A Normal carbonic acid to bicarbonate ratio is 20:1. Normal carbonic acid to bicarbonate ratio is 20:1. Normal carbonic acid to bicarbonate ratio is 20:1. Normal carbonic acid to bicarbonate ratio is 20:1. Cognitive Level: Comprehension 110

3. When planning care for a cardiac patient, the nurse knows that in response to an increased workload, cardiac myocardial cells will: a. Increase in size b. Decrease in length c. Increase in excitability d. Increase in number

ANS: A Organ cells respond to increased workload by increasing in size, much as a muscle when it is exercised. Organ cells increase in size with increased workload; they do not decrease in length. A deficiency of electrolytes or minerals could lead to an increase in excitability; it is not due to increased workload. Increase in cell numbers is referred to as hyperplasia and is not related to increased workload.

5. Water movement between the ICF and ECF compartments is determined by: a. Osmotic forces b. Plasma oncotic pressure c. Antidiuretic hormone d. Buffer systems

ANS: A Osmotic forces determine water movement between the ECF and ICF compartments. Oncotic pressure pulls water at the end of the capillary, which makes it move between intra and extra as interstitial is considered extra. The antidiuretic hormone regulates water balance which would make water move between the intra and extra. Buffer systems help regulate acid balance. Cognitive Level: Comprehension 101

19. A 15-year-old female is diagnosed with Prader-Willi syndrome. This condition is an example of: a. Gene imprinting b. An autosomal recessive trait c. An autosomal dominant trait d. A sex-linked trait

ANS: A Prader-Willi is an example of gene imprinting. Prader-Willi is not an autosomal recessive trait, but due to gene imprinting. Prader-Willi is not an autosomal dominant train, but due to gene imprinting. Prader-Willi is not a sex-linked trait, but due to gene imprinting.

13. A 52-year-old male suffered a myocardial infarction secondary to atherosclerosis and ischemia. Once oxygen returned to the damaged heart, reperfusion injury occurred as a result of: a. Free radical formation b. Vacuolation c. Increased metabolic state d. Lipid acceptor proteins

ANS: A Reperfusion injury results from radicals that cause further membrane damage and mitochondrial calcium overload. Vacuolation leads to cellular swelling and is not associated with reperfusion. Lactic acid increases as a result of damage due to necrosis, and necrosis would interfere with reperfusion, not initiate it. Lipid acceptor proteins bind with triglycerides to create fatty liver, and they do not affect the myocardium.

16. The most common cause of pure water deficit is: a. Renal water loss b. Hyperventilation c. Sodium loss d. Insufficient water intake

ANS: A The most common cause of water loss is increased renal clearance of free water as a result of impaired tubular function. Hyperventilation can cause water loss, but it is not the most common cause. Sodium loss leads to hyponatremia, not pure water deficit. Insufficient water intake causes hypernatremia, not water deficit. Cognitive Level: Comprehension 106

33. A nurse is reviewing the pedigree chart. When checking for a proband, what is the nurse looking for? a. The person who is first diagnosed with a genetic disease b. The individual who has a disease gene but is phenotypically normal c. The phenotype of genetic material d. The codominance

ANS: A The pedigree chart summarizes family relationships and shows which members of a family are affected by a genetic disease. The pedigree begins with the proband. The person who has a disease gene but is phenotypically normal is a carrier. The phenotype is the result of both genotype and environment; it is not a proband. Codominance is not represented by proband, but it occurs when the heterozygote is distinguishable from both homozygotes.

1. When a staff member asks how erythrocytes can carry oxygen, which of the following properties should the nurse describe that allow erythrocytes to function as gas carriers? (Select all that apply.) a. Biconcavity b. Reversible deformability c. Undergoes mitotic division d. Presence of many mitochondria e. Presence of a nucleus

ANS: A, B Erythrocytes are small disks that are biconcave in shape and have the capacity to be reversibly deformed. They cannot undergo mitotic division and do not have many mitochondria. Erythrocytes do not have a nucleus. Cognitive Level: Comprehension 480

2. When a staff member asks how the urine gets from the nephrons to the calyces, what is the nurses best response? The renal structure that drains directly into the calyces is (are) the (select all that apply): a. Distal tubule b. Collecting duct c. Pyramid d. Renal pelvis e. Loop of Henle

ANS: A, B The urine formed by the nephrons flows from the distal tubules and collecting ducts through the duct of Bellini and the renal papillae into the calyces.

3. Direct effects of antibodies include (select all that apply): a. Neutralization b. Agglutination c. Precipitation d. Phagocytosis e. Division

ANS: A, B, C Directly, antibodies can affect infectious agents or their toxic products by neutralization (inactivating or blocking the binding of antigen to receptors), agglutination (clumping insoluble particles that are in suspension), or precipitation (making a soluble antigen into an insoluble precipitate). Indirectly, antibodies activate components of innate resistance, including complement and phagocytes. Antibodies are generally a mixed population of classes, specificities, and capacity to provide the functions listed above.

A 54-year-old female was recently diagnosed with degenerative joint disease. This condition is characterized by loss of: a. The epiphyses b. Articular cartilage c. Synovial fluid d. The joint capsule

B

2. A nurse is caring for an elderly patient. Which of the following are true regarding the hematological system and aging? (Select all that apply.) a. Total serum iron is decreased. b. Total iron-binding capacity is decreased. c. Intestinal iron absorption is decreased. d. Lymphocyte function is unchanged. e. Platelet aggregation is unchanged.

ANS: A, B, C Total serum iron, total iron-binding capacity, and intestinal iron absorption are all decreased somewhat in elderly persons. Lymphocyte function decreases with age. Platelet adhesiveness probably increases.

1. A 60-year-old male with a 30-year history of smoking is diagnosed with a hormone-secreting lung tumor. Further testing indicates that the tumor secretes ADH. Which of the following assessment findings should the nurse expect? (Select all that apply.) a. Confusion b. Weakness c. Nausea d. Muscle twitching e. Weight loss

ANS: A, B, C, D Secretion of ADH leads to water intoxication with symptoms of cerebral edema, with confusion and convulsions; weakness; nausea; muscle twitching; headache; and weight gain, not loss. Cognitive Level: Comprehension 107

1. When a student asks what the components of the nephron are, how should the nurse respond? The components of the nephron include (select all that apply): a. Loop of Henle b. Renal corpuscle c. Proximal tubule d. Renal pelvis e. Convoluted tubule

ANS: A, B, C, E The nephron is a tubular structure with subunits that include the renal corpuscle, proximal convoluted tubule, loop of Henle, distal convoluted tubule, and collecting duct, all of which contribute to the formation of final urine. Cognitive Level: Comprehension 726

2. The nurse would anticipate the patient with syndrome of inappropriate ADH (SIADH) to demonstrate which of the following symptoms? (Select all that apply.) a. Weakness b. Nausea c. Headache d. Weight loss e. Muscle twitching

ANS: A, B, C, E Weakness, nausea, muscle twitching, headache, and weight gain, not loss, are common symptoms of chronic water accumulation.

1. A 50-year-old female confirms chronic alcohol intake. This practice places the patient at risk for cancer in which organs? (Select all that apply.) a. Larynx b. Esophagus c. Liver d. Lung e. Brain f. Breast

ANS: A, B, C, F Chronic alcohol consumption is a strong risk factor for colorectal cancer and cancer of the oral cavity, pharynx, hypopharynx, larynx, esophagus, liver, and breast. It is not associated with lung or brain cancer.

2. When a patient asks about secondary lymph organs, how should the nurse respond? Secondary lymph organs include (select all that apply): a. Spleen b. Peyer patches c. Adenoids d. Liver e. Bone marrow f. Appendix

ANS: A, B, C, F The secondary lymphoid organs include the spleen, lymph nodes, adenoids, tonsils, Peyer patches (intestines), and the appendix. The liver and bone marrow are not secondary lymph organs. Cognitive Level: Comprehension 155

1. The patient asks the student to explain what the gastrointestinal (GI) tract actually does. The student would be correct in telling the patient the GI tract performs which of the following processes? (Select all that apply.) a. Absorption of digested food b. Chemical breakdown of food particles c. Erythropoiesis d. Mechanical breakdown of food particles e. Secretion of mucus

ANS: A, B, D, E The GI tract carries out the following process: (1) ingestion of food; (2) propulsion of food and wastes from the mouth to the anus; (3) secretion of mucus, water, and enzymes; (4) mechanical digestion of food particles; (5) chemical digestion of food particles; (6) absorption of digested food; and (7) elimination of waste products by defecation. Cognitive Level: Comprehension 871

2. When dealing with repression in patients, which of the following assessment findings are associated with repression? (Select all that apply.) a. Decreased monocyte counts b. Increased eosinophil counts c. Decreased serum glucose d. Increased pulse rates e. Increased medication reactions

ANS: A, B, E Repression was associated with lower monocyte counts, higher eosinophil counts, higher serum glucose, and more self-reported medication reactions in medical outpatients. It is not associated with increased pulse rates and glucose increases, rather than decreases.

2. A 70-year-old male reports frequent GI problems. His primary care provider explains that during aging many GI processes decrease, including (select all that apply): a. Gastric juice secretion b. Gallbladder function c. Hepatic regeneration d. Efficiency of drug metabolism e. Mucosal barrier protection

ANS: A, C, D, E GI processes that decrease include gastric motility, blood flow, and volume, and acid content of gastric juice may be reduced, particularly with gastric atrophy. Protective mucosal barrier decreases. Decreased hepatic regeneration occurs; size and weight decrease. Blood flow decreases, influencing efficiency of drug metabolism. No changes occur in the gallbladder and bile ducts.

1. When a nurse is teaching about urinary pathogens in men. Which information should the nurse include? Mechanisms for defense against urinary pathogens in men include (select all that apply): a. The long length of the urethra b. Alkaline pH of urine c. Secretion of mucus that traps bacteria d. Antimicrobial secretions from the prostate e. Implantation of the ureters in the bladder

ANS: A, D Both the longer urethra and prostatic secretions decrease the risk of infection in men. The urine is not more alkaline, and they do not secrete mucus that traps bacteria. The ureters in men and women are implanted in similar positions and in their normal position; it is not a factor in the development of cystitis.

15. A 25-year-old male is in a car accident and sustains a fracture to his left femur with extensive soft tissue injury. The pain associated with the injury is related to: a. Histamine and serotonin b. Kinins and prostaglandins c. Vasoconstriction d. Immune complex formation

ANS: B Prostaglandins cause increased vascular permeability, neutrophil chemotaxis, and pain by direct effects on nerves. Kinins also promote pain. Prostaglandins produce pain; histamine promotes vasodilation. Prostaglandins produce pain, not vasoconstriction. Prostaglandins produce pain, not the immune complex. Cognitive Level: Comprehension 129

27. While checking lab results, the nurse remembers the normal leukocyte count is: a. 1,000-2,000/mm3 b. 5,000-10,000/mm3 c. 4.2-6.2 million/mm3 d. 1.2-2.2 million/mm3

ANS: B 5,000-10,000/mm3 is the normal leukocyte count. 1,000-2,000/mm3 is an abnormally low leukocyte count. 4.2-6.2 million/mm3 is the normal erythrocyte count. 1.2-2.2 million/mm3 would indicate anemia. Cognitive Level: Comprehension 479

The final stage of gout, characterized by crystalline deposits in cartilage, synovial membranes, and soft tissue, is called: a. Monarticular arthritis b. Tophaceous gout c. Asymptomatic hyperuricemia d. Complicated gout

B

31. A 70-year-old male is brought to the emergency department, where he dies shortly thereafter. Autopsy reveals polycythemia vera (PV). His death was most likely the result of: a. Acute renal failure b. Cerebral thrombosis c. Sepsis d. Acute leukemia

ANS: B 50% of individuals with PV die within 18 months of the onset of initial symptoms because of thrombosis or hemorrhage. 50% of individuals with PV die within 18 months of the onset of initial symptoms because of thrombosis or hemorrhage, not renal failure. 50% of individuals with PV die within 18 months of the onset of initial symptoms because of thrombosis or hemorrhage, not infection and sepsis. 50% of individuals with PV die within 18 months of the onset of initial symptoms because of thrombosis or hemorrhage, not acute leukemia. Cognitive Level: Comprehension 508

19. A 22-year-old male underwent brain surgery to remove a tumor. Following surgery, he experienced a peptic ulcer. His ulcer is referred to as a(n) _____ ulcer. a. Infectious b. Cushing c. Ischemic d. Curling

ANS: B A Cushing ulcer is a stress ulcer associated with severe head trauma or brain surgery that results from decreased mucosal blood flow and hypersecretion of acid caused by overstimulation of the vagal nerve. The stress ulcer is termed Cushing, not infectious. The stress ulcer is termed Cushing, not ischemic. Curling ulcers develop secondary to burns. Cognitive Level: Comprehension 907

11. A nurse assesses a patient with a complicated urinary tract infection (UTI) for: a. Several species of bacteria b. Other health problems c. Urosepsis d. urethral obstruction

ANS: B A complicated UTI develops when there is an abnormality in the urinary system or a health problem that compromises host defenses or response to treatment. UTI may occur alone or in association with pyelonephritis, prostatitis, or kidney stones. A complicated UTI develops when there is an abnormality in the urinary system or a health problem that compromises host defenses or response to treatment. UTI may occur alone or in association with pyelonephritis, prostatitis, or kidney stones. It is not due to the involvement of several species of bacteria. A complicated UTI develops when there is an abnormality in the urinary system or a health problem that compromises host defenses or response to treatment. UTI may occur alone or in association with pyelonephritis, prostatitis, or kidney stones. It does not always cause urosepsis. A complicated UTI develops when there is an abnormality in the urinary system or a health problem that compromises host defenses or response to treatment. UTI may occur alone or in association with pyelonephritis, prostatitis, or kidney stones. It does not always result in obstruction. Cognitive Level: Comprehension 747

17. A 67-year-old female has chronic gastrointestinal bleeding. A nurse recalls the primary cause of her anemia is: a. Vitamin B12 deficiency b. Iron deficiency c. Folate deficiency d. Bone marrow failure

ANS: B A continuous loss of blood is one of the most common causes of iron deficiency anemia (IDA). A continuous loss of blood is one of the most common causes of IDA; gastrointestinal bleeding does not lead to vitamin B12 deficiency. A continuous loss of blood is one of the most common causes of IDA; gastrointestinal bleeding does not lead to folate deficiency. A continuous loss of blood is one of the most common causes of IDA; gastrointestinal bleeding does not lead to bone marrow failure. Cognitive Level: Comprehension 504

2. A newborn is diagnosed with congenital intrinsic factor deficiency. Which of the following types of anemia will the nurse see documented on the chart? a. Iron deficiency anemia b. Pernicious anemia c. Sideroblastic anemia d. Hemolytic anemia

ANS: B A lack of the intrinsic factor leads to pernicious anemia. Iron deficiency anemia is not related to the intrinsic factor. Pernicious anemia is due to the lack of the intrinsic factor, sideroblastic anemia is not related to the intrinsic factor. Hemolytic anemia results from destruction of cells; pernicious is related to the lack of the intrinsic factor. Cognitive Level: Comprehension 502

13. A nurse is discussing a cell that can differentiate into any tissue type. Which term is the nurse describing? a. Hematopoietic b. Pluripotent c. Blastocyst d. Progenitor

ANS: B A pluripotent cell can grow into different kinds of tissue: blood, nerves, heart, bone, and so forth. A pluripotent cell, not a hematopoietic cell, can grow into different kinds of tissue: blood, nerves, heart, bone, and so forth. A pluripotent cell can grow into different kinds of tissue: blood, nerves, heart, bone, and so forth. A blastocyst is an embryonic cell. A pluripotent cell can grow into different kinds of tissue: blood, nerves, heart, bone, and so forth. A progenitor becomes a hematologic cell. Cognitive Level: Comprehension 485

17. When a nurse observes poststreptococcal glomerulonephritis as a diagnosis on a patient, which principle will the nurse remember? Acute poststreptococcal glomerulonephritis is primarily caused by: a. Swelling of mesangial cells in the Bowman space in response to the presence of bacteria b. Antigen-antibody complex deposition in the glomerular capillaries and inflammatory damage c. Inflammatory factors that stimulate cellular proliferation of epithelial cells d. Accumulation of antiglomerular basement membrane antibodies

ANS: B Acute poststreptococcal glomerulonephritis is caused by an antigen-antibody complex. Acute poststreptococcal glomerulonephritis is not due to swelling of the Bowman space. Acute poststreptococcal glomerulonephritis is caused by an antigen-antibody complex. It is not an inflammatory process. Acute poststreptococcal glomerulonephritis is caused by an antigen-antibody complex. Cognitive Level: Comprehension 752

20. After erythrocytes have circulated for about 120 days, they are removed by macrophages, which are mainly in the: a. Liver b. Spleen c. Appendix d. Bone marrow

ANS: B Aged or damaged erythrocytes are removed from the bloodstream by macrophages of the MPS chiefly in the spleen. Aged or damaged erythrocytes are removed from the bloodstream by macrophages of the MPS chiefly in the spleen, not the liver. Aged or damaged erythrocytes are removed from the bloodstream by macrophages of the MPS chiefly in the spleen, not the appendix. Aged or damaged erythrocytes are removed from the bloodstream by macrophages of the MPS chiefly in the spleen, not the bone marrow. Cognitive Level: Comprehension 482

6. An oncologist is discussing when a cancer cell loses differentiation. Which of the following is the oncologist describing? a. Autonomy b. Anaplasia c. Pleomorphic d. Metastasis

ANS: B Anaplasia is the loss of differentiation. The loss of differentiation is anaplasia, not autonomy. Malignant cells are pleomorphic. A malignant tumors ability to spread far beyond the tissue of origin is metastasis. Cognitive Level: Comprehension 223

The nurse observed the patients muscle contracted, but the limb did not move. How should the nurse chart this muscle contraction? a. Isotonic contraction b. Isometric contraction c. Eccentric contraction d. Concentric contraction

B

27. A 51-year-old male presents with recurrent chest pain on exertion. He is diagnosed with angina pectoris. When he asks what causes the pain, how should the nurse respond? The pain occurs when: a. Cardiac output has fallen below normal levels. b. The myocardial oxygen supply has fallen below demand. c. Myocardial stretch has exceeded the upper limits. d. The vagus nerve is stimulated.

ANS: B Angina is chest pain caused by myocardial ischemia, which develops if the flow or oxygen content of coronary blood is insufficient to meet the metabolic demands of myocardial cells. A decrease in cardiac output would lead to general systemic symptoms, not just chest pain, which is due to a decrease in myocardial oxygenation. Myocardial stretch does not affect angina symptoms. When the vagus nerve is stimulated, a change in rate occurs; it does not precipitate chest pain. Cognitive Level: Comprehension 601

16. A 25-year-old male was diagnosed with Goodpasture syndrome. While planning care for this patient, which of the following mechanisms would cause tissue injury? a. Viral infection of the Bowman capsule b. Production of antibodies against the glomerular basement membrane c. Antigen-antibody complex deposition with complement activation d. Abnormal activation of clotting factors and microclotting in the glomerulus

ANS: B Antiglomerular basement membrane disease (Goodpasture syndrome) is associated with antibody formation against both pulmonary capillary and glomerular basement membranes. Antiglomerular basement membrane disease (Goodpasture syndrome) is associated with antibody formation against both pulmonary capillary and glomerular basement membranes. It is not due to a viral infection. Antiglomerular basement membrane disease (Goodpasture syndrome) is associated with antibody formation against both pulmonary capillary and glomerular basement membranes. It is not an antigen-antibody complex. Antiglomerular basement membrane disease (Goodpasture syndrome) is associated with antibody formation against both pulmonary capillary and glomerular basement membranes. It is not due to abnormal clotting. Cognitive Level: Comprehension 752

2. A patient wants to know what causes atherosclerosis. How should the nurse respond? In general, atherosclerosis is caused by: a. High serum cholesterol levels b. Endothelial injury and inflammation c. An increase in antithrombotic substances d. Congenital heart disease

ANS: B Atherosclerosis begins with injury to the endothelial cells that line artery walls. High serum cholesterol levels are associated with atherosclerosis, but are not its cause. Atherosclerosis begins with injury to the endothelial cells that line artery walls. It is not due to an increase in antithrombotic substances. Atherosclerosis begins with injury to the endothelial cells that line artery walls. It is not related to congenital heart disease. Cognitive Level: Comprehension 594

10. A 10-year-old male presents to his primary care provider reporting wheezing and difficulty breathing. History reveals that both of the childs parents suffer from allergies. Which of the following terms would be used to classify the child? a. Desensitized b. Atopic c. Hyperactive d. Autoimmune

ANS: B Atopic individuals are genetically predisposed to the development of allergies. Desensitization is a term used to describe the process of reducing the severity to allergies. Hypersensitivity is an altered immunologic response to an antigen that results in disease or damage to the individual. Autoimmunity is a disturbance in the immunologic tolerance of self-antigens. Cognitive Level: Comprehension 201

36. A 35-year-old male with hyperthyroidism begins treatment to decrease thyroid activity. A nurse monitors for which of the following conditions that could result secondary to the treatment? a. Eosinophilia b. Basophilia c. Monocytosis d. Lymphocytosis

ANS: B Basophilia results from antithyroid therapy. Basophilia, not eosinophilia, results from antithyroid therapy. Basophilia, not monocytosis, results from antithyroid therapy. Basophilia, not lymphocytosis, results from antithyroid therapy. Cognitive Level: Comprehension 509

31. A 13-year-old female confides to her mother that she binge eats and induces vomiting to prevent weight gain. This disease is referred to as: a. Anorexia nervosa b. Bulimia nervosa c. Long-term starvation d. Laxative abuse

ANS: B Binge eating and vomiting is characteristic of bulimia nervosa. Anorexia nervosa is starvation eating. Binge eating and vomiting is characteristic of bulimia nervosa, not long-term starvation. Binge eating and vomiting is characteristic of bulimia nervosa, not laxative abuse. Cognitive Level: Comprehension 914

20. In addition to matching ABO antigens, a blood transfusion must also be matched for: a. HLA type b. Rh antigen c. Immunoglobulins d. Platelet compatibility

ANS: B Blood transfusions must also be matched for the Rh antigen. Blood transfusions do not need to be matched to HLA. Blood transfusions do not need to be matched to immunoglobulins. Blood transfusions do not need to be matched to platelet compatibility. Cognitive Level: Comprehension 198

11. Stress-induced catecholamine release from the adrenal medulla may result in: a. Decreased blood flow to the brain b. Elevated blood pressure c. Increased glycogen synthesis in the liver d. Decreased muscle contraction resulting from energy depletion

ANS: B Catecholamine release results in elevated blood pressure. Catecholamine release results in increased blood flow to the brain. Catecholamine release results in increased glucose production, not glycogen synthesis. Catecholamine release results in increased muscle contraction. Cognitive Level: Comprehension 209

17. An experiment is designed to determine specific cell types involved in cell-mediated immune response. The experimenter is interested in finding cells that attack cells that have specific antigens. Which cells should be isolated? a. Lymphokine-producing cells b. Cytotoxic T cells c. Helper T cells d. Macrophages

ANS: B Cell-mediated immunity is driven by T-cytotoxic (Tc) cells that attack antigens directly and destroy cells that bear foreign antigens. Cell-mediated immunity is driven by T-cytotoxic (Tc) cells, not lymphokine-producing cells, that attack antigens directly and destroy cells that bear foreign antigens. Cell-mediated immunity is driven by T-cytotoxic (Tc) cells, not helper T cells, that attack antigens directly and destroy cells that bear foreign antigens. Cell-mediated immunity is driven by T-cytotoxic (Tc) cells, not macrophages, that attack antigens directly and destroy cells that bear foreign antigens. Cognitive Level: Comprehension 144

14. Chronic gastritis is classified according to the: a. Severity b. Location of lesions c. Patients age d. Signs and symptoms

ANS: B Chronic gastritis is classified as type A (fundal) or type B (antral), depending on the pathogenesis and location of the lesions. Gastritis is not classified by severity but by location. Gastritis is not classified by age, but by location. Gastritis is not classified by symptoms, but by location. Cognitive Level: Comprehension 903

17. Which information by the nurse indicates a good understanding of depolarization? Depolarization of a cardiac muscle cell occurs as the result of: a. A decrease in the permeability of the cell membrane to ions b. The rapid movement of ions across the cell membrane c. A blockade by calcium ions d. Stimuli instigated during the refractory period

ANS: B Depolarization is caused by the movement of electrically charged solutes (ions) across cardiac cell membranes. Permeability must be increased for movement to occur. A blockage of calcium ions would decrease depolarization. No cardiac action potential can be initiated in the refractory period. Cognitive Level: Comprehension 558

5. A 25-year-old female presents to her primary care provider reporting vaginal discharge of a white, viscous, and foul-smelling substance. She reports that she has been taking antibiotics for the past 6 months. Which finding will the nurse most likely see on the microorganism report? a. Clostridium difficile overgrowth b. Decreased Lactobacillus c. Streptococcus overgrowth d. Decreased Candida albicans

ANS: B Diminished colonization with Lactobacillus that occurs as a result of prolonged antibiotic treatment increases the risk for vaginal infections, such as vaginosis. Clostridium difficile occurs in the colon, not the vagina. Streptococcus overgrowth will occur in the mouth. Candida albicans occurs in the colon, not the vagina. Cognitive Level: Comprehension 120

15. Which of the following characteristics is shared by both the secretory and systemic immune systems? a. Lymphocytic paths of migration b. Neutralization following antigen-antibody binding c. Timing of response d. Location of response

ANS: B Direct effects result from the binding of antibody directly to a harmful antigen or infectious agent. These include inhibition of processes that are necessary for infection, such as the reaction of an infectious agent with a particular cell in the body or neutralization of harmful bacterial toxins. The effect of the immune system is neutralization, not lymphocytic paths of migration. The effect of the immune system is neutralization, not timing of response. The effect of the immune system is neutralization, not location of response. Cognitive Level: Comprehension 150

36. Outcomes of laboratory tests include an elevated level of natriuretic peptides. Which organ is the priority assessment? a. Lungs b. Heart c. Liver d. Brain

ANS: B Elevated natriuretic peptides indicate problems with the heart or the vasculature. Elevated natriuretic peptides indicate problems with the heart or the vasculature, not the lungs. Elevated natriuretic peptides indicate problems with the heart or the vasculature, not the liver. Elevated natriuretic peptides indicate problems with the heart or the vasculature, not the brain. Cognitive Level: Comprehension 103

23. A 10-year-old male is diagnosed with a parasite. Which lab result should the nurse check for a response to the parasite? a. Monocytes b. Eosinophils c. Neutrophils d. Macrophages

ANS: B Eosinophils serve as the bodys primary defense against parasites. Monocytes are not the bodys primary defense against parasite; eosinophils are. Monocytes are phagocytic. Neutrophils are phagocytic; they are not the bodys defense against parasites. Macrophages are not active against parasites; they act as long-term defense against infections. Cognitive Level: Comprehension 131

12. A water test recently revealed arsenic levels above 200 g/L. Which of the following cancers would be most likely to develop in those who consistently drank the water? a. Liver b. Skin c. Colon d. Kidney

ANS: B Evidence indicates an increased risk of bladder, skin, and lung cancers following consumption of water with high levels of arsenic. Evidence indicates an increased risk of bladder, skin, and lung cancers, not liver, following consumption of water with high levels of arsenic. Evidence indicates an increased risk of bladder, skin, and lung cancers, not colon, following consumption of water with high levels of arsenic. Evidence indicates an increased risk of bladder, skin, and lung cancers, not kidneys, following consumption of water with high levels of arsenic. Cognitive Level: Comprehension 278

36. An increase in the rate of red blood cell breakdown causes which form of jaundice? a. Obstructive b. Hemolytic c. Hepatocellular d. Metabolic

ANS: B Excessive hemolysis (breakdown) of red blood cells can cause hemolytic jaundice (prehepatic jaundice). Excessive hemolysis (breakdown) of red blood cells can cause hemolytic jaundice (prehepatic jaundice). It is not due to obstruction. Excessive hemolysis (breakdown) of red blood cells can cause hemolytic jaundice (prehepatic jaundice). It is not hepatocellular. Excessive hemolysis (breakdown) of red blood cells can cause hemolytic jaundice (prehepatic jaundice). It is not metabolic. Cognitive Level: Comprehension 918

27. A 5-year-old male becomes ill with a severe cough. Histologic examination reveals a bacterial infection, and further laboratory testing reveals cell membrane damage and decreased protein synthesis. Which of the following is the most likely cause of this illness? a. Endotoxin b. Exotoxin c. Hemolysis d. Septicemia

ANS: B Exotoxins are enzymes that can damage the plasma membranes of host cells or can inactivate enzymes critical to protein synthesis, and endotoxins activate the inflammatory response and produce fever. Endotoxins released by blood-borne bacteria cause the release of vasoactive enzymes that increase the permeability of blood vessels. Hemolysis is the breakdown of red cells. Septicemia is the growth of bacteria in the blood. Cognitive Level: Comprehension 171

29. The risk of hypovolemic shock is high with acute mesenteric arterial insufficiency because: a. The resulting liver failure causes a deficit of plasma proteins and a loss of oncotic pressure. b. Ischemia alters mucosal membrane permeability, and fluid is shifted to the bowel wall and peritoneum. c. Massive bleeding occurs in the GI tract. d. Overstimulation of the parasympathetic nervous system results in ischemic injury to the intestinal wall.

ANS: B Fluid shifts lead to hypovolemia. Arterial insufficiency is not related to liver failure. Bleeding may occur, but hypovolemia is related to fluid shifts. Fluid shifts lead to hypovolemia; it is not due to overstimulation of the parasympathetic nerves. Cognitive Level: Comprehension 911

15. A 40-year-old male vegetarian is diagnosed with folate deficiency anemia. He reports that he is an alcoholic. Which of the following factors put him at greatest risk for developing his disease? a. Being vegetarian b. Being alcoholic c. Age d. Gender

ANS: B Folate deficiency occurs more often in alcoholics and individuals who are malnourished because of fad diets or diets low in vegetables. A diet high in vegetables would help prevent folate deficiency. Age is not a factor in folate deficiency. Being alcoholic promotes the greatest risk, since this disorder is diet related. Gender is not a factor in folate deficiency. Being alcoholic promotes the greatest risk, since this disorder is diet related. Cognitive Level: Comprehension 503

4. A 5-year-old male was diagnosed with normocytic-normochromic anemia. Which of the following anemias does the nurse suspect the patient has? a. Sideroblastic anemia b. Hemolytic anemia c. Pernicious anemia d. Iron deficiency anemia

ANS: B Hemolytic anemia is an example of normocytic-normochromic anemia. Sideroblastic anemia is an example of microcytic hypochromic anemia. Pernicious anemia is an example of a macrocytic anemia. Iron deficiency anemia is an example of microcytic hypochromic anemia. Cognitive Level: Comprehension 501

3. When a nurse is reviewing lab results and notices that the erythrocytes contain an abnormally low concentration of hemoglobin, the nurse calls these erythrocytes: a. Hyperchromic b. Hypochromic c. Macrocytic d. Microcytic

ANS: B Hypochromic erythrocytes have low concentrations of hemoglobin. Hyperchromic erythrocytes have high concentrations of hemoglobin. Macrocytic refers to larger cell size, not hemoglobin concentration. Microcytic refers to small cell size, not hemoglobin concentration. Cognitive Level: Comprehension 501

11. When describing the male urinary anatomy, which information should the nurse include? The portion of the male urethra that is closest to the bladder is the _____ portion. a. Membranous b. Prostatic c. Cavernous d. Vas deferens

ANS: B In the male, the prostatic urethra is closest to the bladder. In the male, the prostatic urethra is closest to the bladder; the membranous follows after the prostatic. In the male, the prostatic urethra is closest to the bladder; the cavernous follows after the membranous. In the male, the prostatic urethra is closest to the bladder. The vas deferens is not a portion of the male urethra. Cognitive Level: Comprehension 729

30. A 49-year-old male presents to his primary care provider reporting chest pain. EKG reveals ST elevation. He is diagnosed with myocardial ischemia. Which of the following interventions would be most beneficial? a. Administer a diuretic to decrease volume. b. Apply oxygen to increase myocardial oxygen supply. c. Encourage exercise to increase heart rate. d. Give an antibiotic to decrease infection.

ANS: B Increase myocardial oxygen supply is indicated to treat ischemia. Oxygen is indicated, not a decrease in volume. Heart rate should be decreased to decrease cardiac workload. Antibiotics are not the most beneficial; oxygen is. Cognitive Level: Comprehension 603

15. A 10-year-old male was physically abused for the previous 4 years. He reports feeling stressed all the time and frequently gets ill. Which of the following hormones is most likely suppressed in this child? a. Endorphins b. Growth hormone c. Prolactin d. Cortisol

ANS: B Increased levels of growth hormone are associated with psychological stimuli associated with stress and, thus, increase the chance of becoming ill. Increased levels of growth hormone are associated with psychological stimuli associated with stress; endorphins do not affect this. Increased levels of growth hormone are associated with psychological stimuli associated with stress; prolactin does not affect this. Increased levels of growth hormone are associated with psychological stimuli associated with stress; cortisol does not affect this. Cognitive Level: Comprehension 219

12. A 50-year-old female is diagnosed with a hormone-secreting tumor of the adrenal cortex. Which of the following would be expected? a. Decreased blood pressure b. Increased lipogenesis of face and trunk c. Increased lipogenesis of extremities d. Decreased gastric secretion

ANS: B Increased release of cortisol leads to increased lipogenesis of face and trunk. Increased release of cortisol leads to increased blood pressure. Increased release of cortisol leads to lipolysis, not lipogenesis, of the extremities. Increased release of cortisol leads to increased gastric secretion. Cognitive Level: Comprehension 206 | p. 211

8. When histamine is released in the body, which of the following responses would the nurse expect? a. Bronchial dilation b. Edema c. Vasoconstriction d. Constipation

ANS: B Increased vascular permeability leads to edema and is a direct response to histamine. Histamine produces bronchoconstriction. Histamine produces vasodilation. Histamine does not produce constipation; it increases gastric acidity. Cognitive Level: Comprehension 190

17. A nurse is giving an example of inflammation as an etiology for cancer development. What is the best example the nurse should give? a. Pneumonia and lung cancer b. Ulcerative colitis and colon cancer c. Prostatic hypertrophy and prostate cancer d. Hypercholesteremia and leukemia

ANS: B Individuals with a 10+ year history of ulcerative colitis have a 30-fold increase in developing colon cancer. There is no relationship between pneumonia and lung cancer, but there is a relationship between ulcerative colitis and colon cancer. There is no relationship between prostatic hypertrophy and cancer of the prostate, but there is a relationship between ulcerative colitis and colon cancer. There is no relationship between hypercholesteremia and leukemia, but there is a relationship between ulcerative colitis and colon cancer. Cognitive Level: Comprehension 240

11. A patient asks why indoor pollution is worse than outdoor pollution. How should the nurse respond? Indoor pollution is considered worse than outdoor pollution because of cigarette smoke and: a. Fireplace wood smoke b. Radon c. Benzene d. Chlorine

ANS: B Indoor pollution is related to cigarette smoke and radon. Indoor pollution is related to cigarette smoke and radon, not fireplace smoke. Indoor pollution is related to cigarette smoke and radon, not benzene. Indoor pollution is related to cigarette smoke and radon, not chlorine. Cognitive Level: Comprehension 268

1. A 34-year-old male was diagnosed with a bacterial GI infection. Which of the following types of diarrhea would most likely occur with his condition? a. Osmotic b. Secretory c. Hypotonic d. Motility

ANS: B Infections lead to secretory diarrhea. A nonabsorbable substance in the intestine leads to osmotic diarrhea. Hypotonic diarrhea is not a form of diarrhea. Food is not mixed properly, digestion and absorption is impaired, and motility is increased, leading to motility diarrhea.

37. A 15-year-old male is diagnosed with infectious mononucleosis. When the patient asks how he got this disease, how should the nurse respond? The most likely cause is: a. Adenovirus b. Epstein-Barr virus (EBV) c. Cytomegalovirus (CMV) d. Toxoplasma gondii

ANS: B Infectious mononucleosis (IM) is an acute infection of B lymphocytes (B cells) with EBV. IM is an acute infection of B cells with EBV, not adenovirus. IM is an acute infection of B cells with EBV, not cytomegalovirus. IM is an acute infection of B cells with EBV, not Toxoplasma gondii. Cognitive Level: Comprehension 511

. When a staff member asks how remodeling of bone occurs, what is the nurses best response? Remodeling is done by clusters of bone cells called: a. Precursor stimulating cells b. Osteoclastic cutting cones c. Basic multicellular units d. Haversian system cells

C

2. A new drug is being tested that is known to inhibit parasympathetic nervous system activity. It is expected that the salivary glands will become: a. Hyperactive b. Hypoactive c. Atrophied d. Enlarged

ANS: B Inhibition of the parasympathetic nervous system would result in decreased (hypoactive) functions of the salivary glands. The individual would experience dry mouth. The glands would be hypoactive, not hyperactive. The gland would become hypoactive, but would not atrophy. The gland would become hypoactive; it would not enlarge. Cognitive Level: Comprehension 873

1. A public health nurse is teaching the community about health promotion. Which information should the nurse include for innate immunity? Innate immunity is gained: a. Following an illness b. At birth c. Via injection of specific antibodies d. In adulthood

ANS: B Innate immunity is present at birth. Innate immunity is present at birth and does not require an illness. Innate immunity is present at birth and does not require injection. Innate immunity is present at birth. Cognitive Level: Comprehension 119

26. A cardiologist is teaching about myocardial cells. Which information should the cardiologist include? _____ are thickened areas of the sarcolemma of myocardial cells that enable electrical impulses to travel in a continuous cell-to-cell fashion. a. Myosins b. Intercalated disks c. Troponin Ts d. I bands

ANS: B Intercalated discs allow electrical impulses to be transmitted rapidly from cardiac fiber to cardiac fiber because the network of fibers is connected at these discs. Myosins are filaments that are a part of cardiac tissue. They do not play a role in conduction. Troponin Ts are involved in contraction, but are not part of the sarcolemma. I bands are a part of the filaments bud, which do not conduct impulses in a cell-to-cell fashion. Cognitive Level: Comprehension 560

22. A 35-year-old female was severely burned and is hospitalized. She is now suffering from acute tubular necrosis (ATN). Which of the following is the most likely diagnosis the nurse will observe on the chart? a. Prerenal b. Intrarenal c. Extrarenal d. Postrenal

ANS: B Intrarenal acute kidney failure is associated with several systemic diseases but is commonly related to ATN. Prerenal renal failure occurs anterior to the kidney. Extrarenal renal failure occurs outside the kidney. Postrenal is due to diseases that obstruct the flow of urine from the kidneys. Cognitive Level: Comprehension 755

21. A 21-year-old female was recently diagnosed with iron deficiency anemia. In addition to fatigue and weakness, which of the following clinical signs and symptoms would she most likely exhibit? a. Hyperactivity b. Spoon-shaped nails c. Gait problems d. Petechiae

ANS: B Iron deficiency anemia is manifested by fingernails that become brittle and spoon shaped or concave. Iron deficiency anemia is manifested by fingernails that become brittle and spoon shaped or concave. It does not involve hyperactivity. Iron deficiency anemia is manifested by fingernails that become brittle and spoon shaped or concave. It does not involve gait problems. Iron deficiency anemia is manifested by fingernails that become brittle and spoon shaped or concave. It does not involve petechiae. Cognitive Level: Comprehension 504

45. The people from which country have the lowest risk for Hodgkin lymphoma? a. United States b. Japan c. Denmark d. Great Britain

ANS: B Japan has the lowest rate of Hodgkin lymphoma. The United States has one of the highest rates. Denmark has the one of the highest rates. Great Britain is not ranked. Cognitive Level: Comprehension 517

30. Which of the following conditions is thought to contribute to the development of obesity? a. Insulin excess b. Leptin resistance c. Adipocyte failure d. Malabsorption

ANS: B Leptin resistance disrupts hypothalamic satiety signaling and promotes overeating and excessive weight gain and is a factor in the development of obesity. Insulin becomes resistant, not present in excess. Leptin resistance, not adipocyte failure, leads to obesity. Malabsorption does not lead to obesity, but primarily to weight loss. Cognitive Level: Comprehension 912

33. A staff member asks what leukocytosis means. How should the nurse respond? Leukocytosis can be defined as: a. A normal leukocyte count b. A high leukocyte count c. A low leukocyte count d. Another term for leukopenia

ANS: B Leukocytosis is present when the count is higher than normal. Leukocytosis is present when the count is higher than normal, not normal. Leukocytosis is present when the count is higher than normal, not low. Leukocytosis is present when the count is higher than normal. It is not another term for leukopenia. Cognitive Level: Comprehension 508

10. A nurse monitors the patient for _____ when rapid onset of malignant hypertension results. a. Atherosclerosis b. Encephalopathy c. Pulmonary edema d. Acute renal failure

ANS: B Malignant hypertension leads to cerebral edema and cerebral dysfunction (encephalopathy) and even death. Malignant hypertension leads to cerebral edema; atherosclerosis does not. Malignant hypertension could lead to heart failure, which could lead to pulmonary edema, but encephalopathy occurs more frequently. Malignant hypertension could lead to heart failure, which could lead to renal failure, but encephalopathy occurs more frequently.

4. When the immunoglobulin crosses the placenta, what type of immunity does the fetus receive? a. Active b. Passive c. Innate d. Cell-mediated

ANS: B Maternal antibodies that pass across the placenta into the fetus before birth provide passive systemic immunity. Maternal antibodies that pass across the placenta into the fetus before birth provide passive systemic immunity, not active immunity. Maternal antibodies that pass across the placenta into the fetus before birth provide passive systemic immunity, not innate immunity. Maternal antibodies that pass across the placenta into the fetus before birth provide passive systemic immunity, not cell-mediated immunity. Cognitive Level: Comprehension 145

19. A 58-year-old female presents in the clinic presenting with fatigue, weight loss, and tingling in her fingers. Laboratory findings show low hemoglobin and hematocrit, a high mean corpuscular volume, and normal plasma iron. These assessment findings are consistent with which type of anemia? a. Hemolytic anemia b. Pernicious anemia c. Iron deficiency anemia d. Aplastic anemia

ANS: B Pernicious anemia is manifested by tingling paresthesias of feet and fingers. Pernicious, not hemolytic, anemia is manifested by tingling paresthesias of feet and fingers. Pernicious, not iron deficiency, anemia is manifested by tingling paresthesias of feet and fingers. Pernicious, not aplastic, anemia is manifested by tingling paresthesias of feet and fingers. Cognitive Level: Comprehension 501

. When a staff member asks the nurse about the contractile unit of a muscle, how should the nurse reply? The contractile unit of a muscle cell is called the: a. Motor unit b. Basement membrane c. Myofibril d. Ribosome

C

42. A 65-year-old male is diagnosed with multiple myeloma. He reports severe pain. This pain can be attributed to: a. Neuropathic infiltrations b. Destruction of bone tissue c. Tissue hypoxia d. Accumulation of toxic proteins

ANS: B Multiple myeloma is a B cell cancer characterized by the proliferation of malignant plasma cells that infiltrate the bone marrow and aggregate into tumor masses throughout the skeletal system; thus, the pain is related to bone destruction. Multiple myeloma is a B cell cancer characterized by the proliferation of malignant plasma cells that infiltrate the bone marrow; thus, pain is related to destruction of bone tissue, not neuropathic infiltrations. Multiple myeloma is a B cell cancer characterized by the proliferation of malignant plasma cells that infiltrate the bone marrow; thus, pain is related to destruction of bone tissue, not tissue hypoxia. Multiple myeloma is a B cell cancer characterized by the proliferation of malignant plasma cells that infiltrate the bone marrow; thus, pain is related to destruction of bone tissue, not accumulation of toxic proteins. Cognitive Level: Comprehension 520

41. An oncologist is discussing multiple myeloma. Which information should the oncologist include? Multiple myeloma can be defined as a neoplasm of: a. T cells b. B cells c. Immature plasma cells d. Mature red blood cells

ANS: B Multiple myeloma is a B cell cancer. Multiple myeloma is a B cell cancer, not T cells. Multiple myeloma is a B cell cancer, not immature plasma cells. Multiple myeloma is a B cell cancer, not mature red blood cells. Cognitive Level: Comprehension 518 | p. 520

8. A 35-year-old male received a traumatic brain injury in a motor vehicle accident. CT scan revealed a lesion above the pontine micturition center. Which of the following would the nurse expect? a. Dyssynergia b. Detrusor hyperreflexia c. Detrusor areflexia d. Detrusor sphincter dyssynergia

ANS: B Neurologic disorders that develop above the pontine micturition center result in detrusor hyperreflexia, also known as an uninhibited or reflex bladder. Lesions that develop in upper motor neurons of the brain and spinal cord result in dyssynergia. Lesions that involve the sacral micturition center (below S1; may also be termed cauda equina syndrome) or peripheral nerve lesions result in detrusor areflexia (acontractile detrusor), a lower motor neuron disorder. Neurologic lesions that occur below the pontine micturition center but above the sacral micturition center (between C2 and S1) are also upper motor neuron lesions and result in detrusor hyperreflexia with vescico-sphincter dyssynergia. Cognitive Level: Comprehension 744

12. The predominant phagocyte of early inflammation is the: a. Eosinophil b. Neutrophil c. Lymphocyte d. Macrophage

ANS: B Neutrophils are the predominant phagocytes in the early inflammatory site, arriving within 6 to 12 hours after the initial injury. Eosinophils help limit and control inflammation, but they are not the prominent phagocyte. Lymphocytes are part of the innate immune response. Macrophages kill microorganisms. Cognitive Level: Comprehension 129

11. A 20-year-old male shoots his hand with a nail gun while replacing roofing shingles. Which of the following cell types would be the first to aid in killing bacteria to prevent infection in his hand? a. Eosinophils b. Neutrophils c. Leukotrienes d. Monocytes

ANS: B Neutrophils are the predominant phagocytes in the early inflammatory site, arriving within 6 to 12 hours after the initial injury. Eosinophils help limit and control inflammation. Leukotrienes are activators of the inflammatory response. Monocytes enter much later and replace leukocytes. Cognitive Level: Comprehension 129

4. A nurse takes an adult patients blood pressure and determines it to be normal. What reading did the nurse obtain? a. Systolic pressure between 140 mm Hg and 150 mm Hg b. Systolic pressure less than 120 mm Hg and diastolic pressure less than 80 mm Hg c. Systolic pressure less than 100 mm Hg regardless of diastolic pressure d. Systolic pressure greater than 140 mm Hg and a diastolic pressure of 100 mm Hg

ANS: B Normal blood pressure has a systolic pressure less than 120 mm Hg and diastolic pressure less than 80 mm Hg. A systolic pressure of 140 mm Hg or more would indicate stage I hypertension. A systolic pressure of less than 100 mm Hg would indicate low blood pressure. A diastolic pressure greater than 90 mm Hg would indicate hypertension. Cognitive Level: Comprehension 587

7. A 65-year-old male presents for a routine checkup. A blood pressure check reveals a systolic pressure of 160 mm Hg and a diastolic pressure of 70 mm Hg. Which of the following is the most likely cause of this type of pressure elevation? a. Vasospasm b. Rigidity of the aorta c. Decreased sodium intake d. Lung disease

ANS: B Older adults experience stiffening of the arteries, which could lead to hypertension. A vasospasm could lead to chest pain, not hypertension. Increased sodium intake, not decreased, could lead to increased blood volume and hypertension. Lung disease would not lead to hypertension. Cognitive Level: Comprehension 587

6. Which statement indicates the nurse understands blood flow? Oxygenated blood flows through the: a. Superior vena cava b. Pulmonary veins c. Pulmonary artery d. Cardiac veins

ANS: B Oxygenated blood flows through the pulmonary veins. Oxygenated blood flows through the pulmonary veins, not the superior vena cava, which carries venous blood. Oxygenated blood flows through the pulmonary veins, not the pulmonary artery, which carries unoxygenated blood to the lungs. Oxygenated blood flows through the pulmonary veins, not the cardiac veins, which carry unoxygenated blood. Cognitive Level: Comprehension 554

27. Parasympathetic stimulation to the pancreas will cause which of the following reactions? a. Hormonal inhibition b. Enzyme secretion c. Vasoconstriction d. Decreased bicarbonate production

ANS: B Pancreatic innervation arises from preganglionic parasympathetic fibers of the vagus nerve. These fibers activate postganglionic fibers, which stimulate enzymatic and hormonal secretion. Pancreatic innervations stimulate hormone secretion, not inhibit it. Sympathetic postganglionic fibers from the celiac and superior mesenteric plexuses innervate the blood vessels, cause vasoconstriction, and inhibit pancreatic secretion. Parasympathetic stimulation is not involved in bicarbonate production. Cognitive Level: Comprehension 888

16. A 54-year-old male is diagnosed with peptic ulcer disease. This condition is most likely caused by: a. Hereditary hormonal imbalances with high gastrin levels b. Breaks in the mucosa and presence of corrosive secretions c. Decreased vagal activity and vascular engorgement d. Gastric erosions related to high ammonia levels and bile reflux

ANS: B Peptic ulcer disease is caused by breaks in the mucosa and the presence of corrosive substances. High gastrin occurs, but the disease is due to breaks in the mucosa. Vagal activity increases. Gastric erosions occur, but they are not due to high ammonia. Cognitive Level: Comprehension 903

8. A nurse is teaching the staff about platelets. Which information should the nurse include? In addition to playing a role in hemostasis, platelets have the ability to: a. Stimulate bone marrow production of erythrocytes b. Release biochemical mediators of inflammation c. Undergo cell division in response to bleeding d. Activate a humoral response

ANS: B Platelets contain cytoplasmic granules capable of releasing proinflammatory biochemical mediators when stimulated by injury to a blood vessel. Platelets assist with inflammation; they do not produce erythrocytes. Platelets assist with inflammation; they do not undergo cell division in response to bleeding. Platelets assist with inflammation; they do not activate the humoral response. Cognitive Level: Comprehension 481

32. A 54-year-old male complains that he has been vomiting blood. Tests reveal portal hypertension. Which of the following is the most likely cause of his condition? a. Thrombosis in the spleen b. Cirrhosis of the liver c. Left ventricular failure d. Renal stenosis

ANS: B Portal hypertension occurs secondarily to cirrhosis of the liver. Portal hypertension occurs secondarily to cirrhosis of the liver, not thrombosis of the spleen. Portal hypertension occurs secondarily to cirrhosis of the liver, not left ventricular failure. Portal hypertension occurs secondarily to cirrhosis of the liver, not renal stenosis. Cognitive Level: Comprehension 915

30. A nurse is discussing the pressure generated at the end of diastole. Which term is the nurse describing? a. Preload b. Afterload c. Systemic vascular resistance d. Total peripheral resistance

ANS: B Preload is the volume and associated pressure generated in the ventricle at the end of diastole. B Preload, not afterload, is the volume and associated pressure generated in the ventricle at the end of diastole. Afterload is the resistance to ejection of blood from the left ventricle. Systemic vascular resistance is related to afterload. Total peripheral resistance increases afterload. Cognitive Level: Comprehension 563

29. A patient with end-stage renal disease has pruritus. When the patient asks what causes this, what is the nurses best response? Pruritus, seen in patients with end-stage renal disease, is caused by high levels of: a. Potassium b. Calcium c. Sodium d. Magnesium

ANS: B Pruritus is related to high levels of calcium. Pruritus is related to high levels of calcium, not potassium. Pruritus is related to high levels of calcium, not sodium. Pruritus is related to high levels of calcium, not magnesium. Cognitive Level: Comprehension 760

12. A 28-year-old female presents with fever, chills, and flank pain. She is diagnosed with pyelonephritis. A nurse recalls the patients infection is located in the: a. Bladder b. Renal pelvis c. Renal tubules d. Glomerulus

ANS: B Pyelonephritis is an infection of one or both upper urinary tracts (ureter, renal pelvis, and interstitium). Pyelonephritis is an infection of one or both upper urinary tracts (ureter, renal pelvis, and interstitium). It does not affect the bladder. Pyelonephritis is an infection of one or both upper urinary tracts (ureter, renal pelvis, and interstitium). It does not affect the tubules. Pyelonephritis is an infection of one or both upper urinary tracts (ureter, renal pelvis, and interstitium). It does not involve the glomerulus. Cognitive Level: Comprehension 749

13. When an oncologist is teaching about how radiation induces genomic instability, which of the following should the oncologist discuss? a. Increasing hypersensitivity b. Facilitating new mutations c. Promoting cell death d. Enhancing mitosis

ANS: B Radiation induces genomic instability because it facilitates new mutations. Radiation does not promote hypersensitivity. Radiation may promote cell death, but this is not its role in inducing genomic instability. Radiation does not enhance mitosis but halts it. Cognitive Level: Comprehension 274

18. Individuals with Raynaud disease need to be counseled to avoid which of the following conditions to prevent severe symptoms? a. Allergic reactions b. Cold exposure c. Hot water immersion d. Tissue injury

ANS: B Raynaud disease consists of vasospastic attacks triggered by brief exposure to cold. Raynaud disease demonstrates symptoms when extremities are exposed to cold. It is not an allergic reaction. Raynaud disease demonstrates symptoms when extremities are exposed to cold. It is not due to hot water immersion. Raynaud disease demonstrates symptoms when extremities are exposed to cold. It is not the result of tissue injury. Cognitive Level: Comprehension 594

9. A 75-year-old male reports to his primary care provider loss of urine with cough, sneezing, or laughing. Which of the following is the most likely diagnosis the nurse will observe on the chart? a. Urge incontinence b. Stress incontinence c. Overflow incontinence d. Functional incontinence

ANS: B Reduced resistance is associated with the symptom of stress incontinence, which is incontinence with coughing or sneezing. Urge incontinence is the inability to hold the urine when the urge is felt. There is coordination between the contracting bladder and the external sphincter, but the detrusor is too weak to empty the bladder, resulting in urinary retention with overflow or stress incontinence. Functional incontinence is similar to overflow and is not associated with coughing. Cognitive Level: Comprehension 745

21. While planning care for a patient with superior vena cava syndrome (SVCS), which principle should the nurse remember? SVCS is a progressive _____ of the superior vena cava (SVC) that leads to venous distention of the upper extremities and head. a. Inflammation b. Occlusion c. Distention d. Sclerosis

ANS: B SVCS is a progressive occlusion of the SVC that leads to venous distention in the upper extremities and head. SVCS is a progressive occlusion, not inflammation, of the SVC that leads to venous distention in the upper extremities and head. SVCS is a progressive occlusion, not distention, of the SVC that leads to venous distention in the upper extremities and head. SVCS is a progressive occlusion, not sclerosis, of the SVC that leads to venous distention in the upper extremities and head. Cognitive Level: Comprehension 586

1. The nursing faculty is discussing digestion and indicates it begins in the mouth with salivary - amylase (ptyalin) that initiates the digestion of: a. Proteins b. Carbohydrates c. Fats d. Amino acids

ANS: B Salivary amylase initiates the digestion of carbohydrates. Protein digestion occurs in the stomach and is influenced by pepsin. Fat digestion is aided by bile. Amino acids are not involved in the digestive process. Cognitive Level: Comprehension 873

1. Which of the following is responsible for initiating clonal selection? a. T cells b. B cells c. Antigens d. Lymphocytes

ANS: C The lymphocytes remain dormant until an antigen initiates clonal selection. T cells are part of the immune response, but do not initiate clonal selection. B cells are antibodies. Lymphocytes are released into the circulation as immature cells that react with antigens. Cognitive Level: Comprehension 144

2. Selye reported several structural changes in rats exposed to repeated stressors. These included: a. Hypertrophy of the thymus gland b. Ulceration in the gastrointestinal system c. Atrophy of the cortex in the adrenal gland d. Fight or flight response

ANS: B Selye found the development of bleeding ulcers in response to stress. Selye found atrophy, not hypertrophy, of the thymus gland. Selye found enlargement of the cortex, rather than atrophy. The flight or fight response was part of the general adaptation syndrome, not a structural change. Cognitive Level: Comprehension 204

27. A patient has a disease state that results from the secretion of toxins by bacteria. Which medical diagnosis will the nurse see documented on the chart? a. Malaria b. Tetanus c. Smallpox d. Hepatitis

ANS: B Some bacteria secrete toxins that harm individuals. For instance, specific bacterial toxins cause the symptoms of tetanus or diphtheria. Tetanus, not malaria, is due to toxin secretion. Tetanus, not smallpox, is due to toxin secretion. Tetanus, not hepatitis, is due to toxin secretion. Cognitive Level: Comprehension 150

9. A serious complication of paraesophageal hiatal hernia is: a. Hemorrhage b. Strangulation c. Peritonitis d. Ascites

ANS: B Strangulation of the hernia is a major complication. Strangulation of the hernia, not hemorrhage, is a major complication. Strangulation of the hernia, not peritonitis, is a major complication. Strangulation of the hernia, not ascites, is a major complication. Cognitive Level: Comprehension 899

1. After teaching about stress, which information indicates a correct understanding of stress? a. It worsens but does not cause disease. b. It can be viewed as transactional. c. It is a psychologic, rather than biologic, event. d. It is a passive event.

ANS: B Stress has come to be defined by most as a transactional concept. There is a relationship between stress and disease. Stress is both a psychologic and a biologic event. Stress is an active, rather than a passive, event. Cognitive Level: Comprehension 208

20. A 52-year-old male presents with pooling of blood in the veins of the lower extremities and edema. The diagnosis is chronic venous insufficiency, and an expected assessment finding of this disorder is: a. Deep vein thrombus formation b. Skin hyperpigmentation c. Gangrene d. Edema above the knee

ANS: B Symptoms include edema of the lower extremities and hyperpigmentation of the skin of the feet and ankles. Edema in these areas may extend to the knees. Symptoms include edema and hyperpigmentation of the skin, but deep vein thrombi do not form. Gangrene does not occur in veins, but in arteries. Edema does not occur above the knee. Cognitive Level: Comprehension 586

31. Which principle should the nurse remember when planning nursing care for a patient with heart problems? As stated by the Frank-Starling law, there is a direct relationship between the _____ of the blood in the heart at the end of diastole and the _____ of contraction during the next systole. a. Pressure, duration b. Volume, force c. Viscosity, force d. Viscosity, duration

ANS: B The Frank-Starling law of the heart describes the length-tension relationship of ventricular end- diastolic volume (VEDV) (preload) to myocardial contractility (as measured by stroke volume). The Frank-Starling law of the heart describes the length-tension relationship of VEDV (preload) to myocardial contractility (as measured by stroke volume). It does not refer to duration. The Frank-Starling law of the heart describes the length-tension relationship of VEDV (preload) to myocardial contractility (as measured by stroke volume). It does not refer to viscosity. The Frank-Starling law of the heart describes the length-tension relationship of VEDV (preload) to myocardial contractility (as measured by stroke volume). It does not refer to duration.

20. The _____ represents the sum of all ventricular muscle cell depolarization. a. PR interval b. QRS complex c. QT interval d. P wave

ANS: B The QRS complex represents the sum of all ventricular muscle cell depolarizations. The PR interval represents the onset of atrial activation to the onset of ventricular activity. The QT interval represents electrical systole of the ventricles. The P wave represents atrial depolarization. Cognitive Level: Comprehension 559

15. A 50-year-old female received trauma to the chest that caused severe impairment of the primary pacemaker cells of the heart. Which of the following areas received the greatest damage? a. Atrioventricular (AV) node b. Sinoatrial (SA) node c. Bundle of His d. Ventricles

ANS: B The SA node is considered the pacemaker of the heart. The SA node, not the AV node, is considered the pacemaker of the heart. The SA node is considered the pacemaker of the heart; the bundle of His is not involved. The SA node is considered the pacemaker of the heart; the ventricles are not involved. Cognitive Level: Comprehension 560

10. A nurse recalls stress-induced stimulation of the adrenal cortex causes it to secrete: a. Estrogen b. Cortisol c. Parathyroid hormone d. Adrenocorticotropin hormone (ACTH)

ANS: B The adrenal cortex secretes cortisol. The adrenal cortex secretes cortisol, not estrogen. The adrenal cortex secretes cortisol, not parathyroid hormone. The adrenal cortex secretes cortisol, not ACTH. Cognitive Level: Comprehension 210

18. The colon receives splanchnic blood flow from which artery(ies)? a. Celiac b. Superior and inferior mesenteric c. Hepatic and portal d. Omental

ANS: B The blood supply of the large intestine and rectum is derived primarily from branches of the superior and inferior mesenteric arteries. The blood supply of the large intestine and rectum is derived primarily from branches of the superior and inferior mesenteric arteries, not the celiac. The blood supply of the large intestine and rectum is derived primarily from branches of the superior and inferior mesenteric arteries, not the hepatic or portal. The blood supply of the large intestine and rectum is derived primarily from branches of the superior and inferior mesenteric arteries, not the omental. Cognitive Level: Comprehension 882

14. A nurse recalls the blood vessels of the kidneys are innervated by the: a. Vagus nerve b. Sympathetic nervous system c. Somatic nervous system d. Parasympathetic nervous system

ANS: B The blood vessels of the kidney are innervated by the autonomic nervous system through sympathetic fibers. The blood vessels of the kidney are innervated by the autonomic nervous system through sympathetic fibers, not the vagus nerve. The somatic system regulates the musculoskeletal system. The sympathetic system, not the parasympathetic system, innervates the blood vessels of the kidneys. Cognitive Level: Comprehension 730

The primary care provider states that the patient has vasogenic edema. The nurse realizes vasogenic edema is clinically important because: a. It usually has an infectious cause. b. The blood-brain barrier is disrupted. c. Increased intracranial pressure (ICP) is excessively high. d. It always causes herniation.

B

13. While viewing the electrocardiogram, the nurse recalls the _____ conducts action potentials down the atrioventricular septum. a. Bachmann bundle b. Bundle of His c. Sinoatrial node d. Atrioventricular node

ANS: B The bundle of His conducts action potentials down the atrioventricular septum. The bundle of His conducts action potentials down the atrioventricular septum, not the Bachmann bundle. The bundle of His conducts action potentials down the atrioventricular septum; the sinoatrial node conducts the potential along the atria. The bundle of His conducts action potentials down the atrioventricular septum; the atrioventricular node conducts impulses to the ventricles. Cognitive Level: Comprehension 557

17. Which statement indicates teaching was successful regarding the classic pathway of the complement system? The classic pathway of the complement system is activated by: a. Histamine b. Antigen-antibody complexes c. Leukotrienes d. Prostaglandins

ANS: B The classic pathway of the complement system is activated by antibodies of the immune system. The classic pathway of the complement system is activated by antibodies, not by histamine. The classic pathway of the complement system is activated by antibodies, not by leukotrienes. The classic pathway of the complement system is activated by antibodies, not by prostaglandins. Cognitive Level: Comprehension 123

37. What common symptom should be assessed in individuals with immunodeficiency? a. Anemia b. Recurrent infections c. Hypersensitivity d. Autoantibody production

ANS: B The clinical hallmark of immunodeficiency is a propensity to unusual or recurrent severe infections. The type of infection usually reflects the immune system defect. Anemia is not a manifestation of immunodeficiency. Hypersensitivity is not a symptom of immunodeficiency. Autoantibody production is not a symptom of immunodeficiency.

19. After teaching the staff about the clotting system, which statement indicates teaching was successful? The end product of the clotting system is: a. Plasmin b. Fibrin c. Collagen d. Factor X

ANS: B The end product of the clotting system is fibrin. Plasmin activates the complement cascade. Collagen plays a factor in wound healing. Factor X is a first step in the clotting system. Cognitive Level: Comprehension 124

26. The function of the acinar cells of the pancreas is to secrete: a. Bicarbonate b. Enzymes c. Mucus d. Electrolytes

ANS: B The exocrine pancreas is composed of acinar cells that secrete enzymes. The exocrine pancreas is composed of acinar cells that secrete enzymes, not bicarbonate. The exocrine pancreas is composed of acinar cells that secrete enzymes, not mucus. The exocrine pancreas is composed of acinar cells that secrete enzymes, not electrolytes. Cognitive Level: Comprehension 888

2. When the nurse is discussing the functional unit of the kidney, what other term should the nurse use? a. Calyx b. Nephron c. Collecting duct d. Pyramid

ANS: B The functional unit of the kidney is the nephron. The functional unit of the kidney is the nephron, not the calyx. The functional unit of the kidney is the nephron, not the collecting duct. The functional unit of the kidney is the nephron, not the pyramid. Cognitive Level: Comprehension 726

38. The icteric phase of hepatitis is characterized by which of the following clinical manifestations? a. Fatigue, malaise, vomiting b. Jaundice, dark urine, enlarged liver c. Resolution of jaundice, liver function returns to normal d. Fulminant liver failure, hepatorenal syndrome

ANS: B The icteric phase is manifested by jaundice, dark urine, and clay-colored stools. The liver is enlarged, smooth, and tender, and percussion causes pain; this is the actual phase of illness. Fatigue and vomiting occur during the prodromal stage. Resolution occurs in the recovery phase. Fulminant liver failure does not involve icterus. Cognitive Level: Comprehension 920

11. The ileum and jejunum are suspended by folds of the peritoneum known as the _____ that contains an extensive vascular and nervous network. a. Myenteric plexus b. Mesentery c. Auerbach fold d. Meissner plexus

ANS: B The ileum and jejunum are suspended in loose folds from the posterior abdominal wall by a peritoneal membrane called the mesentery. Intrinsic motor innervation is mediated by the myenteric plexus (Auerbach plexus) and the submucosal plexus (Meissner plexus). Intrinsic motor innervation is mediated by the myenteric plexus (Auerbach plexus) and the submucosal plexus (Meissner plexus). Intrinsic motor innervation is mediated by the myenteric plexus (Auerbach plexus) and the submucosal plexus (Meissner plexus). Cognitive Level: Comprehension 878

25. A 45-year-old female presents with hypertension, anorexia, nausea and vomiting, and anemia. She is diagnosed with chronic renal failure. When the patient asks what caused this anemia, how should the nurse respond? Your anemia is caused by: a. Red blood cells being lost in the urine b. Inadequate production of erythropoietin c. Inadequate iron absorption in the gut d. Red blood cells being injured as they pass through the glomerulus

ANS: B The kidneys are inadequate in their production of erythropoietin. The kidneys are inadequate in their production of erythropoietin; red cells may be lost in the urine, but it is the lack of erythropoietin that leads to anemia. The kidneys are inadequate in their production of erythropoietin; it is not due to inadequate iron absorption. The kidneys are inadequate in their production of erythropoietin; it is not due to red cell destruction. Cognitive Level: Comprehension 758

23. When a patient asks what role the kidneys play in vitamin D function, how should the nurse reply? a. Synthesizes vitamin D from cholesterol b. Activates intestinally absorbed vitamin D c. Metabolizes and breaks down vitamin D d. Excretes excess vitamin D

ANS: B The kidneys play a role in activating intestinally absorbed vitamin D. The kidneys play a role in activating intestinally absorbed vitamin D. They are not involved in synthesizing vitamin D from cholesterol. The kidneys play a role in activating intestinally absorbed vitamin D. They are not involved in metabolizing vitamin D. The kidneys play a role in activating intestinally absorbed vitamin D. They do not excrete excess vitamin D. Cognitive Level: Comprehension 736

5. For the patient experiencing esophageal reflux, the nurse would expect which sphincter to be malfunctioning? a. Pyloric b. Lower esophageal c. Upper esophageal d. Gastric

ANS: B The lower esophageal sphincter keeps food from refluxing into the esophagus. The pylorus is located at the connection between the stomach and the duodenum. The upper esophageal sphincter keeps air from entering the esophagus during respiration. There is no gastric sphincter. Cognitive Level: Comprehension 873

. Which assessment finding by the nurse characterizes a mild concussion? a. A brief loss of consciousness b. Significant behavioral changes c. Retrograde amnesia d. Permanent confusion

C

7. When the nurse is discussing the sodium-sensing cells of the glomerulus, what term should the nurse use? a. Podocytes b. Macula densa c. Mesangial cells d. Loop of Henle

ANS: B The macula densa are sodium-sensing cells. The epithelium has specialized cells called podocytes from which pedicles (foot projections) radiate and adhere to the basement membrane. Mesangial cells lie between and support the capillaries. Mesangial cells have phagocytic ability similar to monocytes, release inflammatory cytokines, and can contract to regulate glomerular capillary blood flow. The loop of Henle is where fluid reabsorption occurs. Cognitive Level: Comprehension 726

20. A 24-year-old male who sustained a head injury and fractured femur develops a stress ulcer. A common clinical manifestation of this ulcer is: a. Bowel obstruction b. Bleeding c. Pulmonary embolism d. Hepatomegaly

ANS: B The most common clinical manifestation is bleeding. The most common clinical manifestation is bleeding; bowel obstruction would occur much later and would not be expected. The most common clinical manifestation is bleeding, not pulmonary embolism. The most common clinical manifestation is bleeding; hepatomegaly is not associated with stress ulcers. Cognitive Level: Comprehension 907

1. A nurse is teaching the staff about the kidneys. Which information should the nurse include? The region of the kidneys that contains the glomeruli is the: a. Medulla b. Cortex c. Pyramids d. Columns

ANS: B The outer layer of the kidney is called the cortex and it contains all of the glomeruli, most of the proximal tubules, and some segments of the distal tubule. The cortex, not the medulla, contains the glomeruli. The cortex, not the pyramids, contains the glomeruli. The cortex, not the columns, contains the glomeruli. Cognitive Level: Comprehension 724

7. A 20-year-old male underwent an echocardiogram to assess chest pain. Results revealed a congenital defect in papillary muscles. Which of the following would the nurse expect to occur? a. Closure of the semilunar valve b. Backward expulsion of the atrioventricular valves c. Closure of the atrioventricular valve d. Backward expulsion of the semilunar valves

ANS: B The papillary muscles are extensions of the myocardium that pull the cusps together and downward at the onset of ventricular contraction, thus preventing their backward expulsion into the atria. Defects in the papillary muscles would not affect the semilunar valve closure. Defects in the papillary muscles do not affect the atrioventricular valves. The papillary muscles prevent backward expulsion into the atria. They do not affect the semilunar valves. Cognitive Level: Comprehension 554

7. Which cells of the gastric glands secrete hydrochloric acid? a. Chief b. Parietal c. Zymogenic d. Surface epithelial

ANS: B The parietal cells secrete hydrochloric acid. The chief cells secrete pepsinogen. Zymogenic cells do not secrete acid. Epithelial cells line the GI tract and do not secrete acid. Cognitive Level: Comprehension 876

46. A 62-year-old female tells her health care provider she has been experiencing regular night sweats that cause her to wake up drenched. She also remarks that she has been unintentionally losing weight. Physical exam reveals enlarged lymph nodes on her neck that do not appear to be painful. She should be screened for which of the following cancers? a. Epstein-Barr virus b. Hodgkin lymphoma c. Acute leukemia d. Burkitt lymphoma

ANS: B The patient has symptoms of Hodgkin lymphoma. About a third of individuals will have some degree of systemic symptoms. Intermittent fever, without other symptoms of infection, drenching night sweats, itchy skin (pruritus), and fatigue are relatively common. The patient with Epstein-Barr virus does not have night sweats. The patient with acute leukemia does not have night sweats or enlarged lymph nodes. The patient with Burkitt lymphoma does not have enlarged lymph glands or night sweats. Cognitive Level: Comprehension 517

11. A 30-year-old female presents to her primary care provider reporting fatigue, excessive sweating, and increased appetite. Physical examination reveals protruding eyes, and laboratory testing reveals hyperthyroidism secondary to autoantibody production. This disorder falls into the category of type _____ hypersensitivity. a. I b. II c. III d. IV

ANS: B The patient is experiencing type II hypersensitivity. In hyperthyroidism, autoantibody binds to and activates receptors for thyroid-stimulating hormone (TSH) (a pituitary hormone that controls the production of the hormone thyroxine by the thyroid). The patient is experiencing a type II reaction, not type I, which is the most common type of allergic reaction. The patient is experiencing a type II reaction, not type III, which is not organ specific. The patient is experiencing a type II reaction; type IV is a delayed reaction. Cognitive Level: Comprehension 193

16. A 30-year-old female complains of fatigue, arthritis, rash, and changes in urine color. Laboratory testing reveals anemia, lymphopenia, and kidney inflammation. Assuming a diagnosis of SLE, which of the following is also likely to be present? a. Anti-LE antibodies b. Antinuclear antibodies c. Antiherpes antibodies d. Anti-CMV antibodies

ANS: B The presence of antinuclear antibodies is a diagnostic criterion for SLE. The presence of antinuclear antibodies is a diagnostic criterion for SLE. It would be positive LE. The presence of antinuclear antibodies is a diagnostic criterion for SLE, not antiherpes. The presence of antinuclear antibodies is a diagnostic criterion for SLE, not anti-CMV. Cognitive Level: Comprehension 198

19. A 75-year-old obese female presents to her primary care provider reporting edema in the lower extremities. Physical exam reveals that she has varicose veins. Upon performing the history, which of the following is a possible cause for the varicose veins? a. Extreme exercise b. Long periods of standing c. Trauma to the deep veins d. Ischemia

ANS: B The probable cause of the patients varicose veins is gradual venous distention caused by the action of gravity on blood in the legs due to long periods of standing. Varicose veins are most likely due to long periods of standing leading to the action of gravity promoting venous distention. Exercise would help prevent this. Trauma can occur, but usually this affects the more superficial veins. Ischemia affects arteries not veins. Cognitive Level: Comprehension 585

7. A 60-year-old female emphysema patient experiences a rapid and pounding heart, dizziness, and fatigue with exertion. Which of the following respiratory assessment findings indicate the respiratory system is compensating for the increased oxygen demand? a. Bronchoconstriction b. Increased rate and depth of breathing c. Dyspnea d. Activation of the renin-angiotensin response

ANS: B The rate and depth of breathing increase in an effort to increase oxygen availability accompanied by an increase in the release of oxygen from hemoglobin. Bronchodilation occurs, not constriction. Dyspnea is not a compensatory mechanism but a side effect of the bodys attempt to increase oxygen. The respiratory system does not activate the renin-angiotensin response; the kidneys are involved. Cognitive Level: Comprehension 501

2. A 27-year-old male has a severe kidney obstruction leading to removal of the affected kidney. Which of the following would the nurse expect to occur? a. Atrophy of the remaining kidney b. Compensatory hypertrophy of the remaining kidney c. Dysplasia in the remaining kidney d. Renal failure

ANS: B The remaining kidney would hypertrophy to compensate for the increased workload of the loss of the affected kidney. The remaining kidney would enlarge, not atrophy. The remaining kidney would hypertrophy, not change cell structure. The remaining kidney would hypertrophy; the patient would not go into renal failure. Cognitive Level: Comprehension 742

20. Which of the following liver cells are phagocytic? a. Glisson b. Kupffer c. Meissner d. Lieberkhn

ANS: B The sinusoids are also lined with phagocytic Kupffer cells and are part of the mononuclear phagocyte system. The Glisson is the capsule that covers the liver. The Meissner is a part of the nerve plexus that innervated the liver. The Lieberkhn is a part of the livers secretory cells. Cognitive Level: Comprehension 884

23. A new drug is released to aid in weight loss. It stimulates the sympathetic system. The nurse monitors for an increased heart rate. The sympathetic system causes this by: a. Inhibiting the release of catecholamines b. Altering the threshold potential c. Decreasing the conduction time through the atrioventricular (AV) node d. Increasing the influx of calcium

ANS: B The sympathetic system increases heart rate by altering the threshold potential. The sympathetic system increases heart rate by altering the threshold potential. It does not inhibit the release of catecholamines. The sympathetic system increases heart rate by altering the threshold potential. It does not decrease conduction through the AV node. The sympathetic system increases heart rate by altering the threshold potential. It does not increase the influx of calcium. Cognitive Level: Comprehension 560

29. During inflammation, the liver is stimulated to release plasma proteins, collectively known as: a. Opsonins b. Acute phase reactants c. Antibodies d. Phagolysosome

ANS: B The synthesis of many plasma proteins by the liver is increased during inflammation. These proteins, which can be either proinflammatory or antiinflammatory in nature, are referred to as acute phase reactants. Opsonins coat the surface of bacteria and increase their susceptibility to being phagocytized. Antibodies are proteins of the immune system. Phagolysosome destroys bacterium. Cognitive Level: Comprehension 133

38. A 15-year-old male with infectious mononucleosis is being given instructions on how to prevent the spread of this infection to others. Which statement represents a correct instruction? a. Wear a surgical mask when others are in the room. b. Do not share drinking glasses or eating utensils. c. Avoid all contact with other people. d. No precautions are necessary.

ANS: B Transmission of mononucleosis is usually through saliva from close personal contact. The virus also may be secreted in other mucosal secretions of the genital, rectal, and respiratory tract, as well as blood. A surgical mask is not necessary because the disorder is not airborne. The patient does not need to avoid all contact. Precautions should be taken. Cognitive Level: Comprehension 511

4. A 25-year-old male develops a tumor of the breast glandular tissue. What type of tumor will be documented on the chart? a. Carcinoma b. Adenocarcinoma c. Sarcoma d. Lymphoma

ANS: B Tumors that arise from or form ductal or glandular structures are named adenocarcinomas. Cancers arising in epithelial tissue are called carcinomas. Cancers arising from mesenchymal tissue (including connective tissue, muscle, and bone) usually have the suffix sarcoma. Cancers of lymphatic tissue are called lymphomas. Cognitive Level: Comprehension 223

31. A 68-year-old male presents to the ER reporting chest pain. He has a history of stable angina that now appears to be unstable. He most likely has: a. Mild to moderate atherosclerosis b. Impending myocardial infarction (MI) c. Electrical conduction problems in the heart d. Decreased myocardial oxygen demand

ANS: B Unstable angina is an indication of impending MI. Unstable angina could be caused by moderate atherosclerosis, but it is an indication of impending MI. Unstable angina could be caused by altered electrical conduction, but it is an indication of impending MI. Unstable angina is due to a decrease in myocardial oxygen, but it is an indication of impending MI.

8. When a nurse is preparing to teach about urine, which information should the nurse include? Just before entering the ureter, urine passes through the: a. Collecting duct b. Renal pelvis c. Urethra d. Major calyx

ANS: B Urine is collected in the renal pelvis and then funneled into the ureters. Urine is collected in the renal pelvis, not the collecting duct, and then funneled into the ureters. Urine is collected in the renal pelvis and then funneled into the ureters. The urethra receives urine from the bladder. Urine is collected in the renal pelvis, not the major calyx, and then funneled into the ureters. Cognitive Level: Comprehension 729

3. A 55-year-old male presents reporting urinary retention. Tests reveal that he has a lower urinary tract obstruction. Which of the following is of most concern to the nurse? a. Vesicoureteral reflux and pyelonephritis b. Formation of renal calculi c. Glomerulonephritis d. Increased bladder compliance

ANS: B Urine stasis occurs with urinary tract obstruction and can lead to the formation of renal calculi. Reflux and pyelonephritis would not lead to calculi as much as stasis. Glomerulonephritis would not lead to calculi formation. Increased compliance would not lead to calculi. Cognitive Level: Comprehension 743

12. Absorption of nutrients from the intestine occurs initially through the: a. Mesentery b. Villi c. Lamina propria d. Splenic veins

ANS: B Villi are small finger-like projections that extend from the small intestinal mucosa and increase its absorptive surface area. The ileum and jejunum are suspended in loose folds from the posterior abdominal wall by a peritoneal membrane called the mesentery. The lamina propria (a connective tissue layer of the mucous membrane) lies beneath the epithelial cells of the villi and contains lymphocytes and plasma cells, which produce immunoglobulins. The splenic vein drains the stomach and is not involved in absorption. Cognitive Level: Comprehension 878

34. Which information indicates a correct understanding of viral vaccines? Most viral vaccines contain: a. Active viruses b. Attenuated viruses c. Killed viruses d. Viral toxins

ANS: B Viral vaccines contain live viruses that are weakened (attenuated). Viral vaccines contain live viruses that are weakened (attenuated), not active. Viral vaccines contain live viruses that are weakened (attenuated), not killed. Viral vaccines contain live viruses that are weakened (attenuated), not toxins. Cognitive Level: Comprehension 187

7. Reflux esophagitis is defined as a(n): a. Immune response to gastroesophageal reflux b. Inflammatory response to gastroesophageal reflux c. Congenital anomaly d. Secretory response to hiatal hernia

ANS: B When gastroesophageal reflux leads to an inflammatory response, it is termed reflux esophagitis. Esophagitis is due to an inflammatory response, not immune. Esophagitis is due to an inflammatory response, not a congenital anomaly. Esophagitis is due to an inflammatory response, not a secretory response. Cognitive Level: Comprehension 899

24. Which sphincter, when it relaxes, allows bile to flow into the duodenum? a. Pyloric sphincter b. Sphincter of Oddi c. Ampulla of Vater d. Ileocecal valve

ANS: B When the Sphincter of Oddi relaxes, bile flows into the duodenum. The pyloric sphincter controls flow of fluid from the stomach to the duodenum. Secretions from the pancreas empty into the common bile duct at the ampulla of Vater. The ileocecal valve prevents reflux of digested material from the colon into the small intestine. Cognitive Level: Comprehension 887

17. A nurse is reviewing urinalysis results and notices glucose is present in the urine. A nurse realizes glucose will be excreted in the urine when: a. The maximum rate of glucose filtration is achieved b. The carrier molecules have reached their maximum c. Glucose is consumed d. The ability of the kidneys to regulate blood glucose is lost

ANS: B When the carrier molecules for glucose become saturated (i.e., with the development of hyperglycemia), the excess will be excreted in the urine. There is not a maximum rate of glucose filtration. Glucose should not be found in the urine. The kidneys do not regulate blood glucose. Cognitive Level: Comprehension 733

10. After a geneticist talks to the patient about being a chromosomal mosaic, the patient asks the nurse what that means. How should the nurse respond? You may _____ genetic disease(s). a. Only be a carrier of the b. Have a mild form of the c. Have two d. Be sterile as a result of the

ANS: B A chromosomal mosaic means the body has two or more different cell lines, each of which has a different karyotype; thus, the person has a mild form of the disease. Mosaics are not only carriers; they have the disease. Mosaics have two different lines but not two different diseases. Mosaics are not sterile.

10. A nurse is teaching the staff about antidiuretic hormone (ADH). Which information should the nurse include? Secretion of ADH is stimulated by: a. Increased serum potassium b. Increased plasma osmolality c. Decreased renal blood flow d. Generalized edema

ANS: B ADH is secreted when plasma osmolality increases or circulating blood volume decreases and blood pressure drops. ADH is secreted when plasma osmolality increases, not by an increase in potassium. ADH is secreted when plasma osmolality increases; it is not affected by decreased renal blood flow. Edema does not affect the secretion of ADH. Cognitive Level: Comprehension 104

12. A 25-year-old male is diagnosed with a hormone-secreting tumor of the adrenal cortex. Which finding would the nurse expect to see in the lab results? a. Decreased blood volume b. Decreased blood K+levels c. Increased urine Na+levels d. Increased white blood cells

ANS: B Aldosterone is secreted from the adrenal cortex. It promotes renal sodium and water reabsorption and excretion of potassium, leading to decreased potassium levels. Blood volume actually increases with aldosterone secretion. Aldosterone promotes sodium reabsorption, leading to normal or decreased Na+levels. Aldosterone is not associated with white blood cells. Cognitive Level: Comprehension 103

17. A 20-year-old male presents to the emergency department with a jagged sharp-force injury that is longer than it is deep. Which type of wound will the nurse be caring for? a. Stab wound b. Incised wound c. Puncture wound d. Chopping wound

ANS: B An incised wound is a cut that is longer than it is deep. A stab wound is a penetrating sharp-force injury that is deeper than it is long. A puncture wound is without sharp edges and is made with an instrument like a nail. Heavy, edged instruments (axes, hatchets, propeller blades) produce wounds with a combination of sharp and blunt force characteristics.

5. When planning care for the pregnant patient, the nurse will recall that the mammary glands enlarge as a consequence of: a. Compensatory hyperplasia b. Hormonal hyperplasia c. Hormonal anaplasia d. Compensatory anaplasia

ANS: B An increase in the mammary glands during pregnancy is a result of hormonal changes. The number of mammary cells increases in response to increased hormone levels. Hyperplasia is correct, but it is not a compensatory response, but a hormonal response. Hormonal is correct, but the growth is due to an increased number of cells, not enlargement. Compensatory is not correct, nor is anaplasia, a reversal to more immature cells. Mammary enlargement is a normal response.

1. A report comes back indicating that muscular atrophy has occurred. A nurse recalls that muscular atrophy involves a decrease in muscle cell: a. Number b. Size c. Vacuoles d. Lipofuscin

ANS: B Atrophy is a decrease or shrinkage in cellular size. Hyperplasia is an increase in the number of cells. Vacuoles are membrane-bound vesicles within the cell that contain cellular debris and hydrolytic enzymes. Lipofuscin is the yellow-brown age pigment.

25. A 2-year-old swallowed watch batteries. Following ingestion, kidney function was impaired, and the heart began to fail. Which of the following was the most likely cause? a. Karyorrhexis b. Coagulative necrosis c. Ammonia accumulation d. Caseous necrosis

ANS: B Coagulative necrosis occurs primarily in the kidneys, heart, and adrenal glands and commonly results from hypoxia. Karyorrhexis means fragmentation of the nucleus into smaller particles or nuclear dust. Ammonia accumulation is not associated with this toxicity. Caseous necrosis results from tuberculosis pulmonary infection.

1. A nurse recalls the basic components of DNA are: a. Pentose sugars and four phosphate bases b. A phosphate molecule, deoxyribose, and four nitrogenous bases c. Adenine, guanine, and purine d. Codons, oxygen, and cytosine

ANS: B DNA has three basic components: the pentose sugar molecule, deoxyribose; a phosphate molecule; and four types of nitrogenous bases. DNA contains four nitrogenous bases, not phosphate bases. Adenine and quinine are purines and are only a portion of the components of DNA. DNA synthesizes body protein, of which a codon is a component.

15. Which of the following patients is the most at risk for developing hypernatremia? A patient with: a. Vomiting b. Diuretic use c. Dehydration d. Hypoaldosteronism

ANS: C Dehydration leads to hypernatremia because an increase in sodium occurs with a net loss in water. Vomiting leads to hyponatremia. Diuretic use would lead to sodium loss. Hypoaldosteronism leads to hyponatremia. Cognitive Level: Comprehension 106

24. A 19-year-old male presents to his primary care provider reporting restlessness, muscle cramping, and diarrhea. Lab tests reveal that he is hyperkalemic. Which of the following could have caused his condition? a. Primary hyperaldosteronism b. Acidosis c. Insulin secretion d. Diuretic use

ANS: B During acute acidosis, hydrogen ions accumulate in the ICF and potassium shifts out of the cell to the ECF, causing hyperkalemia. Primary hyperaldosteronism is associated with hypokalemia, not hyperkalemia. Insulin secretion helps reduce potassium levels in the cell, not cause it. Diuretics would cause hypokalemia, not hyperkalemia. Cognitive Level: Comprehension 110

17. An aide asks the nurse why people who have neurofibromatosis will show varying degrees of the disease. Which genetic principle should the nurse explain to the aide? a. Penetrance b. Expressivity c. Dominance d. Recessiveness

ANS: B Expressivity is the extent of variation in phenotype associated with a particular genotype. For neurofibromatosis, a variety of manifestations occur among individuals. The penetrance of a trait is the percentage of individuals with a specific genotype who also exhibit the expected phenotype. Dominance refers to observable traits and risk of transmission. Recessiveness refers to silent strains with reduced risk of occurrence.

18. A 52-year-old diabetic male presents to the ER with lethargy, confusion, and depressed reflexes. His wife indicates that he does not follow the prescribed diet and takes his medication sporadically. Lab results indicate hyperglycemia. Which assessment finding is most likely to occur? a. Clammy skin b. Decreased sodium c. Decreased urine formation d. Metabolic alkalosis

ANS: B Hypertonic hyponatremia develops with hyperglycemia. Increases in plasma lipids displace water volume and decrease sodium concentration, leading to the symptoms described. The patient is experiencing symptoms of hyponatremia and hyperglycemia, not hypernatremia and hypoglycemia. The patient will have increased ECF and would have increased urine formation. Metabolic acidosis would occur, not alkalosis. Cognitive Level: Comprehension 107

17. Hyperlipidemia and hyperglycemia are associated with: a. Hypernatremia b. Hypertonic hyponatremia c. Hypokalemia d. Acidosis

ANS: B Hypertonic hyponatremia develops with hyperlipidemia, hyperproteinemia, and hyperglycemia. Increases in plasma lipids displace water volume and decrease sodium concentration. Hyperlipidemia and hyperglycemia are associated with hyponatremia, not hypernatremia. Hyperlipidemia and hyperglycemia are associated with hyponatremia, not hypokalemia. Hyperlipidemia and hyperglycemia are associated with hyponatremia, not acidosis. Cognitive Level: Comprehension 107

21. Which of the following conditions would cause the nurse to monitor for hyperkalemia? a. Excess aldosterone b. Acute acidosis c. Insulin usage d. Metabolic alkalosis

ANS: B In acidosis, ECF hydrogen ions shift into the cells in exchange for ICF potassium and sodium; hyperkalemia and acidosis therefore often occur together. Acidosis causes hyperkalemia, not excess aldosterone. Insulin would help treat hyperkalemia, not cause it. It is acidosis, not alkalosis, that leads to hyperkalemia. Cognitive Level: Comprehension 110

31. For a patient experiencing metabolic acidosis, the body will compensate by: a. Excreting H+ through the kidneys b. Hyperventilating c. Retaining CO2 in the lungs d. Secreting aldosterone

ANS: B In an attempt to compensate for metabolic acidosis, the lungs hyperventilate to blow off CO2. It is the lungs hyperventilating that would compensate for metabolic acidosis, not the kidneys. CO2 retention would increase the acidotic state. Aldosterone would conserve water, but does not help compensate for acidosis. REF: pp. 112-113

28. While reading a textbook, the student reads the term, apoptosis. The student recalls that apoptosis is a condition in which cells program themselves to: a. Atrophy b. Die c. Regenerate d. Age

ANS: B In apoptosis, cells are programmed to die. In apoptosis, cells do not atrophy; they die. In apoptosis, cells do not regenerate; they die. In apoptosis, cells do not age; they die.

14. An XXY person asks the nurse what this genetic disorder is called. What is the nurses best response? This disorder is _____ syndrome. a. Turner b. Klinefelter c. Down d. Fragile X

ANS: B Individuals with at least two X chromosomes and one Y chromosome in each cell (47 XXY karyotype) have a disorder known as Klinefelter syndrome. A condition with the presence of a single X chromosome and no homologous X or Y chromosome, so the individual has a total of 45 chromosomes, is known as Turner syndrome. Down syndrome is a trisomy. Fragile X syndrome is due to a break or a gap in a chromosome, not an extra chromosome.

29. The regions of the heterogeneous nuclear RNA that must be spliced out to form functional RNA are called: a. Promoter sites b. Introns c. Exons d. Anticodon

ANS: B Introns are spliced from the mRNA before the mRNA leaves the nucleus. A promoter site is a sequence of DNA that specifies the beginning of a gene. In eukaryotes, many RNA sequences are removed by nuclear enzymes; the sequences that are left to code for proteins are called exons. In translation, RNA directs the synthesis of a polypeptide, a cloverleaf-shaped strand of about 80 nucleotides. The tRNA molecule has a site where an amino acid attaches. The three-nucleotide sequence at the opposite side of the cloverleaf is called the anticodon.

4. Which of the following patients should the nurse assess for a decreased oncotic pressure in the capillaries? A patient with: a. A high-protein diet b. Liver failure c. Low blood pressure d. Low blood glucose

ANS: B Liver failure leads to lost or diminished plasma albumin production, and this contributes to decreased plasma oncotic pressure. A high-protein diet would provide albumin for the maintenance of oncotic pressure. Low blood pressure would lead to decreased hydrostatic pressure. Decreased glucose does not affect oncotic pressure. Cognitive Level: Comprehension 101

13. A patient has been searching on the Internet about natriuretic hormones. When the patient asks the nurse what do these hormones do, how should the nurse respond? Natriuretic hormones affect the balance of: a. Calcium b. Sodium c. Magnesium d. Potassium

ANS: B Natriuretic hormones are sometimes called a third factor in sodium regulation. Natriuretic hormones are a factor in sodium balance, not calcium. Natriuretic hormones are a factor in sodium balance, not magnesium. Natriuretic hormones are a factor in sodium balance, not potassium. Cognitive Level: Comprehension 103

Visual disturbances are a common occurrence in patients with untreated Graves disease. The endocrinologist explains to the patient that the main cause of these complications is: a. Decreased blood flow to the eye b. Orbital edema and protrusion of the eyeball c. TSH neurotoxicity to retinal cells d. Local lactic acidosis

B

6. When homologous chromosomes fail to separate during meiosis, which of the following occurs? a. Neurofibromatosis b. Nondisjunction c. Polyploidy d. Conjoined twins

ANS: B Nondisjunction is an error in which homologous chromosomes or sister chromatids fail to separate normally during meiosis or mitosis. Neurofibromatosis is a dominant disorder. It is not due to chromosome failure during meiosis. Polyploidy occurs when a euploid cell has more than the diploid number of chromosomes. Conjoined twins are not due to chromosome failure during meiosis.

9. The condition in which an extra portion of a chromosome is present in each cell is called: a. Reciprocal translocation b. Partial trisomy c. Inversion d. Down syndrome

ANS: B Partial trisomy is a condition in which only an extra portion of a chromosome is present in each cell. A reciprocal translocation occurs when breaks take place in two different chromosomes and the material is exchanged. An inversion occurs when two breaks take place on a chromosome, followed by the reinsertion of the missing fragment at its original site, but in inverted order. Down syndrome is an aneuploidy of the twenty-first chromosome.

24. To express a polygenic trait: a. Genes must interact with the environment. b. Several genes must act together. c. Multiple mutations must occur in the same family. d. Penetrance must occur.

ANS: B Polygenic traits are those that result from several genes acting together. When environmental factors influence the expression of the trait, the term multifactorial inheritance is used. When multiple mutations occur in the same family, the mechanism most likely responsible is termed germline mosaicism. Penetrance of a trait is the percentage of individuals with a specific genotype who also exhibit the expected phenotype.

21. A 35-year-old female is diagnosed with multiple myeloma. Biopsy of the tumor reveals Russell bodies, and laboratory testing reveals kidney dysfunction. Which substance should the nurse monitor as it is accumulating in the patients body? a. Glycogen b. Protein c. Pigment d. Melanin

ANS: B Russell bodies occur due to excess aggregates of protein. Excess glycogen would affect blood glucose and this is not related to Russell bodies. Increased pigment would not lead to kidney dysfunction. Melanin accumulates in epithelial cells (keratinocytes) of the skin and retina. It is an extremely important pigment because it protects the skin against long exposure to sunlight.

14. A family presents to their primary care provider reporting headache, nausea, weakness, and vomiting. Which of the following would be the most likely explanation for these symptoms? a. Lead exposure b. Carbon monoxide poisoning c. Ethanol exposure d. Mercury poisoning

ANS: B Symptoms related to carbon monoxide poisoning include headache, giddiness, tinnitus (ringing in the ears), nausea, weakness, and vomiting. Although nausea and vomiting can occur with lead exposure, lead toxicity is primarily manifested by convulsions and delirium and, with peripheral nerve involvement, wrist, finger, and sometimes foot paralysis. Ethanol exposure has CNS effects and would not affect the whole family. Mercury poisoning is manifested by CNS effects and would not lead to nausea and vomiting.

26. Which of the following buffer pairs is considered the major plasma buffering system? a. Protein/fat b. Carbonic acid/bicarbonate c. Sodium/potassium d. Amylase/albumin

ANS: B The carbonic acid/bicarbonate buffer pair operates in both the lung and the kidney and is a major extracellular buffer. Protein and fat are nutrients not related to the buffering system. Sodium and potassium are electrolytes for fluid and electrolyte balance, not the major plasma buffering system for acid-base balance. Amylase is a carbohydrate enzyme, and albumin is a protein; neither is a buffering system.

4. A DNA strand has a region with the sequence ATCGGAT. Which of the following would be a complementary strand? a. CGATACGT b. TAGCCTAG c. TUGCCTUG d. UAGCCUAG

ANS: B The consistent pairing of adenine with thymine and of guanine with cytosine is known as complementary base pairing; thus, A complements to T and C to G and vice versa throughout the strand. A complements to T; thus, the first letter must be a T. U does not represent a complement in the sequence. U does not represent a complement in the sequence.

3. While planning care for elderly individuals, the nurse remembers the elderly are at a higher risk for developing dehydration because they have a(n): a. Higher total body water volume b. Decreased muscle mass c. Increase in thirst d. Increased tendency towards developing edema

ANS: B The elderly are at higher risk for dehydration due to a decrease in muscle mass. The elderly have a decrease in total body water, not an increase. The elderly have a decrease in thirst. The elderly may develop edema, but this does not lead to dehydration. Cognitive Level: Comprehension 100

21. A child is born with blue eyes (bb). The childs mother has blue eyes and the father has brown eyes. Which of the following represents the father? a. Bb b. Bb c. BB d. Bbb

ANS: B The father would need to have one b and one B because blue eyes are recessive and the father has a dominant eye color. bb is incorrect because, with this description, the father could not have brown eyes. BB would describe the fathers brown eyes, but without a b he could not father a blue-eyed child. The father would have only two, not three.

27. The gradual increase in height among the human population over the past 100 years is an example of: a. A polygenic trait b. A multifactorial trait c. Crossing over d. Recombination

ANS: B The gradual increase in height is an example of multifactorial traits influenced by genes and also by environment. Polygenic traits result from several genes acting together. Crossing over is an abnormal chromosome structure. Recombination results from new arrangements of alleles.

32. When the nurse is checking the urinalysis, the finding that would alert the nurse to cellular injury is the presence of: a. Slight glucose b. Excessive protein c. Blood d. Urea

ANS: B The presence of protein in the urine in significant amounts indicates cellular injury and altered cellular function. Glucose should not normally be present in the urine, but its presence is not indicative of altered cellular function, but of excess glucose in the blood. Blood should not normally be present in the urine, but its presence indicates kidney alteration and not cellular injury. Urea is an expected substance in the kidney.

1. Brachytherapy is being used to treat cancer in a patient. What type of cancers responds well to brachytherapy? (Select all that apply.) a. Heart b. Cervix c. Head d. Neck e. Lung

ANS: B, C, D Radiation sources can be temporarily placed into body cavities through a delivery method termed brachytherapy. Brachytherapy is useful in the treatment of cervical, prostate, and head and neck cancers.

1. When teaching about the stress-age syndrome, what information should the nurse include? (Select all that apply.) a. Decreased cortisol secretion b. Increased thyroxine c. Immunodepression d. Increased catecholamine secretion e. Hypercoagulation of the blood f. Free radical damage

ANS: B, C, D, E, F The stress-age syndrome includes alterations in the excitability of structures of the limbic system and hypothalamus; rise of the blood concentration of catecholamines, ADH, ACTH, and cortisol; decrease in testosterone, thyroxine, and others; alterations of opioid peptides; immunodepression and pattern of chronic inflammation; alterations in lipoproteins; hypercoagulation of the blood; and free radical damage of cells. Cognitive Level: Comprehension 218

1. While planning care for an elderly patient, the nurse remembers that increased age is associated with (select all that apply): a. Increased T cell function b. Decreased immune function c. Increased production of antibodies d. Decreased numbers of circulating immune complexes e. Decreased ability to fight infection

ANS: B, D, E Increased age is associated with diminished T cell function, decreased immune function, diminished production of antibody responses, decreased circulating immune complexes, and decreased ability to fight infection. Cognitive Level: Comprehension 164

19. When a nurse notices that a patient has type O blood, the nurse realizes that anti-_____ antibodies are present in the patients body. a. A only b. B only c. A and B d. O

ANS: C Type O individuals have both anti-A and anti-B antibodies. Type O individuals have both A and B antibodies. Type O individuals have both A and B antibodies. Type O individuals will have A and B antibodies, but not O. Cognitive Level: Comprehension 198

33. A 54-year-old male with a long history of smoking complains of excessive tiredness, shortness of breath, and overall ill feelings. Lab results reveal decreased pH, increased CO2, and normal bicarbonate ion. These findings help to confirm the diagnosis of: a. Respiratory alkalosis b. Metabolic acidosis c. Respiratory acidosis d. Metabolic alkalosis

ANS: C A decreased pH indicates acidosis. With increased CO2, it is respiratory acidosis. The decreased pH indicates acidosis, not alkalosis. It is acidosis, but the bicarbonate is normal, so it cannot be metabolic. The decreased pH indicates acidosis, not alkalosis. Cognitive Level: Comprehension 114

15. What term should the nurse document for a detached blood clot? a. Thrombus b. Embolus c. Thromboembolus d. Infarction

ANS: C A detached thrombus is a thromboembolus. A thrombus is a clot that remains attached to a vessel wall; a detached thrombus is a thromboembolus. An embolus is a bolus of material floating in the blood stream; a detached thrombus is a thromboembolus. An infarction is death of tissue; a detached thrombus is a thromboembolus. Cognitive Level: Comprehension 593

13. A 40-year-old male develops an intestinal obstruction related to protrusion of the intestine through the inguinal ring. This condition is referred to as: a. Intussusception b. A volvulus c. A hernia d. Adhesions

ANS: C A hernia is a protrusion of the intestine through a weakness in the abdominal muscles or through the inguinal ring. Intussusception is telescoping of one part of the intestine into another; this usually causes strangulation of the blood supply and is more common in infants 10 to15 months of age than in adults. A volvulus is a twisting of the intestine on its mesenteric pedicle, with occlusion of the blood supply, often associated with fibrous adhesions. It occurs most often in middle-aged and elderly men. Adhesions are irritation from surgery or trauma that leads to formation of fibrin and adhesions that attach to intestine, omentum, or peritoneum and can cause obstruction; they are most common in small intestine. Cognitive Level: Comprehension 900

18. A patient asks when adjuvant chemotherapy is used. How should the nurse respond? Adjuvant chemotherapy treatment is used: a. As the primary treatment b. Before radiation therapy c. After surgical removal of a tumor d. In cancer with little risk of metastasis

ANS: C Adjuvant chemotherapy is given after surgical excision of a cancer with the goal of eliminating micrometastases. Adjuvant chemotherapy is given after surgery, not as the primary treatment. Adjuvant chemotherapy is given after surgery, not before radiation therapy. Adjuvant chemotherapy is given after surgery and is indicated in the treatment of individuals with metastasis. REF: pp. 248-249

3. When the maternal immune system becomes sensitized against antigens expressed by the fetus, what type of immune reaction occurs? a. Autoimmune b. Anaphylaxis c. Alloimmune d. Allergic

ANS: C Alloimmunity can be observed during immunologic reactions against transfusions, transplanted tissue, or the fetus during pregnancy. Autoimmunity is a disturbance in the immunologic tolerance of self-antigens. The most rapid and severe immediate hypersensitivity reaction is anaphylaxis. An allergic response occurs related to exposure to an allergen. Cognitive Level: Comprehension 190

6. A 10-year-old male is stung by a bee while playing in the yard. He begins itching and develops pain, swelling, redness, and respiratory difficulties. He is suffering from: a. Immunodeficiency b. Autoimmunity c. Anaphylaxis d. Tissue-specific hypersensitivity

ANS: C Anaphylaxis occurs within minutes of reexposure to the antigen and can be either systemic (generalized) or cutaneous (localized). Immunodeficiency is a decrease in the immune response. Autoimmunity is a disturbance in the immunologic tolerance of self-antigens. Tissue-specific reaction is an autoimmune reaction. Cognitive Level: Comprehension 190

1. A patients anemia is described as having erythrocytes that demonstrate anisocytosis. The nurse would recognize the erythrocytes would be: a. Pale in color b. Present in various sizes c. Able to assume various shapes d. Live only a few days

ANS: C Anisocytosis means the erythrocytes are able to assume various shapes. Anisocytosis means the erythrocytes are able to assume various shapes; it does not refer to color. Anisocytosis means the erythrocytes are able to assume various shapes; it does not refer to size. Anisocytosis means the erythrocytes are able to assume various shapes; it does not refer to life span.

26. A 16-year-old female presents with abdominal pain in the right lower quadrant. Physical examination reveals rebound tenderness and a low-grade fever. A possible diagnosis would be: a. Colon cancer b. Pancreatitis c. Appendicitis d. Hepatitis

ANS: C Appendicitis is manifested by right lower quadrant pain with rebound tenderness. Colon cancer may be asymptomatic, followed by bleeding. Pancreatitis is manifested by vomiting. Hepatitis would be manifested by upper abdominal pain, not lower. Cognitive Level: Comprehension 910

What common neurologic disturbances should the nurse assess for in a patient with a pituitary adenoma? a. Coma b. Visual disturbances c. Confused states d. Breathing abnormalities

B

13. A 25-year-old female experiences a headache and takes aspirin for relief. A nurse recalls aspirin relieves the headache by: a. Decreasing leukotriene production b. Increasing histamine release c. Decreasing prostaglandin production d. Increasing platelet-activating factor

ANS: C Aspirin is a prostaglandin inhibitor. Aspirin inhibits prostaglandins; it does not affect leukotriene production. Aspirin inhibits prostaglandins; it does not affect histamine release. Aspirin does not play a role in the platelet activating factor; this is a leukotriene response. Cognitive Level: Comprehension 129

1. A 60-year-old male presents to his primary care provider reporting chest pain. He is diagnosed with atherosclerosis. This disease is caused by: a. Arterial wall thinning and weakening b. Abnormally dilated arteries and veins c. Abnormal thickening and hardening of vessel walls d. Autonomic nervous system imbalances

ANS: C Atherosclerosis is a form of arteriosclerosis characterized by thickening and hardening of the vessel wall. Atherosclerosis is a form of arteriosclerosis characterized by thickening and hardening of the vessel wall. Arteries are thickened, not thinning. Atherosclerosis is a form of arteriosclerosis characterized by thickening and hardening of the vessel wall. Arteries are narrowed, not dilated. Atherosclerosis is a form of arteriosclerosis characterized by thickening and hardening of the vessel wall. It is not related to autonomic nervous system imbalances, which would lead to changes in rate or rhythm. Cognitive Level: Comprehension 594

5. A 50-year-old female develops skin cancer on her head and neck following years of sunbathing. Which of the following cancers is the most likely? a. Lymphoma b. Adenoma c. Basal cell carcinoma d. Leukemia

ANS: C Basal cell is related to UV radiation primarily from the sun. Lymphoma is not related to sunbathing. Adenoma is not related to sunbathing. Leukemia is not related to sunbathing. Cognitive Level: Comprehension 274

21. The primary bile acids are synthesized from _______ by hepatocytes lining the bile canaliculi. a. Lecithin b. Fatty acids c. Cholesterol d. Testosterone

ANS: C Bile acids are synthesized from cholesterol. Bile acids are synthesized from cholesterol, not lecithin. Bile acids are synthesized from cholesterol, not fatty acids. Bile acids are synthesized from cholesterol, not testosterone. Cognitive Level: Comprehension 884

5. A 30-year-old female is diagnosed with cancer. Testing reveals that the cancer cells have spread to local lymph nodes. A nurse realizes this cancer would be documented as stage: a. 1 b. 2 c. 3 d. 4

ANS: C Cancer that has spread to regional structures, such as lymph nodes, is stage 3. Cancer confined to the organ of origin is stage 1. Cancer that is locally invasive is stage 2. Cancer that has spread to distant sites, such as a liver cancer spreading to lung or a prostate cancer spreading to bone, is stage 4. Cognitive Level: Comprehension 244

14. A 60-year-old male is diagnosed with cerebral aneurysm. Where does the nurse suspect the cerebral aneurysm is located? a. Vertebral arteries b. Basilar artery c. Circle of Willis d. Carotid arteries

ANS: C Cerebral aneurysms often occur in the circle of Willis. Cerebral aneurysms often occur in the circle of Willis, not the vertebral arteries. Cerebral aneurysms often occur in the circle of Willis, not the basilar artery. Cerebral aneurysms often occur in the circle of Willis, not the carotid arteries. Cognitive Level: Comprehension 592

40. A 5-year-old female is diagnosed with acute leukemia. The nurse will most likely treat this patient with: a. Bone marrow transplant b. Immunotherapy c. Chemotherapy d. Localized radiation therapy

ANS: C Chemotherapy, used in various combinations, is the treatment of choice for leukemia. Bone marrow transplant may be used, but chemotherapy is the treatment of choice for leukemia. Chemotherapy, used in various combinations, is the treatment of choice for leukemia; immunotherapy is not a treatment of choice. Chemotherapy, used in various combinations, is the treatment of choice for leukemia; localized radiation therapy is not the treatment of choice. Cognitive Level: Comprehension 513

28. A 62-year-old male presents to his primary care provider reporting chest pain at rest and with exertion. He does not have a history of coronary artery disease and reports that the pain often occurs at night. He is most likely experiencing which type of angina? a. Unstable b. Stable c. Prinzmetal d. Silent

ANS: C Chest pain that occurs at rest and at night is descriptive of Prinzmetal angina. Unstable angina is a form of acute coronary syndrome that results from reversible myocardial ischemia. Stable angina is predictable and occurs with activity. Silent angina has few, if any, symptoms. Cognitive Level: Comprehension 601

28. Anemia accompanies chronic renal failure because of: a. Blood loss via the urine b. Renal insensitivity to vitamin D c. Inadequate production of erythropoietin d. Inadequate retention of serum iron

ANS: C Chronic renal failure leads to anemia because of declining erythropoietin production. Chronic renal failure leads to anemia because of declining erythropoietin production. It is not due to blood loss. Chronic renal failure leads to anemia because of declining erythropoietin production. It is not due to insensitivity to vitamin D. Chronic renal failure leads to anemia because of declining erythropoietin production. It is not due to inadequate retention of serum iron. Cognitive Level: Comprehension 760

2. Which statement indicates teaching was successful regarding collectins? Collectins are produced by the: a. Kidneys b. Bowel c. Lungs d. Integument

ANS: C Collectins are produced by the lungs. Collectins are produced by the lungs, not the kidneys. Collectins are produced by the lungs, not the bowel. Collectins are produced by the lungs, not the integument. Cognitive Level: Comprehension 119

22. When an immunologist teaches about the relationship that benefits the organism but causes no harm to the host, the immunologist is describing which of the following? a. Symbiosis b. Mutualism c. Commensalism d. Pathogenicity

ANS: C Commensalism benefits only the microorganism; there is no harm to the human. Parasitic microorganisms establish symbiosis with another species in which the parasite benefits at the expense of the other species. Mutualism benefits the human and the microorganism. Pathogenicity benefits the microorganism, but harms the human. (Opportunism is the situation that occurs when benign microorganisms become pathogenic because of decreased human-host resistance.) Cognitive Level: Comprehension 166

34. Confirmation of somatic death is based on: a. Presence of algor mortis b. Presence of livor mortis c. Complete cessation of respiration and circulation d. Change in skin color to pale yellow

ANS: C The most notable manifestations are complete cessation of respiration and circulation. Algor mortis is postmortem reduction of body temperature and is not confirmation of somatic death.

37. Complete obstruction of bile flow to the liver would be manifested by: a. Elevated hemoglobin and hematocrit b. Lower-leg edema c. Clay-colored stools d. Hypotension

ANS: C Complete obstruction of bile flow leads to clay-colored stools. Complete obstruction of bile flow leads to clay-colored stools, not elevated hemoglobin and hematocrit. Complete obstruction of bile flow leads to clay-colored stools, not lower-leg edema. Complete obstruction of bile flow leads to clay-colored stools, not hypotension. Cognitive Level: Comprehension 920

28. A 67-year-old female is admitted to the emergency department with a diagnosis of polycythemia vera. Upon taking the history, the patient will most likely report: a. Hyperactivity b. Decreased blood pressure c. Chest pain d. A pale skin color

ANS: C Coronary blood flow may be affected, precipitating angina. The patient will experience signs of interrupted blood flow due to increased blood viscosity. They will not experience hyperactivity. Blood pressure will be increased, not decreased. The skin will be red over the face hands, feet, and ears. Cognitive Level: Comprehension 507

26. A nurse recalls posthemorrhagic anemia can result in death when a patients blood loss is in excess of: a. 20% to 29% b. 30% to 39% c. 40% to 49% d. 50% to 59%

ANS: C Death can occur if blood loss exceeds 40% to 49% of plasma volume. Death can occur if blood loss exceeds 40% to 49% of plasma volume, not 20% to 29%. Death can occur if blood loss exceeds 40% to 49% of plasma volume, not 30% to 39%. Death can occur if blood loss exceeds 40% to 49% of plasma volume, not 50% to 59%. Cognitive Level: Comprehension 506

8. A nurse recalls the mast cell, a major activator of inflammation, initiates the inflammatory response through the process of: a. Chemotaxis b. Endocytosis c. Degranulation d. Opsonization

ANS: C Degranulation of mast cells is a major cellular component of inflammation. Chemotaxis is the process of white cell migration. Endocytosis is a part of phagocytosis and is not a factor in mast cell response. Opsonization is part of phagocytosis and is not a factor in mast cell response. Cognitive Level: Comprehension 127

8. A 52-year-old male is diagnosed with primary hypertension. He has no other health problems. Present treatment would cause the nurse to anticipate administering which drug to the patient? a. A beta-adrenergic agonist b. An alpha-adrenergic agonist c. A diuretic d. A calcium channel agonist

ANS: C Diuretics have been shown to be the safest and most effective medications for lowering blood pressure and preventing the cardiovascular complications of hypertension. Diuretics have been shown to be the safest and most effective medications for lowering blood pressure and preventing the cardiovascular complications of hypertension. A beta-adrenergic drug would be used for patients with other concurrent health problems. Diuretics have been shown to be the safest and most effective medications for lowering blood pressure and preventing the cardiovascular complications of hypertension. An alpha-adrenergic drug would be used for patients with other concurrent health problems. Diuretics have been shown to be the safest and most effective medications for lowering blood pressure and preventing the cardiovascular complications of hypertension. A calcium channel agonist drug would be used for patients with other concurrent health problems. Cognitive Level: Comprehension 591

3. A nurse is discussing preinvasive epithelial tumors of glandular or squamous cell origin. What is the nurse describing? a. Tumor in differentiation b. Dysplastic c. Cancer in situ d. Cancer beyond (meta) situ

ANS: C Early stage growths that are localized to the epithelium and have not invaded are called cancer in situ. Cancer in situ is early stage growth and not a tumor in differentiation. Cancer in situ is early stage growth and may contain dysplastic cells, but dysplastic cells do not define cancer in situ. Cancer in situ is early stage growth, and cancer beyond situ is more mature growth. Cognitive Level: Comprehension 223

14. When assessing the effects of elevated -endorphins in a patient, which of the following should the nurse monitor? a. Peripheral vasoconstriction b. Hyperglycemia c. Pain inhibition d. Decreased immune cell activity

ANS: C Elevated -endorphins activate endorphin (opiate) receptors on peripheral sensory nerves leading to pain relief or analgesia. Elevated -endorphins activate endorphin (opiate) receptors on peripheral sensory nerves leading to pain relief or analgesia, not vasoconstriction. Elevated -endorphins activate endorphin (opiate) receptors on peripheral sensory nerves leading to pain relief or analgesia, not hyperglycemia. Elevated -endorphins activate endorphin (opiate) receptors on peripheral sensory nerves leading to pain relief or analgesia, not decreased immune cell activity. Cognitive Level: Comprehension 214

7. Which of the following hormones activates adrenergic receptors? a. Oxytocin b. Prolactin c. Epinephrine d. Somatotropin

ANS: C Epinephrine activates adrenergic receptors. Oxytocin activates receptors in the uterus. Prolactin does not activate adrenergic receptors. Somatotropin activates protein and carbohydrate metabolism. Cognitive Level: Comprehension 209

4. Biochemical secretions that trap and kill microorganisms include: a. Hormones b. Neurotransmitters c. Earwax d. Gastric acid

ANS: C Epithelial cells secrete several substances that protect against infection, including earwax. Hormones do not contain biochemical secretions that trap and kill microorganisms. Neurotransmitters carry important messages, but they do not contain biochemical secretions. Gastric acid helps break down food into its component parts, but does not contain biochemical secretions. Cognitive Level: Comprehension 120

2. While reviewing lab results, the nurse recalls the most abundant cells in the blood are: a. Leukocytes b. Lymphocytes c. Erythrocytes d. Thrombocytes

ANS: C Erythrocytes (red blood cells) are the most abundant cells of the blood, occupying approximately 48% of the blood. Erythrocytes, not leukocytes, are the most abundant cells in the blood. Erythrocytes, not lymphocytes, are the most abundant cells in the blood. Erythrocytes, not thrombocytes, are the most abundant cells in the blood. Cognitive Level: Comprehension 480

18. A nurse is preparing to teach about erythropoietin. Which information should the nurse include? Erythropoietin is produced in the: a. Liver b. Bone marrow c. Kidneys d. Spleen

ANS: C Erythropoietin is produced in the kidneys. Erythropoietin is produced in the kidneys, not the liver. Erythropoietin is produced in the kidneys, not the bone marrow. Erythropoietin is produced in the kidneys, not the spleen. Cognitive Level: Comprehension 486

23. A 12-year-old male is diagnosed with Klinefelter syndrome. His karyotype would reveal which of the following? a. XY b. XX c. XYY d. XXY

ANS: D A person with Klinefelter syndrome has an XXY karyotype. An XY is a normal male. An XX is a normal female. An XYY is an aneuploid karyotype.

25. A patient has searched the Internet for hormones. The patient has a good understanding when she knows that the hormone _____ is synthesized and secreted by the kidneys to stimulate bone marrow production of red blood cells. a. Creatinine b. Aldosterone c. Erythropoietin d. Renin

ANS: C Erythropoietin stimulates the bone marrow to produce red blood cells in response to tissue hypoxia. Erythropoietin, not creatinine, stimulates the bone marrow to produce red blood cells in response to tissue hypoxia. Erythropoietin, not aldosterone, stimulates the bone marrow to produce red blood cells in response to tissue hypoxia. Erythropoietin, not renin, stimulates the bone marrow to produce red blood cells in response to tissue hypoxia. Cognitive Level: Comprehension 736

4. The most common disorder associated with upper GI bleeding is: a. Diverticulosis b. Hemorrhoids c. Esophageal varices d. Cancer

ANS: C Esophageal varices is the most common disorder associated with upper GI bleeding. Diverticulosis could lead to bleeding, but it would be lower GI rather than upper. Hemorrhoids can lead to bleeding, but they would be upper GI. Duodenal ulcers could lead to upper GI bleeding, but peptic ulcers and varices are identified as more common. Cognitive Level: Comprehension 897

27. _____ is the process by which an action potential in the plasma membrane of a myocardial cell triggers the events that directly cause contraction of the myocardial cells. a. Electrocontraction b. Intercalated communication c. Excitation-contraction coupling d. Myosin communication

ANS: C Excitation-contraction coupling is the process by which an action potential in the plasma membrane of the muscle fiber triggers the cycle, leading to cross-bridge activity and contraction. Excitation-contraction coupling, not electrocontraction, is the process by which an action potential in the plasma membrane of the muscle fiber triggers the cycle, leading to cross-bridge activity and contraction. Excitation-contraction coupling, not intercalated communication, is the process by which an action potential in the plasma membrane of the muscle fiber triggers the cycle, leading to cross-bridge activity and contraction. Excitation-contraction coupling, not myosin communication, is the process by which an action potential in the plasma membrane of the muscle fiber triggers the cycle, leading to cross-bridge activity and contraction. Cognitive Level: Comprehension 562

4. Exhaustion occurs if stress continues and _____ is not successful. a. Fight or flight b. Alarm c. Adaptation d. Arousal

ANS: C Exhaustion occurs if adaptation is not successful. Exhaustion occurs if adaptation is not successful, not fight or flight. Alarm is part of the adaptation process, but it is adaptation that prevents exhaustion. Arousal is the recognition of stress. Cognitive Level: Comprehension 205

3. A staff member asks a nurse what foam cells are. What is the nurses best response? Foam cells in a fatty streak are: a. Deposited adipose cells b. Injured neutrophils c. Macrophages that engulf low-density lipoprotein (LDL) d. Lipid-laden mast cells

ANS: C Foam cells are lipid-laden macrophages that engulf LDL. Foam cells are lipid-laden macrophages that engulf LDL. They are deposited in vessels, not adipose cells. Foam cells are lipid-laden macrophages that engulf LDL. They are not injured neutrophils. Foam cells are lipid-laden macrophages that engulf LDL. They are not mast cells. Cognitive Level: Comprehension 595

23. Which information indicates the nurse understands digestion? During the cephalic and gastric phases of digestion, gallbladder contraction is mediated by branches of the: a. Sympathetic nervous system b. Somatic nervous system c. Vagus nerve d. Glossopharyngeal nerve

ANS: C Gallbladder contraction is mediated by cholinergic branches of the vagus nerve. Gallbladder contraction is mediated by cholinergic branches of the vagus nerve, not the sympathetic nervous system. Gallbladder contraction is mediated by cholinergic branches of the vagus nerve, not the somatic nerves. Gallbladder contraction is mediated by cholinergic branches of the vagus nerve, not the glossopharyngeal nerve. Cognitive Level: Comprehension 887

6. A 50-year-old male is experiencing reflux of chyme from the stomach. He is diagnosed with gastroesophageal reflux. This condition is caused by: a. Fibrosis of the lower third of the esophagus b. Sympathetic nerve stimulation c. Loss of muscle tone at the lower esophageal sphincter d. Reverse peristalsis of the stomach

ANS: C Gastroesophageal reflux is due to loss of muscle tone at the lower esophageal sphincter. Gastroesophageal reflux is due to loss of muscle tone at the lower esophageal sphincter; it is not due to fibrosis. Gastroesophageal reflux is due to loss of muscle tone at the lower esophageal sphincter, not stimulation of sympathetic nerves. Gastroesophageal reflux is due to loss of muscle tone at the lower esophageal sphincter, not reverse peristalsis. Cognitive Level: Comprehension 898

5. A 52-year-old presents with bleeding from the rectum. This condition is referred to as: a. Melena b. Occult bleeding c. Hematochezia d. Hematemesis

ANS: C Hematochezia is bleeding from the rectum. Melena is a black or tarry stool. Occult bleeding is hidden bleeding. Hematemesis is vomiting blood. Cognitive Level: Comprehension 898

44. A 35-year-old male has enlarged lymph nodes in the neck and a mediastinal mass. He was diagnosed with Hodgkin lymphoma. Which of the following abnormal cells would the nurse expect to find with this disease? a. Merkel cell b. Schwann cell c. Reed-Sternberg cell d. Kupffer cell

ANS: C Hodgkin lymphoma is manifested by Reed-Sternberg cells. Hodgkin lymphoma is manifested by Reed-Sternberg cells, not Merkel cells. Hodgkin lymphoma is manifested by Reed-Sternberg cells, not Schwann cells. Hodgkin lymphoma is manifested by Reed-Sternberg cells, not Kupffer cells. Cognitive Level: Comprehension 516

1. A 25-year-old female is diagnosed with urinary tract obstruction. While planning care, the nurse realizes that the patient is expected to have hydronephrosis and a decreased glomerular filtration rate caused by: a. decreased renal blood flow. b. decreased peritubular capillary pressure. c. dilation of the renal pelvis and calyces proximal to a blockage. d. stimulation of antidiuretic hormone.

ANS: C Hydronephrosis occurs due to dilation of the renal pelvis and calyces proximal to a blockage. Hydronephrosis is not the result of decreased renal blood flow. Hydronephrosis is not the result of decreased peritubular capillary pressure, but due to dilation of the renal pelvis. Hydronephrosis occurs due to dilation of the renal pelvis and calyces proximal to a blockage, not to stimulation of the antidiuretic hormone. Cognitive Level: Comprehension 741

1. When a patient asks the nurse what hypersensitivity is, how should the nurse respond? Hypersensitivity is best defined as: a. A reduced immune response found in most pathologic states b. A normal immune response to an infectious agent c. An excessive or inappropriate response of the immune system to a sensitizing antigen d. Antigenic desensitization

ANS: C Hypersensitivity is an altered immunologic response to an antigen that results in disease or damage to the individual. Hypersensitivity is not a reduced immune response. Hypersensitivity is not a normal response to an infectious agent. Antigenic desensitization is performed to decrease hypersensitivity. Cognitive Level: Comprehension 165

48. For a patient experiencing hypersplenism, the nurse expects the erythrocytes to be: a. Proliferated b. Activated c. Sequestered d. Infected

ANS: C Hypersplenism results in sequestering of the blood cells. Hypersplenism results in sequestering, not proliferation, of the blood cells. Hypersplenism results in sequestering, not activation, of the blood cells. Hypersplenism results in sequestering, not infection, of the blood cells. Cognitive Level: Comprehension 521

27. While turning a patient with chronic renal failure, which principle should the nurse recall? Bone fractures are a risk factor in chronic renal failure because: a. Calcium is lost in the urine. b. Osteoblast activity is excessive. c. The kidneys fail to activate vitamin D. d. Autoantibodies to calcium molecules develop.

ANS: C Hypocalcemia is accelerated by impaired renal synthesis of 1,25-vitamin D. The combined effect of vitamin D deficiency can result in renal osteodystrophies with increased risk for fractures. Hypocalcemia is accelerated by impaired renal synthesis of 1,25-vitamin D. The combined effect of vitamin D deficiency can result in renal osteodystrophies with increased risk for fractures. Calcium is not lost in the urine. Hypocalcemia is accelerated by impaired renal synthesis of 1,25-vitamin D. The combined effect of vitamin D deficiency can result in renal osteodystrophies with increased risk for fractures. Osteoblast activity is not accelerated. Hypocalcemia is accelerated by impaired renal synthesis of 1,25-vitamin D. The combined effect of vitamin D deficiency can result in renal osteodystrophies with increased risk for fractures. Antibodies do not develop. Cognitive Level: Comprehension 760

13. A 10-year-old male is stung by a bee while playing in the yard. He experiences a severe allergic reaction and has to go to the ER. The nurse providing care realizes this reaction is the result of: a. Toxoids b. IgA c. IgE d. IgM

ANS: C IgE is normally at low concentrations in the circulation. It has very specialized functions as a mediator of many common allergic responses. IgE, not toxoids, is the mediator of common allergic response. IgE, not IgA, is the mediator of common allergic response. IgE, not IgM, is the mediator of common allergic response. Cognitive Level: Comprehension 151

9. A 23-year-old pregnant female visits her primary care provider for her final prenatal checkup. The primary care provider determines that the fetus has developed an infection in utero. Which of the following would be increased in the fetus at birth? a. IgG b. IgA c. IgM d. IgD

ANS: C IgM is synthesized early in neonatal life, and its synthesis may be increased as a response to infection in utero. IgM, not IgG, is synthesized early in neonatal life, and its synthesis may be increased as a response to infection in utero. IgM, not IgA, is synthesized early in neonatal life, and its synthesis may be increased as a response to infection in utero. IgM, not IgD, is synthesized early in neonatal life, and its synthesis may be increased as a response to infection in utero. Cognitive Level: Comprehension 149

13. A nurse recalls an example of an immune-complex-mediated disease is: a. Bronchial asthma b. Contact dermatitis c. Serum sickness d. Rheumatoid arthritis

ANS: C Immune-complex disease can be a systemic reaction, such as serum sickness and related to type III reactions. Bronchial asthma is not an immune-complex mediated disease and related to type I reactions. Contact dermatitis is related to type IV reactions. Rheumatoid arthritis is related to type IV reactions. Cognitive Level: Comprehension 193

35. Manifestations associated with hepatic encephalopathy from chronic liver disease are the result of: a. Hyperbilirubinemia and jaundice b. Fluid and electrolyte imbalances c. Impaired ammonia metabolism d. Decreased cerebral blood flow

ANS: C Impaired ammonia metabolism leads to the symptoms of hepatic encephalopathy. Impaired ammonia metabolism leads to the symptoms of hepatic encephalopathy. Symptoms are primarily neurologic, not jaundice oriented. Impaired ammonia metabolism leads to the symptoms of hepatic encephalopathy. Fluid and electrolyte changes that occur would not lead to the manifestations seen in hepatic encephalopathy. Impaired ammonia metabolism leads to the symptoms of hepatic encephalopathy. Cerebral blood flow is not affected. Cognitive Level: Comprehension 917

35. A 15-year-old female presents with splenomegaly, hepatomegaly, and lymph node enlargement. She is diagnosed with infectious mononucleosis. What should the nurse tell the patient about the recovery time? a. 72 hours b. 3 to 5 days c. A few weeks d. Six months

ANS: C Infectious mononucleosis (IM) is usually self-limiting, and recovery occurs in a few weeks; severe clinical complications are rare (5%). Fatigue may last for 1 to 2 months after resolution of other symptoms. Seventy-two hours is too short; the disorder may last a few weeks. Three to five days is too short; the disorder may last a few weeks. Six months is too long; the disorder may resolve in a few weeks. Cognitive Level: Comprehension 511

20. The nurse will check which of the following tests to directly measure iron stores? a. Serum ferritin b. Transferrin saturation c. Bone marrow biopsy d. Total iron-binding capacity

ANS: C Iron stores are measured directly by bone marrow biopsy. Iron stores are measured directly by bone marrow biopsy, not serum ferritin. Iron stores are measured directly by bone marrow biopsy, not transferrin saturation. Iron stores are measured directly by bone marrow biopsy, not total iron-binding capacity. Cognitive Level: Comprehension 504

20. A macrophage was isolated and analyzed for major histocompatibility complex. Which of the following would be expected? a. MHC I only b. MHC II only c. MHC I and II d. Neither MHC I or MHC II

ANS: C MCH I and II would be expected. MCH I and II would be expected. MCH I and II would be expected. Both MCH I and II would be expected. Cognitive Level: Comprehension 157

2. Which information indicates a nurse understands characteristics of malignant tumors? a. Grows slowly b. Has a well-defined capsule c. Has a high mitotic index d. Is well-differentiated

ANS: C Malignant tumors have a high mitotic index. Malignant tumors grow rapidly. Malignant tumors are not encapsulated. Malignant tumors are poorly-differentiated. Cognitive Level: Comprehension 223

4. A urologist is discussing the phagocytic cells that lie between the layers of the renal corpuscle. What is the urologist describing? a. Podocytes b. Macula densa cells c. Mesangial cells d. Filtration slits

ANS: C Mesangial cells lie between and support the capillaries. Mesangial cells have phagocytic ability similar to monocytes, release inflammatory cytokines, and can contract to regulate glomerular capillary blood flow. The epithelium has specialized cells called podocytes from which pedicles (foot projections) radiate and adhere to the basement membrane. The macula densa are sodium-sensing cells. The pedicles interlock with the pedicles of adjacent podocytes, forming an elaborate network of intercellular clefts called filtration slits, or slit membranes. Cognitive Level: Comprehension 726

16. A patient has done research on monoclonal antibodies on the Internet. Which statement indicates a correct understanding? Pure monoclonal antibodies are produced by: a. T lymphocytes b. Bone marrow c. Laboratories d. Fetuses

ANS: C Monoclonal antibodies are produced in the laboratory from one B cell that has been cloned; thus, the entire antibody is of the same class, specificity, and function. Monoclonal antibodies are produced in the laboratory, not by T lymphocytes. Monoclonal antibodies are produced in the laboratory, not by the bone marrow. Monoclonal antibodies are produced in the laboratory, not by the fetus.

8. A 35-year-old male is diagnosed with a hormone-secreting tumor of the adrenal medulla. He experiences elevated blood pressure, pupil dilation, goose bumps, and increased anxiety. Which of the following hormones is the predominant one released by the tumor? a. Antidiuretic hormone b. Acetylcholine c. Norepinephrine d. Cortisol

ANS: C Norepinephrine regulates blood pressure and promotes arousal, increased vigilance, increased anxiety, and other protective emotional responses. Antidiuretic hormone regulates urine output. Acetylcholine will not result in an increase in blood pressure. Cortisol regulates glucocorticoids. Cognitive Level: Comprehension 209

23. A 5-month-old child is admitted to the hospital with recurring respiratory infections. A possible cause of this condition is: a. Hypergammaglobulinemia b. Increased maternal IgG c. Immune insufficiency d. Decreased maternal antibody breakdown, resulting in hyposensitivity

ANS: C Normal human infants are immunologically immature when born, with deficiencies in antibody production, phagocytic activity, and complement activity, especially components of alternative pathways. Normal human infants are immunologically immature when born. They do not posses hypergammaglobulinemia. Possessing increased maternal IgG would not lead to recurring infections. The recurrent infections are due to decreased immunity, not maternal antibody breakdown. Cognitive Level: Comprehension 164

23. A 30-year-old obese female underwent gastric resection in an attempt to lose weight. Which of the following complications could the surgery cause? a. Constipation b. Acid reflux gastritis c. Anemia d. Hiccups

ANS: C One of the complications is anemia due to iron malabsorption, which may result from decreased acid secretion. Diarrhea, not constipation, occurs. The reflux would be alkaline, not acidic. Hiccups are not associated with gastrectomy. Cognitive Level: Comprehension 907

6. When an aide asks the nurse what is a purpose of the inflammatory process, how should the nurse respond? a. To provide specific responses toward antigens b. To lyse cell membranes of microorganisms c. To prevent infection of the injured tissue d. To create immunity against subsequent tissue injury

ANS: C One purpose of the inflammatory process is to prevent infection and further damage by contaminating microorganisms. Specific response toward antigens is a part of the complement system that assists in the inflammatory response, but not its purpose. Lysis of cell membranes is part of the process of phagocytosis, which removes foreign material, but this is not the purpose of the inflammatory response. Immunity cannot be achieved against future tissue injury. Cognitive Level: Comprehension 121

9. Which of the following individuals would be at greatest risk for an opportunistic infection? a. 18-year-old with diabetes b. 70-year-old with congestive heart failure c. 24-year-old who is immunocompromised d. 30-year-old with pneumonia

ANS: C Opportunistic microorganisms can cause disease if the individuals defenses are compromised. An 18-year-old with diabetes would not be immunocompromised and would not be at risk. A 70-year-old with congestive heart failure would not be immunocompromised and would not be at risk. A 30-year-old with pneumonia would not be immunocompromised and would not be at risk. Cognitive Level: Comprehension 120

11. When a nurse checks the patient for orthostatic hypotension, what did the nurse have the patient do? a. Physical exertion b. Eat c. Stand up d. Lie down

ANS: C Orthostatic hypotension refers to a drop in blood pressure when standing up. Orthostatic hypotension refers to a drop in blood pressure when standing up, not a drop with exertion. Orthostatic hypotension refers to a drop in blood pressure when standing up, not a drop with eating. Orthostatic hypotension refers to a drop in blood pressure when standing up, not with lying down. Cognitive Level: Comprehension 591

15. The role of physical activity in the prevention of colon cancer is identified by which of the following? a. It increases fluid loss leading to thirst and increased fluid intake, hydrating the colon. b. It increases blood supply thereby increasing oxygen to the colon. c. It increases gut motility thereby decreasing the time the bowel is exposed to mutagens. d. It increases the secretion of hydrochloric acid thereby killing mutants.

ANS: C Physical activity increases gut motility thereby decreasing exposure to mutagens. Physical activity increases fluid loss, but this does not prevent mutagens. Physical activity increases blood supply to the colon, but this does not prevent mutagens. Physical activity does not increase hydrochloric acid. Cognitive Level: Comprehension 278

25. A nurse is discussing fibrinolysis. Which information should the nurse share? Fibrinolysis is mediated by: a. Heparin b. Fibrinogen c. Plasmin d. Albumin

ANS: C Plasmin is an enzyme that promotes fibrinolysis by degrading fibrin and fibrinogen into fibrin degradation products. Plasmin mediates fibrinolysis, not heparin. Plasmin mediates fibrinolysis, not fibrinogen. Plasmin mediates fibrinolysis, not albumin. Cognitive Level: Comprehension 496

10. Which portion of the antibody is responsible for the biologic functions of antibodies? a. Heavy chain b. Variable region c. Fc portion d. Epitope

ANS: C The Fc portion is responsible for most of the functions of antibodies. The Fc portion is responsible for most of the functions of antibodies, not the heavy chain. The Fc portion is responsible for most of the functions of antibodies, not the variable region. The Fc portion is responsible for most of the functions of antibodies, not the epitope. Cognitive Level: Comprehension 149

21. When a staff member asks which of the following substances are actively secreted by the renal tubules, what is the nurses best response? a. Sodium and chlorine b. Phosphate and calcium c. Hydrogen and potassium d. Bicarbonate and carbonic acid

ANS: C Principal cells reabsorb sodium and secrete potassium, and intercalated cells reabsorb potassium and bicarbonate and secrete hydrogen. Hydrogen and potassium are excreted, not sodium and chlorine. Hydrogen and potassium are excreted, not phosphate and calcium. Hydrogen and potassium are excreted, not bicarbonate and carbonic acid. Cognitive Level: Comprehension 734

8. Mucus production to form the mucosal barrier in the stomach is stimulated by the release of: a. Somatostatin b. Gastrin c. Prostaglandins d. Histamine

ANS: C Prostaglandins protect the mucosal barrier by stimulating the secretion of mucus and bicarbonate and by inhibiting secretion of acid. Somatostatin is secreted by the pancreas and is not involved in secretion of mucus. Gastrin is not involved in mucus secretion. Histamine initiates secretion of acid, not mucus. Cognitive Level: Comprehension 877

12. After birth, red blood cells are normally made only in the: a. Liver b. Spleen c. Bone marrow d. Kidney

ANS: C Red blood cells are produced in the bone marrow. Red blood cells are produced in the bone marrow, not the liver. Red blood cells are produced in the bone marrow, not the spleen. Red blood bells are produced in the bone marrow, not the kidney. Cognitive Level: Comprehension 485

31. A 12-year-old male is fighting with another child when he receives a puncture wound from a pencil. The school nurse cleans and bandages the wound. After about 1 week, the wound would be in which phase of healing? a. Debridement b. Primary intention c. Resolution d. Maturation

ANS: C Resolution occurs when repaired tissue is approaching close to normal. Debridement is the scraping away of dead tissue and is not a phase of wound healing. Primary intention is the stage of healing of wounds that are closely proximated. Maturation is the result of severe wounds which would begin several weeks after injury and may take 2 years. Cognitive Level: Comprehension 134

32. Which finding would support the diagnosis of respiratory acidosis? a. Vomiting b. Hyperventilation c. Pneumonia d. An increase in noncarbonic acids

ANS: C Respiratory acidosis occurs with hypoventilation, and pneumonia leads to hypoventilation. Vomiting leads to loss of acids and then to alkalosis. Hyperventilation leads to respiratory alkalosis, not acidosis. Metabolic acidosis is caused by an increase in noncarbonic acids. Cognitive Level: Comprehension 114

22. A 50-year-old male with a 30-year history of smoking was diagnosed with bronchogenic cancer. He developed edema and venous distention in the upper extremities and face. Which of the following diagnosis will the nurse observe on the chart? a. Thromboembolism b. Deep vein thrombosis c. Superior vena cava syndrome (SVCS) d. Chronic venous insufficiency

ANS: C SVCS is a progressive occlusion of the superior vena cava that leads to venous distention in the upper extremities and head. Thromboembolism would not lead to the generalized symptoms described in the patient. Deep vein thrombosis would not lead to upper extremity symptoms. Chronic venous insufficiency would primarily affect one extremity. Cognitive Level: Comprehension 586

12. Of the following genetic lesions that cause cancer, which is the most common? a. Insertions b. Deletions c. Point mutations d. Amplification

ANS: C Several types of genetic events can activate oncogenes. The most common are small scale changes in DNA called point mutations. Several types of genetic events can activate oncogenes. The most common are small scale changes in DNA called point mutations, not insertions. Several types of genetic events can activate oncogenes. The most common are small scale changes in DNA called point mutations, not deletions. Several types of genetic events can activate oncogenes. The most common are small scale changes in DNA called point mutations, not amplification. Cognitive Level: Comprehension 229

19. The capillaries of the liver are more commonly known as hepatic: a. Canaliculi b. Ducts c. Sinusoids d. Papillae

ANS: C Small capillaries, or sinusoids, are located between the plates of hepatocytes of the liver. Small capillaries, or sinusoids, not canaliculi, are located between the plates of hepatocytes of the liver. Small capillaries, or sinusoids, not ducts, are located between the plates of hepatocytes of the liver. Small capillaries, or sinusoids, not papillae are located between the plates of hepatocytes of the liver. Cognitive Level: Comprehension 884

14. A 13-year-old female took a weight loss drug that activated the sympathetic nervous system. Which of the following assessment findings would the nurse expect? a. Decreased myocardial contraction b. Decreased heart rate c. Increased cardiac conduction d. Increased intranodal conduction time

ANS: C Stimulation of the SA node by the sympathetic nervous system rapidly increases cardiac conduction. Stimulation of the sympathetic nervous system would increase myocardial contraction. Stimulation of the sympathetic nervous system would increase heart rate. Stimulation of the sympathetic nervous system would not affect conduction time within the node. These are inherent rates. Cognitive Level: Comprehension 560

28. A 50-year-old female experiences decreased blood pressure, decreased oxygen delivery, cardiovascular shock, and subsequent death. A complication of endotoxic shock is suspected. Which of the following is the most likely cause? a. Gram-positive bacteria b. Fungi c. Gram-negative bacteria d. Virus

ANS: C Symptoms of gram-negative septic shock are produced by endotoxins. Once in the blood, endotoxins cause the release of vasoactive peptides and cytokines that affect blood vessels, producing vasodilation, which reduces blood pressure, causes decreased oxygen delivery, and produces subsequent cardiovascular shock. Gram-positive bacteria typically do not produce endotoxins and thus do not manifest in shock. Fungi do not produce the endotoxic shock symptoms described. Viruses do not produce symptoms of shock. Cognitive Level: Comprehension 171

19. After a patient is exposed to a specific antigen, B cells will differentiate into: a. B cytotoxic cells b. Clonal equivalents c. Plasma cells d. Bursal cells

ANS: C The B cell differentiates into plasma cells. The B cell differentiates into plasma cells, not B cytotoxic cells. The B cell differentiates into plasma cells, not clonal equivalents. The B cell differentiates into plasma cells, not bursal cells. Cognitive Level: Comprehension 158

16. The ________ sphincter controls the movement of chyme from the sigmoid colon into the rectum? a. Oddi b. Ileocecal c. OBeirne d. Internal anal

ANS: C The OBeirne sphincter controls the movement of wastes from the sigmoid colon into the rectum. The OBeirne sphincter, not the Oddi sphincter, controls the movement of wastes from the sigmoid colon into the rectum. The OBeirne sphincter, not the ileocecal sphincter, controls the movement of wastes from the sigmoid colon into the rectum. The OBeirne sphincter, not the internal anal sphincter, controls the movement of wastes from the sigmoid colon into the rectum. Cognitive Level: Comprehension 881

19. The nurse is reviewing a normal electrocardiogram. The nurse assesses the PR interval because it represents: a. Atrial depolarization b. Ventricular depolarization c. Onset of atrial activation to onset of ventricular activity d. Electrical systole of the ventricles

ANS: C The PR interval represents the onset of atrial activation to the onset of ventricular activity. The P wave represents atrial depolarization. The QRS complex represents ventricular depolarization. The QT interval represents electrical systole of the ventricles. Cognitive Level: Comprehension 559

20. A 5-year-old male is diagnosed with a bacterial infection. Cultures of the bacteria revealed lipopolysaccharides on the bacterial cell surface. Which of the complement pathways would be activated in this case? a. Classical pathway b. Lectin pathway c. Alternative pathway d. Kinin pathway

ANS: C The alternative pathway is activated by several substances found on the surface of infectious organisms, such as those containing lipopolysaccharides. The classical pathway is primarily activated by antibodies that are proteins of the acquired immune system. The lectin pathway is similar to the classic pathway but is independent of antibody. It is activated by several plasma proteins. The kinin pathway is involved in coagulation. Cognitive Level: Comprehension 124

11. An experiment is carried out to characterize newly discovered immunoglobulins. Specificity of the amino acids within the immunoglobulin could be determined by studying the: a. C region of the light chain b. C region of the H chain c. Complementary-determining region (CDR) d. Framework regions

ANS: C The amino acid differences are clustered into three areas in the variable region. These three areas are called CDRs. The amino acid differences are clustered into three areas in the variable region. These three areas are called CDRs, not the C region of the light chain. The amino acid differences are clustered into three areas in the variable region. These three areas are called CDRs, not the C region of the H chain. The amino acid differences are clustered into three areas in the variable region. These three areas are called CDRs, not the framework regions. Cognitive Level: Comprehension 149

11. The cardinal sign of pyloric stenosis caused by ulceration or tumors is: a. Constipation b. Diarrhea c. Vomiting d. Heartburn

ANS: C The cardinal sign of pyloric stenosis is vomiting. The cardinal sign of pyloric stenosis is vomiting, not constipation. The cardinal sign of pyloric stenosis is vomiting, not diarrhea. The cardinal sign of pyloric stenosis is vomiting, not heartburn. Cognitive Level: Comprehension 900

9. Stress induces sympathetic stimulation of the adrenal medulla. This causes the secretion of catecholamines, which include: a. Epinephrine and aldosterone b. Norepinephrine and cortisol c. Epinephrine and norepinephrine d. Cortisol and aldosterone

ANS: C The catecholamines are epinephrine and norepinephrine. The catecholamines are epinephrine and norepinephrine, not aldosterone. The catecholamines are epinephrine and norepinephrine, not cortisol. The catecholamines are epinephrine and norepinephrine, not cortisol and aldosterone. Cognitive Level: Comprehension 209

49. Thrombocytopenia may be: a. Transient or consistent b. Normal or abnormal c. Congenital or acquired d. Active or inactive

ANS: C The condition may also be either congenital or acquired and may be either primary or secondary to other conditions. The condition may also be either congenital or acquired and may be either primary or secondary to other conditions. It is not referred to as transient or consistent. The condition may also be either congenital or acquired and may be either primary or secondary to other conditions. It is not normal or abnormal. The condition may also be either congenital or acquired and may be either primary or secondary to other conditions. It is not active or inactive. Cognitive Level: Comprehension 523

25. The exocrine portion of the pancreas contains: a. Alpha cells b. Beta cells c. Ducts d. Islets of Langerhans

ANS: C The exocrine portion of the pancreas contains ducts. The endocrine portion contains the alpha cells. The endocrine portion contains the beta cells. The endocrine portion contains the islets of Langerhans. Cognitive Level: Comprehension 888

12. On average the kidneys receive approximately _____ of the cardiac output. a. 10% to 14% b. 15% to 19% c. 20% to 25% d. 26% to 35%

ANS: C The kidney receives 20% to 25% of the cardiac output. The kidney receives 20% to 25%, not 10% to 14% of the cardiac output. The kidney receives 20% to 25%, not 15% to 19% of the cardiac output. The kidney receives 20% to 25%, not 26% to 35% of the cardiac output. Cognitive Level: Comprehension 729

10. The nurse is planning care for a patient with heart problems. Which information should the nurse remember? The _____ artery travels down the interventricular septum and delivers blood to portions of the left and right ventricle. a. Right coronary b. Circumflex c. Left anterior descending d. Cardiac

ANS: C The left anterior descending artery delivers blood to the left and right ventricle. The left anterior descending artery, not the right coronary artery, delivers blood to the left and right ventricle. The left anterior descending artery, not the circumflex, delivers blood to the left and right ventricle. The left anterior descending artery, not the cardiac artery, delivers blood to the left and right ventricle. Cognitive Level: Comprehension 557

4. A nurse is teaching about the heart. Which information should the nurse include? The chamber of the heart that generates the highest pressure is the: a. Right atrium b. Left atrium c. Left ventricle d. Right ventricle

ANS: C The left ventricle generates the highest pressure. The left ventricle, not the right atrium, generates the highest pressure. The left ventricle, not the left atrium, generates the highest pressure. The left ventricle, not the right ventricle, generates the highest pressure. Cognitive Level: Comprehension 553

50. A 30-year-old female presents with hematuria, menorrhagia, and bleeding gums. She is diagnosed with immune thrombocytic purpura (ITP). A nurse realizes the most likely cause is: a. Allergy-induced platelet lysis b. An immune response to hypersplenism c. Antibody destruction of platelets d. T cell injury to megakaryocytes

ANS: C The majority of cases of ITP are due to immune driven destruction of platelets. The majority of cases of ITP are due to immune driven destruction of platelets. It is not due to allergy. The majority of cases of ITP are due to immune driven destruction of platelets. It is not due to hypersplenism. The majority of cases of ITP are due to immune driven destruction of platelets. It is not due to T cell injury. Cognitive Level: Comprehension 524

39. A 55-year-old male died in a motor vehicle accident. Autopsy revealed an enlarged liver caused by fatty infiltration, testicular atrophy, and mild jaundice secondary to cirrhosis. The most likely cause of his condition is: a. Bacterial infection b. Viral infection c. Alcoholism d. Drug overdose

ANS: C The most common cause of cirrhosis is alcoholism. The most common cause of cirrhosis is alcoholism, not a bacterial infection. The most common cause of cirrhosis is alcoholism, not a viral infection. The most common cause of cirrhosis is alcoholism, not drug overdose. Cognitive Level: Comprehension 921

26. When a patient asks the nurse what is the most common cause of myocardial ischemia, which statement is the correct response? The most common cause of myocardial ischemia is: a. Idiopathic vasospasm b. Arterial emboli from heart valve c. Atherosclerosis d. Venous emboli

ANS: C The most common cause of myocardial ischemia is atherosclerosis. Atherosclerosis, not vasospasm, is the major cause of myocardial ischemia. Arterial emboli may cause ischemia, but atherosclerosis is the major cause of myocardial ischemia. Venous emboli would not lead to myocardial ischemia; atherosclerosis is the major cause of myocardial ischemia. Cognitive Level: Comprehension 600

4. When a patient asks what the most common type of renal stones is composed of, how should the nurse respond? The most common type of renal stone is composed of: a. magnesium. b. struvite. c. calcium. d. phosphate.

ANS: C The most common composition of a renal stone is calcium, accounting for 70% to 80%. The most common composition of a renal stone is calcium, accounting for 70% to 80%. Only a limited number of stones are composed of magnesium. The most common composition of a renal stone is calcium, accounting for 70% to 80%. Struvite accounts for less than 10% to 15%. The most common composition of a renal stone is calcium, accounting for 70% to 80%. The percentage of phosphate is very low. Cognitive Level: Comprehension 743

16. Which statement indicates the patient has a correct understanding of metastasis? The most common route of metastasis is through the blood vessels and: a. Lung tissue b. Body cavities c. Lymphatics d. Connective tissues

ANS: C The most common route of metastasis is through the lymphatics. The most common route of metastasis is through the lymphatics, not lung tissue. The most common route of metastasis is through the lymphatics, not the body cavity. The most common route of metastasis if through the lymphatics, not connective tissues. Cognitive Level: Comprehension 242

17. A 28-year-old female presents with severe chest pain and shortness of breath. She is diagnosed with pulmonary embolism, which most likely originated from the: a. Left ventricle b. Systemic arteries c. Deep veins of the leg d. Superficial veins of the arm

ANS: C The most likely origin of the embolism is from the deep veins of the legs. The most likely origin of the embolism is from the deep veins of the legs, not from the left ventricle. The most likely origin of the embolism is from the deep veins of the legs, not the systemic arteries. The most likely origin of the embolism is from the deep veins of the legs, not from the arms. Cognitive Level: Comprehension 593

7. When a patient presents at the emergency department for an allergic reaction, the nurse recognizes the most severe consequence of a type I hypersensitivity reaction is: a. Urticaria b. Hives c. Anaphylaxis d. Antibody-dependent cell-mediated cytotoxicity (ADCC)

ANS: C The most rapid and severe immediate hypersensitivity type I reaction is anaphylaxis. Urticaria, or hives, is a dermal (skin) manifestation of allergic reactions. Hives and urticaria are similar responses. ADCC is a mechanism that involves natural killer (NK) cells. Antibodies on the target cell are recognized by Fc receptors on the NK cells, which release toxic substances that destroy the target cell. Cognitive Level: Comprehension 190

9. A 55-year-old female has undiagnosed hypertension. She presents to her primary care provider reporting impaired vision and chronic edema. Lab tests reveal that she also has renal insufficiency. While planning care, the nurse realizes the most likely cause for these findings is: a. Clotting and gangrene b. Free radical injury and cell toxicity c. End-organ damage d. Hypertrophy and hyperplasia

ANS: C The patient is experiencing end-organ damage, as indicated by renal insufficiency. The patient is not experiencing gangrene, but end-organ damage. The patient is experiencing end-organ damage; it is not due to the formation of free radicals. Hypertrophy and hyperplasia could develop, but the cause of the patients symptoms is end-organ damage. Cognitive Level: Comprehension 590

1. A 52-year-old female is admitted to the cardiac unit with a diagnosis of pericarditis. She asks the nurse to explain where the infection is. In providing an accurate description, the nurse states that the pericardium is: a. The outer muscular layer of the heart b. The innermost layer of the heart chambers c. A membranous sac that encloses the heart d. The hearts fibrous skeleton

ANS: C The pericardium is the membranous sac that surrounds the heart. The outer layer of the heart is the myocardium. The innermost layer of the heart is the endocardium. The hearts fibrous skeleton is attached to the myocardium. Cognitive Level: Comprehension 552

13. A 45-year-old female was recently diagnosed with cervical cancer. She reports a sexual history that includes 43 partners. Which of the following is the most likely cause of her cancer? a. Herpes virus b. Rubella virus c. Human papillomavirus (HPV) d. Hepatitis B virus

ANS: C The presence of HPV is a factor in cervical cancer. The presence of HPV is a factor in cervical cancer, not herpes. The presence of HPV is a factor in cervical cancer, not rubella. The presence of HPV is a factor in cervical cancer, not hepatitis B. Cognitive Level: Comprehension 241

10. A 38-year-old female complains of epigastric fullness following a meal, nausea, and epigastric pain. Tests reveal narrowing of the opening between the stomach and the duodenum. This condition is referred to as: a. Ileocecal obstruction b. Hiatal hernia c. Pyloric obstruction d. Hiatal obstruction

ANS: C The pylorus is the opening between the esophagus and the duodenum; the obstruction is pyloric. Ileocecal obstruction is in the small intestine. Hiatal hernia is related to the esophagus. Hiatal obstruction is related to the esophagus. Cognitive Level: Comprehension 900

15. While planning care for a patient with urinary problems, the nurse recalls that the renin- angiotensin system will be activated by: a. Increased blood volume b. Elevated sodium concentrations c. Decreased blood pressure in the afferent arterioles d. Renal hypertension

ANS: C The renin-angiotensin system is activated by decreased blood pressure. Decreased blood pressure, not increased volume, leads to activation of the renin-angiotensin system. Decreased blood pressure, not elevated sodium, leads to activation of the renin-angiotensin system. Decreased blood pressure, not hypertension, leads to activation of the renin-angiotensin system. Cognitive Level: Comprehension 730

5. A patient presents with poison ivy on the extremities, face, and buttocks. This condition is an example of: a. Anaphylaxis b. Serum sickness c. Delayed hypersensitivity d. Viral disease

ANS: C The response to poison ivy is a delayed hypersensitivity because it takes up 72 hours to develop. Anaphylaxis is immediate. Serum sickness-type reactions are caused by the formation of immune complexes in the blood and their subsequent generalized deposition in target tissues. Poison ivy is not a viral disease. Cognitive Level: Comprehension 190

29. A 68-year-old female is admitted to the emergency department with a diagnosis of polycythemia vera (PV). A nurse realizes the patients symptoms are mainly the result of: a. A decreased erythrocyte count b. Rapid blood flow to the major organs c. Increased blood viscosity d. Vessel injury

ANS: C The symptoms of PV are due to increased blood viscosity. The symptoms of PV are due to increased blood viscosity, and the erythrocyte count is elevated. The symptoms of PV are due to increased blood viscosity, and there will be decreased blood flow to tissues and organs. The symptoms of PV are due to increased blood viscosity; vessel injury does not occur. Cognitive Level: Comprehension 507

10. A nurse is describing the trigone. Which information should be included? The trigone is defined as: a. The orifice of the ureter b. The inner area of the kidney c. A triangular area between the openings of the two ureters and the urethra d. The three divisions of the loop of Henle

ANS: C The trigone is a smooth triangular area between the openings of the two ureters and the urethra. The trigone is a smooth triangular area between the openings of the two ureters and the urethra; it is not the orifice of the ureter. The trigone is a smooth triangular area between the openings of the two ureters and the urethra; it is not the inner area of the kidney. The trigone is a smooth triangular area between the openings of the two ureters and the urethra; it is not a part of the loop of Henle. Cognitive Level: Comprehension 729

12. A 35-year-old female is diagnosed with vitamin B12 deficiency anemia (pernicious anemia). How should the nurse respond when the patient asks what causes pernicious anemia? A decrease in ______ is the most likely cause. a. Ferritin b. Gastric enzymes c. Intrinsic factor d. Erythropoietin

ANS: C The underlying alteration in pernicious anemia (PA) is the absence of intrinsic factor (IF), an enzyme required for gastric absorption of dietary vitamin B12, a vitamin essential for nuclear maturation and DNA synthesis in red blood cells. PA is not due to a decrease in ferritin, but a lack of the intrinsic factor. PA is not due to a decrease in gastric enzymes, but a lack of the intrinsic factor. PA is not due to a decrease in erythropoietin, but a lack of intrinsic factor. Cognitive Level: Comprehension 501

15. The vermiform appendix is attached to the: a. Duodenum b. Ileum c. Cecum d. Sigmoid colon

ANS: C The vermiform appendix is attached to the cecum. The vermiform appendix is attached to the cecum, not the duodenum. The vermiform appendix is attached to the cecum, not the ileum. The vermiform appendix is attached to the cecum, not the sigmoid. Cognitive Level: Comprehension 881

23. A newborn baby is diagnosed with a blood disorder in which her platelet count is low. Which of the following does the nurse suspect could be the reason? a. Increased megakaryocytes b. Increased platelet cell division c. Decreased thrombopoietin d. Decreased spleen function

ANS: C Thrombopoietin (TPO), a hormone growth factor, is the main regulator of the circulating platelet mass, thus, a deficiency of TPO can lead to decreased platelets. TPO is the factor necessary for platelet production; megakaryocytes are platelet precursors, so if they are elevated, the platelets would be as well. Increased cell division would lead to increased platelets. Platelets are produced in the bone marrow, not the spleen. REF: pp. 481-482

28. The molecule that aids in bonding of the troponin complex to actin and tropomyosin is troponin: a. C b. I c. T d. M

ANS: C Troponin T aids in the binding of the troponin complex to actin and tropomyosin. Troponin C contains binding sites for the calcium ions involved in contraction. Troponin I inhibits the ATPase of actomyosin. Troponin M is not a substance. Cognitive Level: Comprehension 563

7. A primary care provider is attempting to diagnose cancer and is looking for a tumor marker. Which of the following could be a possible marker? a. Red blood cells b. Apoptotic cells c. Enzymes d. Neurotransmitters

ANS: C Tumor markers include hormones, enzymes, genes, antigens, and antibodies. Tumor markers include hormones, enzymes, genes, antigens, and antibodies, but not red blood cells. Tumor markers include hormones, enzymes, genes, antigens, and antibodies, but not apoptotic cells. Tumor markers include hormones, enzymes, genes, antigens, and antibodies, but not neurotransmitters. Cognitive Level: Comprehension 225

24. A 65-year-old female is diagnosed with metastatic breast cancer. She has developed muscle wasting. Which of the following substances would be produced in large quantities to eliminate the tumor cells and cause muscle wasting? a. Interleukin-6 b. Eosinophils c. Tumor necrosis factor d. Platelets

ANS: C Tumor necrosis factor causes muscle wasting. Interleukin-6 stimulates growth and differentiation of blood cells. Eosinophils are stimulated for parasites. Platelets stimulate clotting. Cognitive Level: Comprehension 127

18. A 30-year-old male is demonstrating hematuria with red blood cell casts and proteinuria exceeding 3 to 5 g/day, with albumin being the major protein. The most probable diagnosis the nurse will see documented on the chart is: a. Cystitis b. Chronic pyelonephritis c. Acute glomerulonephritis d. Renal calculi

ANS: C Two major symptoms distinctive of more severe glomerulonephritis are: (1) hematuria with red blood cell casts and (2) proteinuria exceeding 3 to 5 g/day with albumin (macroalbuminuria) as the major protein. Cystitis is not a symptom of glomerulonephritis. Chronic pyelonephritis is manifested by dysuria, not proteinuria. Proteinuria is not seen with renal calculi. Cognitive Level: Comprehension 751

12. When a staff member asks where venous blood from the coronary circulation drains into, what is the best response by the nurse? The: a. Superior vena cava b. Inferior vena cava c. Right atrium d. Reft atrium

ANS: C Unoxygenated blood from the coronary veins enters the right atrium through the coronary sinus. Unoxygenated blood from the coronary veins enters the right atrium through the coronary sinus. It does not flow into the superior vena cava. Unoxygenated blood from the coronary veins enters the right atrium through the coronary sinus. It does not flow into the inferior vena cava. Unoxygenated blood from the coronary veins enters the right atrium, not the left, through the coronary sinus. Cognitive Level: Comprehension 557

13. While planning care for a patient who has acute pyelonephritis. A nurse recalls the most common condition associated with the development of acute pyelonephritis is: a. Cystitis b. Renal cancer c. Urinary tract obstruction d. Nephrotic syndrome

ANS: C Urinary obstruction and reflux of urine from the bladder are the most common underlying risk factors. Urinary obstruction and reflux of urine from the bladder are the most common underlying risk factors. Cystitis is not a common risk factor. Urinary obstruction and reflux of urine from the bladder are the most common underlying risk factors. Renal cancer is not a common risk factor. Urinary obstruction and reflux of urine from the bladder are the most common underlying risk factors. Nephrotic syndrome is not a common risk factor. Cognitive Level: Comprehension 749

17. A 60-year-old male presents with GI bleeding and abdominal pain. He reports that he takes NSAIDs daily to prevent heart attack. Tests reveal that he has a peptic ulcer. The most likely cause of this disease is: a. Increasing subepithelial bicarbonate production b. Accelerating the H+ (proton) pump in parietal cells c. Inhibiting mucosal prostaglandin synthesis d. Stimulating a shunt of mucosal blood flow

ANS: C Use of NSAIDs inhibit prostaglandins and maintenance of the mucosal barrier and decrease bicarbonate secretion. NSAIDs decrease bicarbonate production. H. pylori increases hydrogen secretion. NSAIDs do not affect mucosal blood flow. Cognitive Level: Comprehension 904

35. Which information indicates a good understanding of bacterial vaccines? Most bacterial vaccines contain: a. Active bacteria b. Synthetic bacteria c. Dead bacteria d. Bacterial toxins

ANS: C Vaccines are biological preparations of weakened or dead pathogens that when administered stimulate production of antibodies or cellular immunity against the pathogen without causing disease. Vaccines are biological preparations of weakened or dead pathogens, not active bacteria. Vaccines are biological preparations of weakened or dead pathogens, not synthetic bacteria. Vaccines are biological preparations of weakened or dead pathogens, not bacterial toxins. Cognitive Level: Comprehension 178

29. After studying about viruses, which information indicates the student has a good understanding of viruses? Viruses: a. Contain no DNA or RNA b. Are capable of independent reproduction c. Replicate their genetic material inside host cells d. Are easily killed by antimicrobials

ANS: C Virus replication depends totally on their ability to infect a permissive host cell, a cell that cannot resist viral invasion and replication. Viruses contain both DNA and RNA. Viruses are incapable of independent reproduction. Viruses cannot be killed by antimicrobials. Cognitive Level: Comprehension 171

33. The most common clinical manifestation of portal hypertension is _____ bleeding. a. Rectal b. Duodenal c. Esophageal d. Intestinal

ANS: C Vomiting of blood from bleeding esophageal varices is the most common clinical manifestation of portal hypertension. Esophageal bleeding, not rectal bleeding, is the most common clinical manifestation of portal hypertension. Esophageal bleeding, not duodenal bleeding, is the most common clinical manifestation of portal hypertension. Esophageal bleeding, not intestinal bleeding, is the most common clinical manifestation of portal hypertension. Cognitive Level: Comprehension 915

14. A 65-year-old male experienced loss of appetite, weight loss, lemon-yellow skin, liver enlargement, and a beefy red tongue shortly before his death. Autopsy suggested pernicious anemia, and the cause of death would most likely reveal: a. Brain hypoxia b. Liver hypoxia c. Heart failure d. Kidney failure

ANS: C When the hemoglobin has decreased to 7 to 8 g/dL, the individual experiences the classic symptoms of anemia: weakness, fatigue, paresthesias of feet and fingers, difficulty walking, loss of appetite, abdominal pain, weight loss, and a sore tongue that is smooth and beefy red. The skin may become lemon yellow (sallow), caused by a combination of pallor and jaundice. Hepatomegaly, indicating right-sided heart failure, may be present in the elderly. The patient died of heart failure, not brain hypoxia. The patient died of heart failure, not liver hypoxia. The patient died of heart failure, not kidney failure. Cognitive Level: Comprehension 503

21. A 42-year-old male is involved in a motor vehicle accident during which he loses a lot of blood. The nurse realizes he is in acute renal failure caused by: a. Kidney stones b. Immune complex deposition in the glomerulus c. Inadequate renal blood flow d. Obstruction of the proximal tubule

ANS: C With blood loss, renal failure is due to inadequate blood flow. Kidney stones will lead to postrenal renal failure. Intrarenal renal failure is due to glomerular nephritis. Blood loss would not cause obstruction of the proximal tubule. Cognitive Level: Comprehension 754

15. A patient has severe mental retardation caused by a deletion of part of chromosome 5. What genetic disorder will the nurse see documented in the chart? a. Prader-Willi syndrome b. Down syndrome c. Cri du chat syndrome d. Trisomy X

ANS: C Cri du chat syndrome means cry of the cat and describes the characteristic cry of the affected child. Another symptom of the disorder is mental retardation. Prader-Willi syndrome is characterized by short stature, obesity, and hypogonadism. Down syndrome does cause mental retardation, but is due to chromosome 21, not chromosome 5. Trisomy X can result in mental retardation, but is due to an extra X chromosome.

13. Which of the following individuals should the nurse assess first for a vitamin B12 deficiency anemia? a. 3-year-old female who is a fussy eater b. 26-year-old female in the second trimester of her first pregnancy c. 47-year-old male who had a gastrectomy procedure (removal of the stomach) d. 64-year-old male with a history of duodenal ulcers and gastrointestinal bleeding

ANS: C With removal of the stomach, the intrinsic factor is also removed, leading to an inability to absorb B12; thus, the person with removal of the stomach is at greatest risk. With removal of the stomach, the intrinsic factor is also removed, leading to an inability to absorb B12; thus, the person with removal of the stomach is at greatest risk, not the person who is a fussy eater. With removal of the stomach, the intrinsic factor is also removed, leading to an inability to absorb B12; thus, the person with removal of the stomach is at greatest risk, not the person who is pregnant. With removal of the stomach, the intrinsic factor is also removed, leading to an inability to absorb B12; thus, the person with removal of the stomach is at greatest risk, not the person with ulcers. Cognitive Level: Comprehension 503

22. In the liver, free bilirubin moves from the plasma in the sinusoids into the hepatocytes, where it is converted into: a. Unconjugated bilirubin b. Biliverdin c. Conjugated bilirubin d. Urobilinogen

ANS: C Within hepatocytes, unconjugated bilirubin joins with glucuronic acid to form conjugated bilirubin, which is water soluble and is secreted in the bile. Within hepatocytes, unconjugated bilirubin joins with glucuronic acid to form conjugated bilirubin, not unconjugated, which is water soluble and is secreted in the bile. Biliverdin is a precursor of bilirubin. When conjugated bilirubin reaches the distal ileum and colon, it is deconjugated by bacteria and converted to urobilinogen. REF: pp. 884-885

15. An unstable type of hemoglobin that cannot bind with oxygen is termed: a. Deoxyhemoglobin b. Oxyhemoglobin c. Methemoglobin d. Glycosylated hemoglobin

ANS: C Without reactivation, the Fe3+-containing hemoglobin (methemoglobin) cannot bind oxygen. Deoxyhemoglobin is reactivated hemoglobin whose oxygen has been released. Binding of oxygen to ferrous iron temporally oxidizes Fe2+ to Fe3+ as in oxyhemoglobin. Glycosylated hemoglobin is glucose bound to hemoglobin. Cognitive Level: Comprehension 487

13. A 13-year-old girl has a karyotype that reveals an absent homologous X chromosome with only a single X chromosome present. What medical diagnosis will the nurse observe on the chart? a. Down syndrome b. Cri du chat syndrome c. Turner syndrome d. Fragile X syndrome

ANS: C A condition with the presence of a single X chromosome and no homologous X or Y chromosome, so the individual has a total of 45 chromosomes, is known as Turner syndrome. Down syndrome is a change in one arm of a chromosome. Cri du chat syndrome is due to a chromosome deletion. Fragile X syndrome is due to a break or a gap in a chromosome.

7. A cell that does not contain a multiple of 23 chromosomes is called a _____ cell. a. diploid b. euploid c. polyploid d. haploid

ANS: C A polyploid cell is one in which a euploid cell has more than the diploid number of chromosomes. A diploid cell is when the somatic cell nucleus has 46 chromosomes in 23 pairs. A euploid cell is a cell with multiples of the normal number of chromosomes. A haploid cell has only one member of each chromosome pair, for a total of 23 chromosomes.

11. Which statement by the staff indicates teaching was successful concerning aldosterone? Secretion of aldosterone results in: a. Decreased plasma osmolality b. Increased serum potassium levels c. Increased blood volume d. Localized edema

ANS: C Aldosterone promotes renal sodium and water reabsorption and excretion of potassium, thus, increasing blood volume. Aldosterone secretion would cause increased plasma osmolality. Secretion of aldosterone decreases potassium levels because it causes potassium excretion. Secretion of aldosterone does not promote the development of localized edema; it affects blood volume. Cognitive Level: Comprehension 104

26. A group of prison inmates developed tuberculosis following exposure to an infected inmate. On examination, tissues were soft and granular (like clumped cheese). Which of the following is the most likely cause? a. Coagulative necrosis b. Liquefactive necrosis c. Caseous necrosis d. Autonecrosis

ANS: C Caseous necrosis results from tuberculosis pulmonary infection. Coagulative necrosis occurs primarily in the kidneys, heart, and adrenal glands and commonly results from hypoxia. Liquefactive necrosis results from ischemic injury to neurons and glial cells in the brain. Autonecrosis is a process of cellular self-digestion and is not due to infection such as tuberculosis.

7. A 55-year-old male is diagnosed with hepatocellular cancer secondary to hepatitis C. If the cancerous region of the liver were removed, the remaining cells would undergo: a. Pathologic hyperplasia b. Pathologic metaplasia c. Compensatory hyperplasia d. Compensatory aplasia

ANS: C Compensatory hyperplasia is an adaptive mechanism that enables certain organs to regenerate. The cells become compensatory and would not be pathologic. Metaplasia is the reversible replacement of one mature cell type by another, sometimes less differentiated, cell type. Aplasia is not a compensatory mechanism.

22. A child is diagnosed with cystic fibrosis. History reveals that the childs parents are siblings. Cystic fibrosis was most likely the result of: a. X inactivation b. Genomic imprinting c. Consanguinity d. Obligate carriers

ANS: C Consanguinity refers to the mating of two related individuals, and the offspring of such matings are said to be inbred. Consanguineous matings produce a significant increase in recessive disorders and are seen most often in pedigrees for rare recessive disorders. X inactivation occurs when one X chromosome in the somatic cells of females is permanently inactivated. Genomic imprinting is related to methylation and other changes. Obligate carriers are those who have an affected parent and affected children and, therefore, must themselves carry the mutation.

18. A 30-year-old female presents with a gunshot wound to the head. The wound has seared edges and a deep penetration of smoke and gunpowder fragments. This wound would be documented as a(n) _____ wound. a. Exit b. Intermediate range entrance c. Contact range entrance d. Indeterminate range entrance

ANS: C Contact range entrance wound is a distinctive type of wound when the gun is held so the muzzle rests on or presses into the skin surface; in addition to the hole, there is searing of the edges of the wound from flame and soot or smoke on the edges of the wound. It is unlikely it is an exit wound since there is only one wound and it has seared edges and gunpowder fragments. Intermediate range entrance wound is surrounded by gunpowder tattooing or stippling. Indeterminate range entrance wound occurs when flame, soot, or gunpowder does not reach the skin surface, but the bullet does.

12. The early dilation (swelling) of the cells endoplasmic reticulum results in: a. Increased aerobic metabolism b. Failure of DNA c. Reduced protein synthesis d. Decreased Na+-K+ pump function

ANS: C Early dilation of the endoplasmic reticulum causes the ribosomes to detach from the rough endoplasmic reticulum, reducing protein synthesis. Aerobic metabolism is a normal process and would not lead to swelling. Cellular swelling will not alter cellular DNA. A reduction, not an increase, in the Na+-K+ pump leads to an intracellular accumulation of sodium and calcium and diffusion of potassium out of the cell. Sodium and water can then enter the cell freely, and cellular swelling results.

25. A 60-year-old female is diagnosed with hyperkalemia. Which assessment finding should the nurse expect to observe? a. Weak pulse b. Excessive thirst c. Oliguria d. Constipation

ANS: C Hyperkalemia is manifested by oliguria. Hypokalemia is manifested by a weak pulse; it is not caused by hyperkalemia. Hypokalemia is manifested by excessive thirst. Diarrhea, not constipation, is a manifestation of hyperkalemia. Cognitive Level: Comprehension 110

8. A 70-year-old male with chronic renal failure presents with edema. Which of the following is the most likely cause of this condition? a. Increased capillary oncotic pressure b. Decreased interstitial oncotic pressure c. Increased capillary hydrostatic pressure d. Increased interstitial hydrostatic pressure

ANS: C Increased capillary hydrostatic pressure would facilitate increased movement from the capillary to the interstitial space, leading to edema. Increased capillary (plasma) oncotic pressure attracts water from the interstitial space back into the capillary. Decreased interstitial oncotic pressure would keep water in the capillary. Increased interstitial hydrostatic pressure would facilitate increased water movement from the interstitial space into the capillary. Cognitive Level: Comprehension 100

24. Liquefactive necrosis occurs in the brain because: a. Debris is not digested by hydrolases b. Of protein denaturation c. It is rich in hydrolytic enzymes and lipids d. Ischemia results in chemical injury

ANS: C Liquefactive necrosis is due to enzymatic action and because cells of the brain are rich in enzymes. Liquefactive necrosis is not due to hydrolases. Protein denaturation occurs primarily in the kidneys. Liquefactive necrosis is due to enzymatic reaction and not to hypoxia.

22. Which organ system should the nurse monitor when the patient has long-term potassium deficits? a. Central nervous system (CNS) b. Lungs c. Kidneys d. Gastrointestinal tract

ANS: C Long-term potassium deficits lasting more than 1 month may damage renal tissue, with interstitial fibrosis and tubular atrophy. Long-term potassium deficits damage the kidneys, not the CNS. Long-term potassium deficits damage the kidneys, not the lungs. Long-term potassium deficits damage the kidneys, not the gastrointestinal tract. Cognitive Level: Comprehension 108

4. A 55-year-old male with a 30-year history of smoking is examined for respiratory disturbance. Examination of his airway (bronchial) reveals that stratified squamous epithelial cells have replaced the normal columnar ciliated cells. This type of cellular adaptation is called: a. Anaplasia b. Hyperplasia c. Metaplasia d. Dysplasia

ANS: C Metaplasia is the reversible replacement of one mature cell type by another, sometimes a less differentiated cell type. Anaplasia is loss of cellular differentiation. Hyperplasia is an increase in the number of cells resulting from an increased rate of cellular division. Dysplasia refers to abnormal changes in the size, shape, and organization of mature cells.

34. Which of the following disorders is manifested primarily in males? a. Cystic fibrosis b. Neurofibromatosis c. Muscular dystrophy d. Klinefelter syndrome

ANS: C Muscular dystrophy is manifested primarily in males. Cystic fibrosis is manifested in males and females. Neurofibromatosis is manifested in males and females. Klinefelter syndrome is manifested in males and females.

32. Mutations that do not change the amino acid sequence and thus have no consequence are termed _____ mutations. a. Frameshift b. Spontaneous c. Silent d. Missense

ANS: C Silent mutations do not change the amino acid sequence and have no consequences. Frameshift mutation involves the insertion or deletion of one or more base pairs of the DNA molecule. They alter the amino acid sequence. Spontaneous mutations occur in the absence of exposure to a mutagen and produce changes in amino acid sequence. Missense mutations, a form of base pair substitution, alter amino acids, which produce a change (i.e., the sense) in a single amino acid.

8. A 20-year-old pregnant female gives birth to a stillborn child. Autopsy reveals that the fetus has 92 chromosomes. What term may be on the autopsy report to describe this condition? a. Biploidy b. Triploidy c. Tetraploidy d. Aneuploidy

ANS: C Tetraploidy is a condition in which euploid cells have 92 chromosomes. Biploidy is a euploid cell with 2 times more chromosomes, or 46. Triploidy is a zygote that has three copies of each chromosome, rather than the usual two. Aneuploidy is when an aneuploid cell does not contain a multiple of 23 chromosomes.

9. A 75-year-old male presents with chest pain on exertion. The chest pain is most likely due to hypoxic injury secondary to: a. Malnutrition b. Free radicals c. Ischemia d. Chemical toxicity

ANS: C The cardiac cells are deprived of oxygen leading to ischemia, a reduction in blood supply to tissues. The cells are deprived of oxygen; they are not malnourished. Free radicals are electrically uncharged atoms or groups of atoms that have an unpaired electron. Chemical toxicity is not a factor in the chest pain; the pain is related to a lack of oxygen.

Intron mutations are part of RNA sequencing. 3. The base components of DNA are: a. A, G, C, and U b. P, G, C, and T c. A, G, C, and T d. X, XX, XY, and YY

ANS: C The four base components of DNA are cytosine, thymine, adenine, and guanine and are commonly represented by their first letters: A, C, T, and G. The four base components of DNA are cytosine, thymine, adenine, and guanine and are commonly represented by their first letters: A, C, T, and G. U is not included. The four base components of DNA are cytosine, thymine, adenine, and guanine and are commonly represented by their first letters: A, C, T, and G. P is not included. X, XX, XY and YY are components of human chromosomes.

11. The nurse is teaching staff about the most common cause of Down syndrome. What is the nurse describing? a. Paternal nondisjunction b. Maternal translocations c. Maternal nondisjunction d. Paternal translocations

ANS: C The most common cause of Down syndrome is maternal nondisjunction. Down syndrome is not related to paternal nondisjunction. Down syndrome is related to the maternal side, but not due to translocation. Down syndrome is not related to paternal abnormalities.

22. A newborn male is diagnosed with albinism based on skin, eye, and hair appearance. Which finding will support this diagnosis? a. Increased melanin b. Increased hemoproteins c. Inability to convert tyrosine to DOPA (3,4 dihydroxyphenylalanine) d. Inability to convert bile to bilirubin

ANS: C The person with albinism is unable to convert tyrosine to DOPA, an intermediate in melanin biosynthesis. An increase in melanin would cause skin to be darker. Hemoprotein accumulations in cells are caused by excessive storage of iron, which is transferred to the cells from the bloodstream. An inability to convert bile to bilirubin would not lead to albinism.

27. A nurse recalls regulation of acid-base balance through removal or retention of volatile acids is accomplished by the: a. Buffer systems b. Kidneys c. Lungs d. Liver

ANS: C The volatile acid is carbonic acid (H2CO3), which readily dissociates into carbon dioxide (CO2) and water (H2O). The CO2 is then eliminated by the lungs. Buffer systems are throughout the body and operate in the extracellular and intracellular systems. The kidneys release hydrogen ions, not volatile acids. The liver does not regulate acid-base balance. Cognitive Level: Comprehension 111

25. What is the diagnosis of a 13-year-old female who has a karyotype that reveals an absent homologous X chromosome with only a single X chromosome present? Her features include a short stature, widely spaced nipples, reduced carrying angle at the elbow, and sparse body hair. a. Down syndrome b. Cri du chat syndrome c. Turner syndrome d. Klinefelter syndrome

ANS: C Turner syndrome is characterized by short stature, female genitalia, webbed neck, shield-like chest with underdeveloped breasts and widely spaced nipples, and imperfectly developed ovaries. Down syndrome is characterized by distinctive characteristics: low nasal bridge, epicanthal folds, protruding tongue, and low-set ears. Cri du chat syndrome is characterized by low birth weight, severe mental retardation, microcephaly (smaller than normal head size), and heart defects. Klinefelter syndrome is characterized by small testes, some development of the breasts, sparse body hair, and long limbs.

29. Which patient is most prone to metabolic alkalosis? A patient with: a. Retention of metabolic acids b. Hypoaldosteronism c. Excessive loss of chloride (Cl) d. Hyperventilation

ANS: C When acid loss is caused by vomiting, renal compensation is not very effective because loss of Cl stimulates renal retention of bicarbonate, leading to alkalosis. Retention of metabolic acids would lead to acidosis, not alkalosis. Hypoaldosteronism leads to hyponatremia and does not cause alkalosis. Hyperventilation leads to respiratory alkalosis, not metabolic alkalosis. Cognitive Level: Comprehension 113

20. A 55-year-old male has swelling of the feet. Which of the following aided in development of swelling? a. Increased ATP b. Chloride movement out of the cell c. Na+ movement into the cell d. Decreased oncotic pressure

ANS: C When sodium and water enter the cell freely, cellular swelling, as well as early dilation of the endoplasmic reticulum, results. Decreased ATP would lead to swelling, not increased ATP. Chloride movement out of the cell would affect muscle contraction and does not lead to swelling. Increased oncotic pressure would not affect swelling.

29. A 50-year-old male intravenous drug user is diagnosed with hepatitis C. Examination of the liver reveals cell death secondary to: a. Fat necrosis b. Physiologic apoptosis c. Pathologic apoptosis d. Pyknosis

ANS: C With hepatitis C, the liver will demonstrate apoptosis that is pathologic. Fat necrosis occurs with enzymatic action due to lipases. Apoptosis is not a normal physiological process. Pyknosis occurs when the nucleus shrinks and becomes a small, dense mass of genetic material.

21. A 15-year-old male suffers from severe hemorrhage following a motor vehicle accident. He is given a blood transfusion, but shortly afterward the red blood cells are destroyed by agglutination and lysis. Which of the following blood type-transfusion type matches would cause this? a. A-A b. B-O c. AB-O d. A-AB

ANS: D A person with type A blood also has circulating antibodies to the B carbohydrate antigen. If this person receives blood from a type AB or B individual, a severe transfusion reaction occurs, and the transfused erythrocytes are destroyed by agglutination or complement-mediated lysis. Type A can receive type A blood. Type B can receive type O. Type AB can accept type O. Cognitive Level: Comprehension 198

32. A 67-year-old male was diagnosed with polycythemia vera (PV) but refused treatment. His condition is at risk for converting to: a. Chronic lymphocytic leukemia b. Burkitt lymphoma c. Multiple myeloma d. Acute myeloid leukemia

ANS: D A significant potential outcome of PV is the conversion to acute myeloid leukemia (AML), occurring spontaneously in 10% of individuals and generally being resistant to conventional therapy. A significant potential outcome of PV is the conversion to AML, not chronic lymphocytic leukemia. A significant potential outcome of PV is the conversion to AML, not Burkitt lymphoma. A significant potential outcome of PV is the conversion to AML, not multiple myeloma. Cognitive Level: Comprehension 508

18. A 34-year-old male presents in the emergency room with extreme fatigue and shortness of breath. His skin and sclera appear to have a yellowish discoloration. These assessment findings are consistent with which type of anemia? a. Posthemorrhagic anemia b. Iron deficiency anemia c. Aplastic anemia d. Hemolytic anemia

ANS: D A yellow appearance to the skin and sclera are consistent with hemolytic anemia. A yellow appearance to the skin and sclera are consistent with hemolytic anemia, not posthemorrhagic anemia. A yellow appearance to the skin and sclera are consistent with hemolytic anemia, not iron deficiency anemia. A yellow appearance to the skin and sclera are consistent with hemolytic anemia, not aplastic anemia. Cognitive Level: Comprehension 501

24. A nurse assesses the heart after acetylcholine because the effect of acetylcholine on the heart is to: a. Decrease the refractory period b. Increase calcium influx c. Increase the strength of myocardial contraction d. Decrease the heart rate

ANS: D Acetylcholine causes decreased heart rate and slows conduction through the atrioventricular (AV) node. Acetylcholine causes decreased heart rate and slows conduction through the AV node; it does not decrease the refractory period. It decreases calcium influx. Acetylcholine causes decreased heart rate and slows conduction through the AV node; it does not increase the strength of myocardial contraction. Cognitive Level: Comprehension 560

5. After initial compensation, what hemodynamic change should the nurse monitor for in a patient who has a reduction in the number of circulating erythrocytes? a. Increased viscosity of blood b. Decreased cardiac output c. Altered coagulation d. Hyperdynamic circulatory state

ANS: D After initial compensation, the blood flows faster and more turbulently than normal blood, causing a hyperdynamic circulatory state. Blood viscosity decreases rather than increases. Cardiac output increases. Alteration in coagulation does not occur. Cognitive Level: Comprehension 501

36. A nurse recalls bacteria become resistant to antimicrobials by: a. Proliferation b. Attenuation c. Specialization d. Mutation

ANS: D Antibiotic resistance is usually a result of genetic mutations that can be transmitted directly to neighboring microorganisms by plasmid exchange. Antibiotic resistance is a result of mutations, not proliferation, which is increased growth. Antibiotic resistance is a result of genetic mutations, not attenuation. Antibiotic resistance is a result of genetic mutations, not specialization. Cognitive Level: Comprehension 176

6. The patient asks how the GI system works. The nurse would identify that autonomic and hormonal control are involved in GI motility and: a. Defecation b. Swallowing c. Mastication d. Digestive enzyme secretion

ANS: D Autonomic control is involved in digestive enzyme secretion. Autonomic control is involved in digestive enzyme secretion, not defecation. Autonomic control is involved in digestive enzyme secretion, not swallowing. Autonomic control is involved in digestive enzyme secretion, not mastication. Cognitive Level: Comprehension 873

6. A patient has researched white blood cells on the Internet. Which statement indicates the patient has a good understanding? _______________ contain preformed granules of vasoactive amines. a. Neutrophils b. Eosinophils c. Monocytes d. Basophils

ANS: D Basophils have cytoplasmic granules that contain vasoactive amines (e.g., histamine) and an anticoagulant (heparin). Their function is similar to tissue mast cells. Neutrophils are the chief phagocytes of early inflammation. Eosinophils ingest antigen-antibody complexes and are induced by IgE-mediated hypersensitivity reactions to attack parasites. Monocytes do not have amines, and their job it to help fight infection. Cognitive Level: Comprehension 480

3. A 65-year-old male develops blockage in the pulmonary artery. As a result of the blockage, blood would first back up into the: a. Aorta b. Left ventricle c. Pulmonary veins d. Right ventricle

ANS: D Blockage in the pulmonary artery would cause blood to back up into the right ventricle. Blockage in the pulmonary artery would cause blood to back up into the right ventricle, not the aorta. These two vessels do not communicate. Blockage in the pulmonary artery would cause blood to back up into the right ventricle, not the left ventricle. This vessel and chamber do not communicate. Blockage in the pulmonary artery would cause blood to back up into the right ventricle, not the pulmonary vein. The pulmonary vein takes blood to the left atrium. Cognitive Level: Comprehension 553

21. Which electrolyte will the nurse check to ensure normal platelet functioning? a. Sodium b. Potassium c. Magnesium d. Calcium

ANS: D Calcium is necessary for many of the intracellular signaling mechanisms that control platelet activation. Calcium, not sodium, is necessary for many of the intracellular signaling mechanisms that control platelet activation. Calcium, not potassium, is necessary for many of the intracellular signaling mechanisms that control platelet activation. Calcium, not magnesium, is necessary for many of the intracellular signaling mechanisms that control platelet activation. Cognitive Level: Comprehension 491

9. Which information should the nurse include when teaching about angiogenic factors? In cancer, angiogenic factors stimulate: a. Release of growth factors b. Tumor regression c. Apoptosis d. New blood vessel growth

ANS: D Cancers can secrete multiple factors that stimulate new blood vessel growth, which is called angiogenesis. Cancers can secrete multiple factors that stimulate new blood vessel growth called angiogenesis, not release of growth factors. Cancers can secrete multiple factors that stimulate new blood vessel growth called angiogenesis, not tumor regression. Apoptosis is cell death. Cognitive Level: Comprehension 235

1. Which statement indicates the patient has a good understanding of cancer risk factors? The most important environmental risk factor for cancer is exposure to: a. Ultraviolet (UV) radiation b. Radon c. Estrogen d. Cigarette smoke

ANS: D Cigarette smoking is carcinogenic and remains the most important cause of cancer. UV radiation is an important risk factor, but cigarette smoking remains the most important cause of cancer. Radon plays a role as a risk factor for cancer, but cigarette smoking remains the most important cause of cancer. Estrogen plays a role in the risk factors of cancer, but cigarette smoking remains the most important cause of cancer. Cognitive Level: Comprehension 261

24. A 56-year-old male is diagnosed with coronary artery disease. Which of the following modifiable risk factors would the nurse suggest the patient change? a. Eating meat b. Living arrangements c. Drinking tomato juice d. Smoking cigarettes

ANS: D Cigarette smoking leads to vasoconstriction and should be the first behavior the patient changes. Eating meat alone would not lead to the development of coronary artery disease; cigarette smoking leads to vasoconstriction and should be the first behavior the patient changes. The living arrangement of the patients home would not lead to the development of coronary artery disease; cigarette smoking leads to vasoconstriction and should be the first behavior the patient changes. Drinking tomato juice would not lead to the development of coronary artery disease; cigarette smoking leads to vasoconstriction and should be the first behavior the patient changes. Cognitive Level: Comprehension 594

16. After teaching about coping, which information indicates a correct understanding? Coping is best defined as the process of: a. Adjusting to disease b. Preventing psychological distress c. Mediating anger d. Managing stressful challenges

ANS: D Coping is the process of managing stressful challenges that tax the individuals resources. Coping is the process of managing stressful challenges that tax the individuals resources; it is not adjusting to disease. Coping is the process of managing stressful challenges that tax the individuals resources; it is not preventing psychological distress. Coping is the process of managing stressful challenges that tax the individuals resources; it is not mediating anger. Cognitive Level: Comprehension 217

2. When an oncologist is discussing the degree to which an organisms development is contingent on its environment, which of the following is the oncologist explaining? a. Transgenerational inheritance b. Epigenetics c. Histone modification d. Developmental plasticity

ANS: D Developmental plasticity is the degree to which an organisms development is contingent on its environment. Transgenerational inheritance is the heritable transmission to future generations of environmentally caused phenotypes. Epigenetics is the role of genes in development and disease. Histone modifications are changes in genetic acetylation. Cognitive Level: Comprehension 260

31. A 25-year-old female reports having unprotected sexual intercourse with several men. Blood tests reveal that she is positive for human papillomavirus. What else should the nurse assess for? a. Vaginal discharge b. Liver failure c. Breast cancer d. Warts

ANS: D Direct contact with papillomavirus can lead to warts. Direct contact with papillomavirus can lead to warts, not vaginal discharge. Direct contact with papillomavirus can lead to warts, not liver failure. Direct contact with papillomavirus can lead to warts, not breast cancer. Cognitive Level: Comprehension 172

32. After studying about fungi, which information indicates a correct understanding of fungi? Fungi causing deep or systemic infections: a. Are easily treated with penicillin b. Are extremely rare c. Never occur with other infections d. Are commonly opportunistic

ANS: D Diseases caused by fungi are called mycoses. Mycoses can be opportunistic. Diseases caused by fungi are called mycoses. Mycoses can be opportunistic and are not treatable with penicillin. Diseases caused by fungi are called mycoses. Mycoses can be opportunistic and are common. Diseases caused by fungi are called mycoses. Mycoses can be opportunistic and occur with other infections. Cognitive Level: Comprehension 174

47. A 10-year-old male presents with abdominal swelling, night sweats, fever, and weight loss. He is diagnosed with Burkitt lymphoma. Upon obtaining the history, which of the following is the most likely cause? a. Cytomegalovirus (CMV) b. Adenovirus c. Human papillomavirus (HPV) d. Epstein-Barr virus (EBV)

ANS: D EBV is associated with almost all cases of Burkitt lymphoma. EBV, not CMV, is associated with almost all cases of Burkitt lymphoma. EBV, not adenovirus, is associated with almost all cases of Burkitt lymphoma. EBV, not HPV, is associated with almost all cases of Burkitt lymphoma. Cognitive Level: Comprehension 519

4. A group of cells isolated in the laboratory have membrane-bound granules in their cytoplasm and they show phagocytic activity. Which of the following cells is most similar? a. Monocyte b. Macrophage c. Lymphocyte d. Eosinophils

ANS: D Eosinophils, which have large, coarse granules, are capable of ameboid movement and phagocytosis. Monocytes help fight infection. Macrophages are mature monocytes. Lymphocytes are the primary cells of the immune response. Cognitive Level: Comprehension 480

24. A nurse is caring for a patient who cannot clot. Which end product of the clotting cascade is this patient unable to make? a. Collagen b. Fibrinogen c. Thrombin d. Fibrin

ANS: D Fibrin is the end product of the coagulation cascade. Fibrin, not collagen, is the end product of the coagulation cascade. Fibrin, not fibrinogen, is the end product of the coagulation cascade. Fibrin, not thrombin, is the end product of the coagulation cascade. Cognitive Level: Comprehension 491

28. A 35-year-old male is diagnosed with lobar pneumonia (lung infection). Which of the following exudates would be present in highest concentration at the site of this advanced inflammatory response? a. Serous b. Purulent c. Hemorrhagic d. Fibrinous

ANS: D Fibrinous exudates occur in the lungs of individuals with pneumonia. Serous fluid is watery fluid, as in a blister. Purulent is characterized by an abscess, such as pus. Hemorrhagic occurs when the exudates are filled with erythrocytes. Cognitive Level: Comprehension 132

30. A patient has chicken pox. How does the varicella replicate? a. With the host cell DNA b. Using host cell DNA polymerase c. Using reverse transcriptase d. In the cytoplasm

ANS: D Generally, all RNA viruses replicate their genetic material in the cytoplasm of the infected cell. Generally, all RNA viruses replicate their genetic material in the cytoplasm of the infected cell, not the host cell DNA. Generally, all RNA viruses replicate their genetic material in the cytoplasm of the infected cell, not the polymerase. Generally, all RNA viruses replicate their genetic material in the cytoplasm of the infected cell, not using reverse transcriptase. Cognitive Level: Comprehension 171

6. A 30-year-old Caucasian female was recently diagnosed with primary hypertension. She reports that she eats fairly well, usually having red meat and potatoes daily. She also reports that her father has hypertension as well. A nurse determines which of the following risk factors is most likely associated with this diagnosis? a. Race b. Diet c. Age d. Genes

ANS: D Genetic factors, such as family history of hypertension, are the number one factor in the development of hypertension. Race is a factor, but genetic factors are primary. Diet is a factor, but genetic factors are primary. Age is a factor, but not in this case; since the patient is 30, genetics are a greater factor. Cognitive Level: Comprehension 587

15. A 15-year-old male was diagnosed with pharyngitis. Eight days later he developed acute glomerulonephritis. While reviewing the culture results, which of the following is the most likely cause of this disease? a. Klebsiella b. Human immunodeficiency virus (HIV) c. Genital herpes virus d. Group A -hemolytic streptococcus

ANS: D Group A -hemolytic streptococcus is the primary cause of glomerulonephritis associated with a pharyngeal infection. Group A -hemolytic streptococcus is the primary cause of glomerulonephritis associated with a pharyngeal infection. Klebsiella is not associated with pharyngitis. Group A -hemolytic streptococcus is the primary cause of glomerulonephritis associated with a pharyngeal infection. HIV is not associated with pharyngitis. Group A -hemolytic streptococcus is the primary cause of glomerulonephritis associated with a pharyngeal infection. Genital herpes is not associated with pharyngitis. Cognitive Level: Comprehension 753

19. Which condition will cause a patient to secrete erythropoietin? a. Low blood pressure b. Hypercarbia c. Inflammation d. Hypoxia

ANS: D Hypoxia stimulates production of erythropoietin. Hypoxia, not low blood pressure, stimulates production of erythropoietin. Hypoxia, not hypercarbia, stimulates production of erythropoietin. Hypoxia, not inflammation, stimulates production of erythropoietin. Cognitive Level: Comprehension 486

When a patient asks what causes hyperglycemia in type 2 DM, how should the nurse respond? Hyperglycemia is a result of: a. Insulin deficiency b. Hyperinsulinemia c. Glucagon deficiency d. Liver dysfunction

B

27. A 54-year-old male intravenous (IV) drug user is diagnosed with chronic hepatitis C. Testing revealed that he is a candidate for treatment. Which of the following could be used to treat his condition? a. Interleukin-1 b. Interleukin-6 c. Interleukin-10 d. INFs

ANS: D INFs are members of a family of cytokines that protect against viral infections. Interleukin-1 is responsible for fever production. Interleukin-6 stimulates growth and differentiation of blood cells. Interleukin-10 helps decrease the immune response. Cognitive Level: Comprehension 127

25. A nurse is preparing to teach the staff about aplastic anemia. Which information should the nurse include? Aplastic anemia is caused by: a. Iron deficiency b. Excess levels of erythropoietin c. Hemolysis d. Stem cell deficiency

ANS: D In aplastic anemia, erythrocyte stem cells are underdeveloped, defective, or absent. In aplastic anemia, erythrocyte stem cells are underdeveloped, defective, or absent; iron is not deficient. In aplastic anemia, erythrocyte stem cells are underdeveloped, defective, or absent. There are no excess levels of erythropoietin. In aplastic anemia, erythrocyte stem cells are underdeveloped, defective, or absent. Cells do not undergo hemolysis. Cognitive Level: Comprehension 506

7. A child fell off the swing and scraped the right knee. The injured area becomes painful. What else will the nurse observe upon assessment? a. Vasoconstriction at injured site b. Decreased RBC concentration at injured site c. Pale skin at injured site d. Edema at injured site

ANS: D Increased vascular permeability and leakage of fluid out of the vessel cause edema at the site of injury. Vasodilation occurs, not vasoconstriction. Increased RBCs come to the site, not fewer. Redness occurs, not paleness, during inflammation. Cognitive Level: Comprehension 121

14. Which of the following patients would be at greatest risk for basal cell carcinoma? a. Dark complexion, light eyes, underweight b. Light complexion, dark eyes, overweight c. Medium complexion, light eyes, smoker d. Light complexion, light eyes, fair hair

ANS: D Individuals at risk for basal cell carcinoma are light complected and have light eyes and fair hair. Individuals at risk for basal cell carcinoma are light complected and have light eyes and fair hair. Individuals at risk for basal cell carcinoma are light complected and have light eyes and fair hair. Individuals at risk for basal cell carcinoma are light complected and have light eyes and fair hair. Cognitive Level: Comprehension 274

11. A 20-year-old female undergoes lab testing for anemia. Results show high iron, bilirubin, and transferrin and low hemoglobin and hematocrit. Which of the following is the most likely diagnosis to be documented on the chart? a. Pernicious anemia b. Folate deficiency anemia c. Iron deficiency anemia d. Sideroblastic anemia

ANS: D Individuals with sideroblastic anemia may show signs of iron overload (hemosiderosis), including mild to moderate enlargement of the liver (hepatomegaly) and spleen (splenomegaly). High levels of iron indicate sideroblastic anemia, not pernicious anemia. High levels of iron indicate sideroblastic anemia, not folate deficiency. High levels of iron indicate sideroblastic anemia, not iron deficiency anemia. Cognitive Level: Comprehension 505

17. A student asks the instructor about recycled iron. What information should the instructor share? Recycled iron from macrophages is delivered to the bone marrow bound to: a. Ferritin b. Hemoglobin c. Hemosiderin d. Transferrin

ANS: D Iron is transported in the blood bound to transferrin. Iron is transported in the blood bound to transferrin, ferritin is the storage form of iron. Iron is transported in the blood bound to transferrin, hemoglobin carries oxygen. Iron is transported in the blood bound to transferrin, hemosiderin is also a storage form of iron. Cognitive Level: Comprehension 488

14. Which factor will help the nurse differentiate leukotrienes from histamine? a. Site of production b. Vascular effect c. Chemotactic ability d. Time of release

ANS: D Leukotrienes are released slower and longer than histamine. Leukotrienes and histamine are produced from mast cells. Leukotrienes and histamine have similar vascular effects. Leukotrienes and histamine have similar chemotactic ability. Cognitive Level: Comprehension 129

6. When a nurse is checking the urinalysis, plasma proteins should be absent from the urine because: a. All proteins filtered are subsequently reabsorbed. b. All of the plasma proteins are too large to fit through the filtration slits. c. All proteins filtered are subsequently degraded before elimination. d. The negative charge of the glomerular filtration membrane repels the plasma proteins.

ANS: D Like other capillary membranes, the glomerulus is freely permeable to water and relatively impermeable to large colloids, such as plasma proteins. The molecules size and electrical charge affect the permeability of substances crossing the glomerulus. Proteins are not filtered because the negative charge of the glomerulus repels them. Proteins are not filtered because the negative charge of the glomerulus repels them. It is not only because of their size. Proteins are not filtered because the negative charge of the glomerulus repels them. It is not because of degradation. Cognitive Level: Comprehension 726

43. A 35-year-old female is diagnosed with lymphadenopathy. Which assessment finding will help confirm this diagnosis? a. Small, hard lymph nodes b. Disordered lymph nodes c. Nonpalpable, nontender lymph nodes d. Enlarged lymph nodes

ANS: D Lymphadenopathy is evidenced by enlarged lymph nodes. Lymphadenopathy is evidenced by enlarged lymph nodes, not small and hard nodes. Lymphadenopathy is evidenced by enlarged lymph nodes, not disordered nodes. Lymphadenopathy is evidenced by enlarged lymph nodes, not non palpable lymph nodes. Cognitive Level: Comprehension 515

32. The macrophage secretion that stimulates procollagen synthesis and secretion is: a. Angiogenesis factor b. Matrix metalloproteinase c. Vascular endothelial growth factor d. Transforming growth factor-beta

ANS: D Macrophages secrete transforming growth factor-beta to stimulate fibroblasts to secrete the collagen precursor procollagen.

25. Which of the following symptoms would help a health care provider distinguish between ulcerative colitis and Crohn disease? a. Abdominal pain b. Pattern of remission/exacerbations c. Diarrhea d. Malabsorption

ANS: D Malabsorption is common in Crohn disease and is rare in ulcerative colitis. Both disorders can lead to abdominal pain. Both disorders have a clinical course of remissions and exacerbations. Both disorders lead to diarrhea. REF: pp. 908-909

20. A 54-year-old female is diagnosed with nephrotic syndrome. Which of the following is a common symptom of this disease? a. Hematuria b. Dysuria c. Oliguria d. Proteinuria

ANS: D Nephrotic syndrome is manifested by proteinuria. Nephrotic syndrome is diagnosed when the protein level in a 24-hour urine collection is greater than 3.5 g. Proteinuria is the diagnostic criteria for nephrotic syndrome. The patient with nephrotic syndrome does not experience dysuria. The patient with nephrotic syndrome does not experience oliguria. Cognitive Level: Comprehension 753

5. While planning care for a patient with inflammation, which principle will the nurse remember? The predominant phagocyte of early inflammation is the: a. Eosinophil b. Lymphocyte c. Macrophage d. Neutrophil

ANS: D Neutrophils are the chief phagocytes of early inflammation. Eosinophils ingest antigen-antibody complexes and are induced by IgE-mediated hypersensitivity reactions to attack parasites. Most lymphocytes transiently circulate in the blood and eventually reside in lymphoid tissues as mature T cells, B cells, or plasma cells. Macrophages migrate out of the vessels in response to infection or inflammation, but are not the early responders. Cognitive Level: Comprehension 480

23. The microorganisms that make up the normal human flora are important for: a. Regulating inflammation b. Secreting bacteriostatic substances c. Activating white blood cells d. Preventing the colonization and multiplication of pathogens

ANS: D Normal flora are provided with nutrients from ingested food, and in exchange they produce enzymes that produce antibacterial factors that prevent colonization by pathogenic microorganisms. Normal flora prevent colonization of pathogens; they do not regulate inflammation. Normal flora prevent colonization of pathogens; they do not secrete bacteriostatic substances. Normal flora prevent colonization of pathogens; they do not activate white blood cells. Cognitive Level: Comprehension 166

3. The chemist analyzing a sample of normal saliva would expect it to reveal which immunoglobulin? a. IgG b. IgD c. IgE d. IgA

ANS: D Normal saliva contains IgA. Normal saliva contains IgA, not IgG. Normal saliva contains IgA, not IgD. Normal saliva contains IgA, not IgE. Cognitive Level: Comprehension 873

18. What is the chance that two siblings share both HLA haplotypes, making them a good match for an organ transplant from one to the other? a. 100% b. 75% c. 50% d. 25%

ANS: D Odds dictate that children will share one haplotype with half their siblings and either no haplotypes or both haplotypes with a quarter of their siblings. Thus, the chance of finding a match among siblings is much higher (25%) than the general population. 100% is too high; the chance is 25%. 75% is too high; the chance is 25%. 50% is too high; the chance is 25%. Cognitive Level: Comprehension 199

22. A nurse is preparing to teach on the subject of opsonins. Which information should the nurse include? Opsonins are molecules that: a. Are composed of fatty acids b. Regulate inflammation c. Degranulate mast cells d. Enhance phagocytosis

ANS: D Opsonins coat the surface of bacteria and increase their susceptibility to being phagocytized. Opsonins are not composed of fatty acids; they are antibodies. Opsonins coat the surface of bacteria and increase their susceptibility to being phagocytized. They do not regulate inflammation; mast cells do. Opsonins coat the surface of bacteria and increase their susceptibility to being phagocytized; they do not react with mast cells. Cognitive Level: Comprehension 122

12. A 50-year-old male is diagnosed with orthostatic hypotension. Which of the following symptoms would he most likely experience? a. Headache and blurred vision b. Nausea and vomiting c. Chest pain and palpitations d. Syncope and fainting

ANS: D Orthostatic hypotension is often accompanied by dizziness, blurring or loss of vision, and syncope or fainting. Orthostatic hypotension is often accompanied by dizziness, blurring or loss of vision, and syncope or fainting, not by headache or blurred vision, which are symptoms of hypertension. Orthostatic hypotension is often accompanied by dizziness, blurring or loss of vision, and syncope or fainting, not nausea and vomiting. Orthostatic hypotension is often accompanied by dizziness, blurring or loss of vision, and syncope or fainting, not chest pain and palpitations, which may be symptomatic of myocardial infarction. Cognitive Level: Comprehension 591

17. A 40-year-old female is diagnosed with SLE. Which of the following findings would be considered a symptom of this disease? a. Gastrointestinal ulcers b. Decreased glomerular filtration rate c. Rash on trunk and extremities d. Photosensitivity

ANS: D Photosensitivity is one of the 11 common clinical findings in SLE. Gastrointestinal ulcers are not a finding in SLE. Proteinuria is a symptom of SLE. A rash on the face is a symptom, but not a rash on the body. Cognitive Level: Comprehension 198

9. A nurse recalls physical activity was shown to reduce the risk of which of the following types of cancer? a. Prostate b. Lung c. Bone d. Colon

ANS: D Physical activity reduces the risk for breast and colon cancers. Physical activity reduces the risk for breast and colon cancers, but not prostate. Physical activity reduces the risk for breast and colon cancers, but not lung. Physical activity reduces the risk for breast and colon cancers, but not bone. Cognitive Level: Comprehension 278

8. A 45-year-old male complains of heartburn after eating and difficulty swallowing. He probably has: a. Pyloric stenosis b. Gastric cancer c. Achalasia d. Hiatal hernia

ANS: D Regurgitation, dysphagia, and substernal discomfort after eating are common in individuals with hiatal hernia. Pyloric stenosis is manifested by projectile vomiting. Gastric cancer is not manifested by heartburn. Achalasia is a form of functional dysphagia caused by loss of esophageal innervation. Cognitive Level: Comprehension 899

2. A 5-year-old female takes a hike through the woods during a school field trip. Upon returning home, she hugs her father, and he later develops poison ivy. Which of the following immune reactions is he experiencing? a. IgE-mediated b. Tissue specific c. Immune complex d. Cell-mediated

ANS: D Secondary contact activates a type IV cell-mediated reaction that causes dermatitis. The fathers response is a secondary, cell mediated, not a primary type IgE mediated. The fathers response is secondary, not tissue specific. The fathers response is a type IV, not an immune complex response. Cognitive Level: Comprehension 189

6. A 24-year-old female is diagnosed with renal calculus that is causing obstruction. Which of the following symptoms would she most likely experience? a. Anuria b. Hematuria c. Pyuria d. Flank pain

ANS: D Significant flank pain is the most common manifestation. Anuria does not occur. Hematuria does occur, but it is not the most common manifestation. Flank pain is the most common symptom; pyuria does not occur. Cognitive Level: Comprehension 743

12. A 10-month-old is brought to the pediatrician by the mother who states the baby has been experiencing colicky pain followed by vomiting, sweating, nausea, and irritability. Testing reveals a condition in which one part of the intestine telescopes into another. From which type of intestinal obstruction is he suffering? a. Hernia b. Ileus c. Torsion d. Intussusception

ANS: D Telescoping of one part of the intestine into another; this usually causes strangulation of the blood supply and is more common in infants 10 to 15 months of age than in adults. A hernia does not involve telescoping of the intestines. An ileus could lead to bowel obstruction, but it does not involve telescoping. Torsion can lead to pain and vomiting, but it does not involve telescoping. Cognitive Level: Comprehension 900

12. A 30-year-old male is having difficulty breathing and has been spitting blood. He reports that he began experiencing this reaction after cleaning his pigeons cages. Testing reveals he is suffering from allergic alveolitis. Which of the following is he experiencing? a. Serum sickness b. Raynaud phenomenon c. Antibody-dependent cytotoxicity d. Arthus reaction

ANS: D The Arthus reaction is a model of localized or cutaneous reactions. Serum sickness type reactions are caused by the formation of immune complexes in the blood and their subsequent generalized deposition in target tissues. Typically affected tissues are the blood vessels, joints, and kidneys. Raynaud phenomenon is a condition caused by the temperature-dependent deposition of immune complexes in the capillary beds of the peripheral circulation. Antibody-dependent cytotoxicity is a type II form. Cognitive Level: Comprehension 193

12. A 20-year-old female is applying for nursing school and is required to be tested for immunity against several illnesses. Testing that looks at which of the following would be the best to determine immunity? a. Culture and sensitivity b. Agglutination c. Precipitation d. Titer

ANS: D The amount of antibody in a serum sample is referred to as the titer; a higher titer indicates more antibodies. Culture and sensitivity determine the type of organism that causes an infection, and sensitivity identifies the antibody it is sensitive to. The amount of antibody in a serum sample is referred to as the titer, not agglutination. The amount of antibody in a serum sample is referred to as the titer, not precipitation. Cognitive Level: Comprehension 155

22. A 6-month-old infant is brought to the pediatrician for recurrent colds. The child is fed formula, and the other family members have not been ill. The primary care provider tells the mother that the childs immune system is not yet competent, but will be about 60% of an adult system around age _____ months. a. 3 b. 6 c. 9 d. 12

ANS: D The child has an immune system of about 60% of the adult by age 12 months. The child has an immune system of about 60% of the adult by age 12 months, not 3 months. The child has an immune system of about 60% of the adult by age 12 months, not 6 months. The child has an immune system of about 60% of the adult by age 12 months, not 9 months. Cognitive Level: Comprehension 164

21. When a student nurse asks the nurse how the cardiac electrical impulse normally begins, what is the most correct response? The cardiac electrical impulse normally begins spontaneously in the sinoatrial (SA) node because it: a. Has a superior location in the right atrium b. Is the only area of the heart capable of spontaneous depolarization c. Has rich sympathetic innervation via the vagus nerve d. Depolarizes more rapidly than other automatic cells of the heart

ANS: D The electrical impulse normally begins in the SA node because its cells depolarize more rapidly than other automatic cells. Cardiac impulses occur in the SA node because its cells depolarize more rapidly than other automatic cells, not because of its superior location. Cardiac impulses occur in the SA node because its cells depolarize more rapidly than other automatic cells. The SA node is not the only area of the heart capable of spontaneous depolarization. Cardiac impulses occur in the SA node because its cells depolarize more rapidly than other automatic cells. It does have rich innervations, but by parasympathetic innervation from the vagus nerve. Cognitive Level: Comprehension 557

10. Which sphincter prevents reflux of digested material from the colon into the small intestine? a. Haustral b. Lower esophageal c. Pyloric d. Ileocecal valve

ANS: D The ileocecal valve prevents reflux of digested material from the colon into the small intestine. The haustral segments facilitate propulsion; they do not prevent reflux. Lower esophageal prevents reflux from the stomach to the esophagus. The pylorus is the opening between the stomach and the duodenum. Cognitive Level: Comprehension 878

16. A 54-year-old male is diagnosed with left bundle branch block. Which of the following structures would not receive an electrical impulse? a. Atrioventricular (AV) node b. Sinoatrial (SA) node c. Bundle of His d. The left ventricle

ANS: D The left bundle branch conducts impulses to the left ventricle. The left bundle branch conducts impulses to the left ventricle; the AV node feeds conduction of the bundles. The left bundle branch conducts impulses to the left ventricle; the SA node initiates conduction. The left bundle branch conducts impulses to the left ventricle; the bundle of His precedes the division of the left and right bundles. Cognitive Level: Comprehension 557

18. Which of the following statements indicates more teaching is needed regarding secondary lymph organs? ________is/are a secondary lymph organ. a. The spleen b. Peyer patches c. Adenoids d. The liver

ANS: D The liver is not a secondary lymph organ. The spleen is a secondary lymphoid organ. Peyer patches are a secondary lymph organ. The adenoids are a secondary lymph organ. Cognitive Level: Comprehension 155

27. The most common cause of chronic vascular insufficiency among the elderly is: a. Anemia b. Aneurysm c. Lack of nutrition in gut lumen d. Atherosclerosis

ANS: D The most common cause of chronic vascular insufficiency is atherosclerosis, especially in the elderly. Anemia does not lead to vascular insufficiency. An aneurysm would lead to acute vascular insufficiency. Lack of nutrition does not lead to vascular insufficiency; it is due to atherosclerosis. Cognitive Level: Comprehension 911

7. A 25-year-old female presents with burning urination. She was diagnosed with a urinary tract infection. When the nurse checks the culture results, which of the following organisms is most likely infecting her urinary tract? a. Streptococcus b. Candida albicans c. Chlamydia d. Escherichia coli

ANS: D The most common infecting microorganisms are uropathic strains of Escherichia coli. The most common infecting microorganisms are uropathic strains of Escherichia coli. Streptococcus in not a factor. The most common infecting microorganisms are uropathic strains of Escherichia coli, not Candida albicans. The most common infecting microorganisms are uropathic strains of Escherichia coli, not Chlamydia. Cognitive Level: Comprehension 748

9. Which statement indicates the nurse understands coronary ostia? The coronary ostia (the openings to the coronary arteries) are found in the: a. Left ventricle b. Inferior vena cava c. Coronary sinus d. Aorta

ANS: D The ostia are found on the aorta. The ostia are found on the aorta, not the ventricle. The ostia are found on the aorta, not the vena cava. The ostia are found on the aorta, not the coronary sinus. Cognitive Level: Comprehension 556

28. The pancreas produces which substance to prevent the premature activation of proteolytic enzymes in the pancreas? a. Bicarbonate b. Carboxypeptidase c. Secretin d. Trypsin inhibitor

ANS: D The pancreas produces trypsin inhibitor, which prevents the activation of proteolytic enzymes while they are in the pancreas. Bicarbonate is responsible for neutralizing fluid. Carboxypeptidase is a protein-digesting enzyme. Secretin is not produced by the pancreas. Cognitive Level: Comprehension 889

23. A 56-year-old male presents with flank pain and polyuria. Tests reveal that he has an enlarged prostate. Which of the following types of renal failure should the nurse monitor for as it is the most likely to occur? a. Prerenal b. Intrarenal c. Extrarenal d. Postrenal

ANS: D The patient will experience postrenal renal failure due to obstruction by the prostate. The patient will experience postrenal renal failure due to obstruction by the prostate. The patient will experience postrenal renal failure due to obstruction by the prostate. The patient will experience postrenal renal failure due to obstruction by the prostate. Cognitive Level: Comprehension 755

9. While planning care for a newborn, the pediatric nurse recalls the main site of hematopoiesis in the fetus is the: a. Bone marrow b. Liver c. Lymph nodes d. Spleen

ANS: D The spleen is the largest of the lymphoid organs and is the site of fetal hematopoiesis. The spleen, not the bone marrow, is the site of fetal hematopoiesis. The spleen, not the liver, is the site of fetal hematopoiesis. The spleen, not the lymph nodes, is the site of fetal hematopoiesis. Cognitive Level: Comprehension 482

5. A male student arrives at school to find that he has an exam for which he is unprepared. Which of the following would be expected? a. Decreased lipolysis b. Bronchoconstriction c. Decreased cortisol release d. Increased glucagon release

ANS: D The student would experience increased glucagon release to supply glucose. Increased lipolysis would occur, not decreased. Bronchodilation would occur, not constriction. The student would experience increased cortisol release. Cognitive Level: Comprehension 206 | p. 211

22. A 28-year-old female with seizure disorder has a vagus nerve stimulator implanted to help control seizure activity. Which of the following would the nurse also expect to occur? a. Increased speed of cardiac cycle b. Increased cardiac contractility c. Decreased vasodilation d. Decreased cardiac conduction

ANS: D The vagus nerve releases acetylcholine. Acetylcholine causes decreased heart rate and slows conduction through the atrioventricular (AV) node. The vagus nerve releases acetylcholine. Acetylcholine causes decreased heart rate and slows conduction through the AV node, thus decreasing speed of the cardiac cycle. The vagus nerve releases acetylcholine. Acetylcholine causes decreased heart rate and slows conduction through the AV node, decreasing contractility. The vagus nerve releases acetylcholine. Acetylcholine causes decreased heart rate and slows conduction through the AV node; it does not cause vasodilation. Cognitive Level: Comprehension 560

9. A urologist is discussing a structure that supplies blood to the medulla. What is the urologist describing? a. Renal arteries b. Arcuate arteries c. Peritubular capillaries d. Vasa recta

ANS: D The vasa recta is a network of capillaries that forms loops and closely follows the loops of Henle and is the only blood supply to the medulla. The vasa recta is the only blood supply to the medulla. The renal arteries do not meet this need. The vasa recta is the only blood supply to the medulla. The arcuate arteries do not meet this need. The vasa recta is the only blood supply to the medulla. The peritubular capillaries do not meet this need. Cognitive Level: Comprehension 734

10. When a patient wants to know why vitamin B12 and folate deficiencies cause anemia, how should the nurse respond? a. Red blood cells are unable to differentiate into erythrocytes. b. Red blood cells have malformed hemoglobin molecules. c. Red blood cells have decreased O2-carrying capacity. d. Red blood cells have a shorter life span.

ANS: D These deficiencies lead to defective erythrocytes that die prematurely, which decreases their numbers in the circulation, causing anemia. These deficiencies lead to defective erythrocytes that die prematurely, which decreases their numbers in the circulation, causing anemia. It is not due to a lack of differentiation. These deficiencies lead to defective erythrocytes that die prematurely, which decreases their numbers in the circulation, causing anemia. It is not due to malformed hemoglobin. These deficiencies lead to defective erythrocytes that die prematurely, which decreases their numbers in the circulation, causing anemia. It is not due to decreased oxygen carrying capacity. Cognitive Level: Comprehension 506

13. A 65-year-old female presents to the emergency department reporting difficulty swallowing and shortness of breath. A CT scan would most likely reveal an aneurysm in the: a. Cerebral vessels b. Renal arteries c. Inferior vena cava d. Thoracic aorta

ANS: D Thoracic aortic aneurysms can cause dysphagia (difficulty swallowing) and dyspnea (breathlessness). Aneurysms in cerebral vessels will produce a headache. Aneurysms in the renal arteries will produce flank pain. Aneurysms in the inferior vena cava may produce chest pain. Cognitive Level: Comprehension 592

51. A 50-year-old female is diagnosed with primary thrombocythemia. A nurse would expect the blood smear to reveal _____ platelets. a. Defective b. Fragmented c. Consumed d. Overproduced

ANS: D Thrombocythemia is characterized by a platelet count more than 400,000 platelets/mm3 of blood and is symptomatic when the count exceeds 1,000,000/ mm3, at which time the risk for intravascular clotting (thrombosis) is high. Thrombocythemia is characterized by a high platelet count, not a defective one. Thrombocythemia is characterized by a high platelet count, not a fragmented one. Thrombocythemia is characterized by a high platelet count, not a consumed one.

25. A 50-year-old obese male with hypertension and coronary artery disease visits a nutritionist for food counseling. He has an elevated level of low-density lipoprotein (LDL) and a low level of high-density lipoprotein (HDL). Which of the following should the nurse advise him to avoid? a. Monounsaturated fats b. Polyunsaturated fats c. Saturated fats d. Trans fats

ANS: D Trans fats are primarily found in artificially solidified (hydrogenated) oils (e.g., margarine and vegetable shortening). By becoming more solid, they lose essential fatty acids (EFAs). They can raise LDL and lower HDL levels. Trans fats, not monounsaturated fats, are primarily found in artificially solidified (hydrogenated) oils (e.g., margarine and vegetable shortening). By becoming more solid, they lose EFAs. They can raise LDL and lower HDL levels. Trans fats, not polyunsaturated fats, are primarily found in artificially solidified (hydrogenated) oils (e.g., margarine and vegetable shortening). By becoming more solid, they lose EFAs. They can raise LDL and lower HDL levels. Trans fats, not saturated fats, are primarily found in artificially solidified (hydrogenated) oils (e.g., margarine and vegetable shortening). By becoming more solid, they lose EFAs. They can raise LDL and lower HDL levels. Cognitive Level: Comprehension 598

14. Several prisoners are experiencing symptoms of tuberculosis. A tuberculin reaction test was ordered. This test is an example of type _____ hypersensitivity. a. I b. II c. III d. IV

ANS: D Tuberculosis testing is an example of type IV hypersensitivity. Tuberculosis testing is an example of type IV hypersensitivity, not type I. Tuberculosis testing is an example of type IV hypersensitivity, not type II. Tuberculosis testing is an example of type IV hypersensitivity, not type III. Cognitive Level: Comprehension 196

9. Which information would indicate more teaching is needed regarding hypersensitivity reactions? Type _______ hypersensitivity reactions involve an antibody response. a. I b. II c. III d. IV

ANS: D Type IV reactions are mediated by T lymphocytes and do not involve antibodies. Type I reactions involve antibody responses. Type II reactions involve antibody responses. Type III reactions involve antibody responses. Cognitive Level: Comprehension 195

21. A 3-month-old female develops colicky pain, abdominal distention, and diarrhea after drinking cows milk. The best explanation for her symptoms is: a. Deficiency of bile that stimulates digestive secretions and bowel motility b. Excess of amylase, which increases the breakdown of starch and causes an osmotic diarrhea c. Overgrowth of bacteria from undigested fat molecules, which leads to gas formation and decreased bowel motility d. Excess of undigested lactose in her digestive tract, resulting in increased fluid movement into the digestive lumen and increased bowel motility

ANS: D Undigested lactose increases the osmotic gradient in the intestine, causing irritation and osmotic diarrhea. The child is experiencing lactose intolerance, not bile deficiency. This child is experiencing lactose intolerance, not an excess of amylase. The child is experiencing lactose intolerance, not in infectious process. Cognitive Level: Comprehension 908

25. A 16-year-old is bitten by a rabid dog while jogging in the park. Upon admission to the emergency department, the nurse will administer which of the following to help prevent infection? a. Cytotoxic T cells b. Helper T cells c. Macrophages d. Immune globulin

ANS: D Unvaccinated individuals who are exposed to particular infectious agents often will be given immune globulins, which are prepared from individuals who already have antibodies against that particular pathogen. The patient would be given immune globulin, not cytotoxic T cells. The patient would be given immune globulin, not helper T cells. The patient would be given immune globulin, not macrophages. Cognitive Level: Comprehension 145

20. A nurse is preparing to teach about the loop of Henle. Which information should be included? The descending segment of the loop of Henle primarily allows for: a. Sodium secretion b. Potassium secretion c. Hydrogen ion reabsorption d. Water reabsorption

ANS: D Water reabsorption occurs in the descending segment of the loop of Henle. Water reabsorption occurs in the loop of Henle; sodium is not affected. Water reabsorption occurs in the loop of Henle; potassium is not affected. Water reabsorption occurs in the loop of Henle; hydrogen ion reabsorption does not occur. Cognitive Level: Comprehension 734

6. A 25-year-old female has a heavy menses during which she loses a profuse amount of blood. Which of the following adaptations should the nurse expect? a. Movement of fluid into the cell b. Decreased cardiac output c. Decreased oxygen release from hemoglobin d. Peripheral vasoconstriction

ANS: D When the anemia is severe or acute in onset (e.g., hemorrhage), the initial compensatory mechanism is peripheral blood vessel constriction, diverting blood flow to essential vital organs. Fluid moves into the vascular space, not the cell. Blood volume increases; thus, cardiac output increases. There is an increase in hemoglobin release of oxygen. Cognitive Level: Comprehension 501

26. A newborn baby has a clotting disorder that results in her body being unable to produce thrombin. Which of the following phases of platelet degranulation would be impossible? a. Subendothelial exposure b. Adhesion c. Activation d. Aggregation

ANS: D Without thrombin, aggregation cannot occur. Without thrombin, aggregation, not subendothelial exposure, cannot occur. Without thrombin, aggregation, not adhesion, cannot occur. Without thrombin, aggregation, not activation, cannot occur. REF: pp. 491-492

14. A 5-year-old male presents to the ER with delirium and sunken eyes. After diagnosing him with severe dehydration, the primary care provider orders fluid replacement. The nurse administers a hypertonic intravenous solution. Which of the following would be expected? a. Symptoms subside quickly b. Increased ICF volume c. Decreased ECF volume d. Intracellular dehydration

ANS: D A hypertonic solution would cause fluid to move into the extracellular space, leading to intracellular dehydration. With this solution, his symptoms will not subside quickly because his cells will lose fluid. His intracellular volume will decrease, not increase. His extracellular volume will increase, not decrease. Cognitive Level: Comprehension 106

31. A 50-year-old male was recently diagnosed with Huntington disease. Transmission of this disease is associated with: a. Penetrance b. Recurrence risk c. Expressivity d. Delayed age of onset

ANS: D A key feature of Huntington disease is its delayed age of onset such that symptoms are not seen until 40 years of age or later. The penetrance of a trait is the percentage of individuals with a specific genotype who also exhibit the expected phenotype. Recurrence risk is the percentage of family members that will inherit the disease. Expressivity is the extent of variation in phenotype associated with a particular genotype.

26. A normal male and a female carrier for red-green color blindness mate. Given that red-green color blindness is an X-linked recessive trait, what is the likelihood of their children being affected? a. 25% b. 50% c. Females most affected; no males affected d. Males most affected; no females affected

ANS: D Because a single copy of an X-linked recessive gene will cause disease in a male, whereas two copies are required for disease expression in females, more males are affected by X-linked recessive diseases than are females. Males are more often affected at a greater than 25% rate. Males are more often affected at a greater than 50% rate. Males are most often affected.

6. A 24-year-old female presents with excessive menstrual bleeding. The physician identified endometrial changes that are due to hormonal imbalances. These cellular changes would be referred to as: a. Dysplasia b. Pathologic dysplasia c. Hyperplasia d. Pathologic hyperplasia

ANS: D Because the changes are due to an imbalance, they would be considered pathologic hyperplasia. The endometrial changes were not abnormal in size and shape; thus, it is not dysplasia. The description of alteration in hormone levels is pathologic. Hyperplasia is not as accurate as pathologic because there is an imbalance of hormone levels.

7. When planning care for a dehydrated patient, the nurse remembers the principle of water balance is closely related to _____ balance. a. Potassium b. Chloride c. Bicarbonate d. Sodium

ANS: D Because water follows the osmotic gradients established by changes in salt concentration, water balance is tied to sodium balance. Because water follows the osmotic gradients established by changes in salt concentration, water balance is tied to sodium balance, not potassium. Because water follows the osmotic gradients established by changes in salt concentration, water balance is tied to sodium balance, not chloride. Because water follows the osmotic gradients established by changes in salt concentration, water balance is tied to sodium balance, not bicarbonate. Cognitive Level: Comprehension 103

1. A nurse is reviewing lab reports. The nurse recalls blood plasma is located in which of the following fluid compartments? a. Intracellular fluid (ICF) b. Extracellular fluid (ECF) c. Interstitial fluid d. Intravascular fluid

ANS: D Blood plasma is the intravascular fluid. ICF is fluid in the cells. ECF is all the fluid outside the cells. Interstitial fluid is fluid between the cells and outside the blood vessels.

18. When a patient asks what causes cystic fibrosis, how should the nurse respond? Cystic fibrosis is caused by an _____ gene. a. X-linked dominant b. X-linked recessive c. Autosomal dominant d. Autosomal recessive

ANS: D Cystic fibrosis is an autosomal recessive disorder. Cystic fibrosis is not X linked, but autosomal. Cystic fibrosis is not X linked, but recessive. Cystic fibrosis is not dominant.

28. When discussing DNA replication, which enzyme is most important? a. RNA polymerase b. Transfer RNA c. Messenger RNA d. DNA polymerase

ANS: D DNA polymerase is the primary enzyme involved in replication. It adds bases to the new DNA strand and performs proofreading functions. It is DNA polymerase, not RNA polymerase, that is most important for DNA replication. It is DNA polymerase, not transfer RNA, that is most important for DNA replication. It is DNA polymerase, not messenger RNA, that is most important for DNA replication.

20. A patient, age 9, is admitted to a pediatric unit with Duchenne muscular dystrophy. When planning care the nurse recalls the patient inherited this condition through a _____ trait. a. Sex-linked dominant b. Sex-influenced c. Sex-limited d. Sex-linked recessive

ANS: D Duchenne muscular dystrophy is a relatively common X-linked recessive disorder. Duchenne is a recessive trait, not dominant. Duchenne is a sex-linked, not sex-influenced, trait. Duchenne is a sex-linked, not sex-limited, trait.

27. A 50-year-old female became infected with Clostridium bacteria and died a week later. Examination of her red blood cells revealed lysis of membranes. Which of the following was the most likely cause of her death? a. Fat necrosis b. Wet gangrene c. Gangrenous necrosis d. Gas gangrene

ANS: D Gas gangrene is a special type of gangrene caused by infection of injured tissue by one of many species of Clostridium. Fat necrosis is cellular dissolution caused by powerful enzymes, called lipases, that occur in the breast, pancreas, and other abdominal structures. It is not due to infection. Wet gangrene develops when neutrophils invade the site, causing liquefactive necrosis. It is not due to infection. Gangrenous necrosis is due to death of tissue and results from severe hypoxic injury, not infection.

23. A 23-year-old male develops a black eye following a fight. When the aide asks the nurse why this occurred, the nurses best response is that the bruising is due to an accumulation of: a. Transferrin b. Bilirubin c. Albumin d. Hemosiderin

ANS: D Hemosiderin is responsible for the color changes in a black eye. Transferrin is a transport protein responsible for iron transport. Bilirubin is the normal, yellow-to-green pigment of bile derived from the porphyrin structure of hemoglobin. Albumin is the protein in the serum, responsible for cellular integrity.

20. Which of the following patients is most prone to hypochloremia? A patient with: a. Hypernatremia b. Hypokalemia c. Hypercalcemia d. Increased bicarbonate intake

ANS: D Hypochloremia is the result of elevated bicarbonate concentration, as occurs in metabolic alkalosis. Hypochloremia is the result of hyponatremia, not hypernatremia. Hypochloremia is the result of hyponatremia, not hypokalemia. Hypochloremia is the result of hyponatremia, not hypercalcemia. Cognitive Level: Comprehension 106

5. A biologist is explaining how RNA directs the synthesis of protein. Which process is the biologist describing? a. Termination b. Transcription c. Translocation d. Translation

ANS: D In translation, RNA directs the synthesis of a polypeptide, interacting with transfer RNA (tRNA), a cloverleaf-shaped strand of about 80 nucleotides. At a termination signal, translation and polypeptide formation cease. This does not involve synthesis of protein. Transcription is the process by which DNA specifies a sequence of messenger RNA (mRNA). Translocation is the interchange of genetic material between nonhomologous chromosomes.

6. An experiment was designed to test the effects of the Starling forces on fluid movement. Which of the following alterations would result in fluid moving into the interstitial space? a. Increased capillary oncotic pressure b. Increased interstitial hydrostatic pressure c. Decreased capillary hydrostatic pressure d. Increased interstitial oncotic pressure

ANS: D Increased interstitial oncotic pressure would attract water from the capillary into the interstitial space. Increased capillary oncotic pressure would attract water from the interstitial space back into the capillary. Increased interstitial hydrostatic pressure would attract movement of water from the interstitial spaces into the capillary. Decreased capillary hydrostatic pressure would move water into the capillaries. Cognitive Level: Comprehension 101

15. A common pathway of irreversible cell injury involves increased intracellular: a. Sodium b. Potassium c. Magnesium d. Calcium

ANS: D Irreversible cellular injury is due to increased intracellular calcium and loss of calcium steady state. It is calcium, not sodium, that is involved in cellular injury. It is calcium, not potassium, that is involved in cellular injury. It is calcium, not magnesium, that is involved in cellular injury.

10. A patient has a heart attack that leads to progressive cell injury that causes cell death with severe cell swelling and breakdown of organelles. What term would the nurse use to define this process? a. Adaptation b. Pathologic calcification c. Apoptosis d. Necrosis

ANS: D Necrosis is the sum of cellular changes after local cell death. Cellular adaptation is a reversible, structural, or functional response to both normal or physiologic conditions and adverse or pathologic conditions. Calcification is an accumulation of calcium salts. Apoptosis is an active process of cellular self-destruction.

23. A 42-year-old female presents to her primary care provider reporting muscle weakness and cardiac abnormalities. Laboratory tests indicate that she is hypokalemic. Which of the following could be the cause of her condition? a. Respiratory acidosis b. Constipation c. Hypoglycemia d. Primary hyperaldosteronism

ANS: D Primary hyperaldosteronism, with excessive secretion of aldosterone from an adrenal adenoma (tumor) also causes potassium wasting. Acidosis is related to hyperkalemia, not hypokalemia. Constipation can occur with hypokalemia but does not cause it. Hypoglycemia is not related to muscle weakness. Cognitive Level: Comprehension 108

30. Which patient should the nurse assess for both hyperkalemia and metabolic acidosis? A patient diagnosed with: a. Diabetes insipidus b. Pulmonary disorders c. Cushing syndrome d. Renal failure

ANS: D Renal failure is associated with hyperkalemia and metabolic acidosis. Diabetes insipidus results in hypernatremia. Pulmonary disorders are a cause of respiratory acidosis or alkalosis but do not affect hyperkalemia. Cushing syndrome results in hypernatremia. Cognitive Level: Comprehension 113

31. When a nurse observes muscle stiffening occurring within 6 to 14 hours after death, the nurse should document this finding as _____ present. a. Livor mortis b. Gangrene c. Algor mortis d. Rigor mortis

ANS: D Rigor mortis occurs within 6 hours after death and is evidence by muscle stiffening. Livor mortis is a purple discoloration . Gangrene refers to death of tissue and results from severe hypoxic injury and does not lead to stiffening. Algor mortis is postmortem reduction of body temperature.

19. A 15-year-old female presents to the ER following a physical assault. She has internal damage to the neck with deep bruising. X-ray reveals fractures of the hyoid bone and tracheal and cricoid cartilage. Which of the following most likely caused her injuries? a. Chemical asphyxiation b. Choking asphyxiation c. Ligature strangulation d. Manual strangulation

ANS: D Squeezing of the neck as with strangulation would fraction the hyoid bone. Chemical asphyxiation would lead to breathing problems but would not result in fracture. Choking asphyxiation would lead to swelling of tissues but would not result in fracture. In ligature strangulation, the mark on the neck is horizontal without the inverted V pattern seen in hangings. It would not lead to fracture.

2. Which of the following mutations have the most significant effect on protein synthesis? a. Base pair substitutions b. Silent mutations c. Intron mutations d. Frameshift mutations

ANS: D The frameshift mutation involves the insertion or deletion of one or more base pairs of the DNA molecule. They alter the amino acid sequence. The base pair substitution is a type of mutation in which one base pair replaces another. Silent mutations do not change amino acids or protein synthesis.

12. A patient wants to know the risk factors for Down syndrome. What is the nurses best response? a. Fetal exposure to mutagens in the uterus b. Increased paternal age c. Family history of Down syndrome d. Pregnancy in women over age 35

ANS: D The primary risk for Down syndrome is pregnancy in women over 35. Down syndrome is a trisomy and not due to fetal exposure. Paternal age is not a risk factor in Down syndrome. Down syndrome is a chromosomal abnormality and is not related to family history.

2. A 35-year-old male weighs 70 kg. Approximately how much of this weight is ICF? a. 5 L b. 10 L c. 28 L d. 42 L

ANS: D The total volume of body water for a 70-kg person is about 42 L. 5 L is incorrect because a 70-kg person has about 42 L of body water. 10 L is incorrect because a 70-kg person has about 42 L of body water. 28 L is incorrect because a 70-kg person has about 42 L of body water.

30. A 5-year-old male presents with mental retardation and is diagnosed with fragile X syndrome. When the parents ask what caused this, how should the geneticist respond? This was most probably caused from: a. Translocation b. Inversion c. Nondisjunction d. Duplication at fragile sites

ANS: D Unaffected transmitting males have been shown to have more than about 50 repeated DNA sequences near the beginning of the fragile X gene. Translocation effects do not result in mental retardation. Inversions have no physical effects. Fragile X is not related to nondisjunction.

8. A 40-year-old female is diagnosed with cervical cancer after a Pap smear. Which of the following cellular changes would the nurse most likely see on the report? a. Metaplasia b. Atrophy c. Hypertrophy d. Dysplasia

ANS: D When cervical cells become cancerous, they have undergone dysplasia, a change in the size and shape of cells. The cells have changed; they have not been replaced, so metaplasia is not the correct choice. The cells have not decreased in size; therefore, atrophy is incorrect. The cells have not increased in size; therefore, they have not hypertrophied.

. Researchers now believe that RA is: a. Curable with antiviral agents b. An autoimmune disease c. A complication of rheumatic fever d. Related superficial joint injury

B

. When a nurse is assessing the physical features of individuals with Cushing syndrome, these findings will include: a. Weight loss and muscle wasting b. Truncal obesity and moon face c. Pallor and swollen tongue d. Depigmented skin and eyelid lag

B

. When a patient asks about target cell receptors, which is the nurses best response? Target cell receptors for most water-soluble hormones are located in the: a. Cytosol b. Cell membrane c. Endoplasmic reticulum d. Nucleus

B

. When a patient asks how bad the injuries will be from a spinal injury, what is the nurses best response? It is difficult to know the full extent of the injury because of: a. Incomplete transection of the spinal cord b. Swelling within the spinal cord c. Necrosis of the spinal cord d. Free radical injury and scarring of the glial cells

B

. When a staff member asks the nurse what causes the chronic complications of DM such as microvascular and macrovascular disease, how should the nurse respond? These complications are primarily related to: a. Pancreatic changes b. Hyperglycemia c. Ketone toxicity d. Hyperinsulinemia

B

. When a staff member asks the nurse which gland secretes ADH and oxytocin, how should the nurse respond? a. Anterior pituitary b. Posterior pituitary c. Hypothalamus d. Pineal gland

B

. Which information indicates the nurse has a good understanding of osteoblasts? Once their initial function is complete, osteoblasts become: a. Osteoclasts b. Osteocytes c. Chondroblasts d. Osteoids

B

. Which principle should the nurse include while planning care for a patient with an ADH problem? ADH release from the posterior pituitary is stimulated by: a. Low blood pressure sensed by baroreceptors in the kidneys b. High serum osmolarity sensed by osmoreceptors in the hypothalamus c. Low osmolality sensed by osmoreceptors in the kidneys d. High concentration of potassium sensed by chemoreceptors in the carotid body

B

10. Prenatal diagnosis methods include the use of ultrasonography for identifying ________ abnormalities. A) cytogenic B) skeletal C) chromosomal D) a-fetoprotein

B

14. A patient with a diagnosis of aplastic anemia has undergone allogenic bone marrow transplantation. Which of the following signs and symptoms would most clearly suggest the existence of graft-versus-host disease (GVHD)? A) Shortness of breath, audible crackles, and decreasing PaO2 B) Presence of a pruritic rash that has begun to slough off C) Development of metabolic acidosis D) Diaphoresis, fever, and anxiety

B

16. Genetic testing has revealed that a male infant has been born with an extra X chromosome. What are the most likely implications of this finding? A) The child is unlikely to survive infancy B) The child is likely to have no manifestations of this chromosomal abnormality C) The child will have significant neurological and cognitive defects D) The child will be unable to reproduce

B

18. A patient was diagnosed as HIV positive several years ago. Which of the following blood tests is most clinically useful for determining the stage and severity of her disease? A) Plasma levels B) CD4+ cell counts C) Viral load D) White blood cell count with differential

B

20. Shortly after being diagnosed with HIV, a patient has begun highly active antiretroviral therapy (HAART). What is the primary goal of the patients drug regimen? A) To limit the latent period of HIV B) To slow the progression of the disease C) To minimize opportunities for transmission D) To prevent seroconversion

B

20. Ultrasonography is most likely to detect which of the following fetal abnormalities? A) Neural tube defects B) Skeletal abnormalities C) Chromosomal defects D) Single-gene disorders

B

3. Mismatched blood transfusion reaction with hemolysis of blood cells is an example of type II, _____ mediated hypersensitivity reaction. A) T-cell B) antibody C) leukotriene D) complement

B

5. The mechanism by which humans recognize self-cells from non-self (antigens)-cells is _________. A) autoimmunity B) self-tolerance C) non-self anergy D) immunocompatibility

B

6. Organ rejection is a complication of organ transplantation caused by recipient immune cells: A) destroying the host T cells. B) attack on the donor cells. C) combining with grafts HLA. D) being recognized as foreign.

B

7. The leading cause of death for people with HIV is opportunistic ____________. A) leukemia B) tuberculosis C) pneumonia D) toxoplasmosis

B

9. Fetal alcohol syndrome (FAS) is unlike other teratogens in that the harmful effects on the fetus: A) directly result in liver damage. B) extend throughout the pregnancy. C) is most noticeable in adulthood. D) cause death in early childhood.

B

9. The window period of HIV infection refers to the period of time between infection and: A) transmission. B) seroconversion. C) initial symptoms. D) antibody screening.

B

A 10-year-old female was brought to the ER following a sudden onset of convulsions. The primary care provider thinks that she experienced an explosive, disorderly discharge of cerebral neurons referred to as: a. Reflex b. Seizure c. Inattentiveness d. Brain death

B

A 10-year-old male was climbing on a house and fell. He suffered a severe brain injury. His Glasgow Coma Scale (GCS) was 5 initially and 7 after 1 day. He remained unconscious for 2 weeks, then was confused and suffered from anterograde amnesia. Which of the following is he most likely experiencing? a. Mild diffuse brain injury b. Moderate diffuse brain injury c. Severe diffuse brain injury d. Postconcussive syndrome

B

A 15-year-old male is brought to the ER for treatment of injuries received in a motor vehicle accident. An MRI reveals spinal cord injury, and his body temperature fluctuates markedly. The most accurate explanation of this phenomenon is that: a. He developed pneumonia. b. His sympathetic nervous system has been damaged and thermal control disturbed. c. He has a brain injury. d. He has septicemia from an unknown source

B

A 15-year-old male suffered diffuse brain injury after wrecking an all-terrain vehicle. He had momentary confusion and retrograde amnesia after 5 to 10 minutes. His injury could be categorized as: a. Grade I b. Grade II c. Grade III d. A mild concussion

B

A 16-year-old male fell off the bed of a pickup truck and hit his forehead on the road. He now has resistance to passive movement that varies proportionally with the force applied. He is most likely suffering from: a. Spasticity b. Gegenhalten c. Rigidity d. Dystonia

B

A 16-year-old male took a recreational drug that altered his level of arousal. Physical exam revealed a negative Babinski sign, equal and reactive pupils, and roving eye movements. Which of the following diagnosis will the nurse most likely see on the chart? a. Psychogenic arousal alteration b. Metabolically induced coma c. Structurally induced coma d. Structural arousal alteration

B

A 19-year-old female with type 1 DM was admitted to the hospital with altered consciousness and the following lab values: serum glucose 500 mg/dl (high) and serum K+ 2 (low). Her parents state that she has been sick with the flu for a week. The diagnosis is hyperosmolar hyperglycemia nonketotic syndrome (HHNKS). What relationship do these values have with her insulin deficiency? a. Increased glucose utilization causes the shift of fluid from the intravascular to the intracellular space. b. Decreased insulin causes hyperglycemia and osmotic diuresis. c. Increased glucose and fatty acid metabolism stimulates renal diuresis and electrolyte loss. d. Increased insulin use results in protein catabolism, tissue wasting, and electrolyte loss.

B

A 20-year-old male suffers a severe closed head injury in a motor vehicle accident. He remains in a vegetative state (VS) 1 month after the accident. Which of the following structures is most likely keeping him in a vegetative state? a. Cerebral cortex b. Brainstem c. Spinal cord d. Cerebellum

B

A 22-year-old female has a low level of TSH. What condition does the nurse expect the patient is experiencing? a. Primary hypothyroidism b. Secondary hypothyroidism c. Autoimmune hypothyroidism d. Atypical hypothyroidism

B

A 22-year-old male is admitted to the intensive care unit with a closed head injury sustained in a motorcycle accident. The injury has caused severe damage to the posterior pituitary. Which of the following complications should the nurse anticipate? a. Dilutional hyponatremia b. Dehydration from polyuria c. Cardiac arrest from hyperkalemia d. Metabolic acidosis

B

A 23-year-old female begins having problems with tiredness, weakness, and visual changes. Her diagnosis is multiple sclerosis (MS). What is occurring in the patients body? a. Depletion of dopamine in the central nervous system (CNS) b. Demyelination of nerve fibers in the CNS c. The development of neurofibril webs in the CNS d. Reduced amounts of acetylcholine at the neuromuscular junction

B

A 25-year-old female presents to her primary care provider reporting fever, headache, nuchal rigidity, and decreased consciousness. She was previously treated for sinusitis. Which of the following is the most likely diagnosis? a. Aseptic meningitis b. Bacterial meningitis c. Fungal meningitis d. Nonpurulent meningitis

B

A 25-year-old male died from a gunshot wound to the heart. Upon autopsy, abnormalities in the media of the arterial wall and degenerative changes were detected. Which of the following most likely accompanied this finding? a. Fusiform aneurysm b. Saccular aneurysm c. Arteriovenous malformation d. Thrombotic stroke

B

A 30-year-old female with Graves disease is admitted to a hospital unit for the surgical removal of her thyroid gland. During the postoperative period, the nurse notes that the patients serum calcium is low. The nurse should observe the patient for which of the following signs/symptoms? a. Muscle weakness and constipation b. Laryngeal spasms and hyperreflexia c. Abdominal pain and fever d. Anorexia, nausea, and vomiting

B

A 30-year-old male is diagnosed with a hormone-secreting tumor of the pancreas alpha cells. Which of the following would the nurse expect to be most likely increased in this patient? a. Amylin b. Glucagon c. Insulin d. Somatostatin

B

A 35-year-old female suffers a broken clavicle following a motor vehicle accident. X-ray reveals that the bone surfaces in the joint partially lost contact with each other. This condition is called: a. Dislocation b. Subluxation c. Distortion d. Nonunion

B

A 39-year-old male suffers a severe brain injury when he falls off a building while working. CT scan reveals that he has a basilar skull fracture. Based upon his injuries, what major complication should the nurse observe for in this patient? a. Hematoma formation b. Meningeal infection c. Increased intracranial pressure (ICP) d. Cognitive deficits

B

A 40-year-old male complains of uncontrolled excessive movement and progressive dysfunction of intellectual and thought processes. He is experiencing movement problems that begin in the face and arms that eventually affect the entire body. The most likely diagnosis is: a. Tardive dyskinesia b. Huntington disease c. Hypokinesia d. Alzheimer disease

B

A 48-year-old female presents at the ER reporting an acute severe headache, nausea, photophobia, and nuchal rigidity. What does the nurse suspect caused these signs and symptoms? a. Diffuse brain injury b. Subarachnoid hemorrhage c. Epidural hematoma d. Classic concussion

B

When a staff member asks where costal cartilage is located, how should the nurse respond? Between the: a. Vertebrae b. Ribs and sternum c. Sutures of the skull d. Facial bones

B

A 65-year-old Hispanic female is admitted to the hospital with a pathologic, compound, transverse fracture of the femur. Which of the following statements best describes this type of fracture? a. The fracture line is parallel to the bone. b. The fracture line is straight across the bone. c. The fracture line is perpendicular to the bone. d. The fracture line is vertical to the shaft of the bone.

B

A 65-year-old male recently suffered a cerebral vascular accident. He is now unable to recognize and identify objects by touch because of injury to the sensory cortex. How should the nurse document this finding? a. Hypomimesis b. Agnosia c. Dysphasia d. Echolalia

B

A 69-year-old male with a history of alcohol abuse presents to the emergency room (ER) after a month-long episode of headaches and confusion. Based on his alcoholism, a likely cause of his neurologic symptoms is: a. Concussion b. Chronic subdural hematoma c. Epidural hematoma d. Subacute subdural hematoma

B

A 70-year-old female presents with a hip fracture. She is diagnosed with osteoporosis. One a. Increased androgen levels b. Decreased estrogen levels c. Strenuous exercise d. Excessive dietary calcium

B

A 75-year-old male experienced a lacunar stroke. When looking through the history of the patients chart, which of the following would the nurse expect to find? a. An embolus b. Hypertension c. A hemorrhage d. An aneurysm

B

A clinician is preparing to discuss hydroxyapatite. Which information should be included? Hydroxyapatite in the bone matrix contains which of the following molecules in its crystalline structure? a. Calcium and magnesium b. Calcium and phosphate c. Phosphate and magnesium d. Magnesium and hydrogen

B

A diet deficient in calories and protein causes marasmus, which is characterized by: a. discolored hair. b. bradycardia. c. enlarged liver. d. pitting edema.

B

A disorder similar to osteomalacia that occurs in growing bones of children is termed: a. Paget disease b. Rickets c. Osteomyelitis d. Osteosarcoma

B

A male patient complains of tiring easily, has difficulty rising from a sitting position, and cannot stand on his toes. The nurse would expect a diagnosis of: a. Parkinson disease b. Hypotonia c. Huntington disease d. Paresis

B

A neurologist is teaching about encephalitis. Which information should the neurologist include? Most causes of encephalitis are: a. Bacterial b. Viral c. Fungal d. Toxoid

B

A nurse is reviewing lab results for glycosylated hemoglobin (hemoglobin A1c) levels. A nurse recalls the purpose of this test is to: a. Measure fasting glucose levels. b. Monitor long-term serum glucose control. c. Detect acute complications of diabetes. d. Check for hyperlipidemia.

B

A nurse is teaching a patient about insulin. Which information should the nurse include? Insulin is primarily regulated by: a. Metabolic rate b. Serum glucose levels c. Prostaglandins d. Enzyme activation

B

A nurse recalls prolactin-inhibiting factors target tissue is the: a. Hypothalamus b. Anterior pituitary c. Mammary glands d. Posterior pituitary

B

A nurse recalls the elbow joint is an example of a(n): a. Amphiarthrosis b. Diarthrosis c. Synarthrosis d. Symphysis

B

A patient has researched muscles on the internet. Which information indicates the patient has a good understanding? The molecule that prevents a muscle contraction from occurring when the muscle is at rest is: a. Calcium b. Troponin-tropomyosin c. Actin d. Myosin

B

A patient is admitted with amyotrophic lateral sclerosis (ALS). Which classic assessment findings will support the diagnosis of ALS? a. Progressive dementia b. Muscle weakness and atrophy c. Severe paresthesias d. Autonomic dysfunctions

B

An 11-year-old male is newly diagnosed with type 1 DM. Which classic symptoms should the nurse assess the patient for? a. Recurrent infections, visual changes, fatigue, and paresthesias b. Polydipsia, polyuria, polyphagia, and weight loss c. Vomiting; abdominal pain; sweet, fruity breath; dehydration; and Kussmaul breathing d. Weakness, vomiting, hypotension, and mental confusion

B

An aide asks the nurse what is the most common cause of elevated levels of antidiuretic hormone (ADH) secretion. How should the nurse respond? a. Autoimmune disease b. Cancer c. Pregnancy d. Heart failure

B

An infant is diagnosed with noncommunicating hydrocephalus. What is an immediate priority concern for this patient? a. Metabolic edema b. Interstitial edema c. Vasogenic edema d. Ischemic edema

B

An isolated muscle is believed to be a type II fiber. Which of the following should be observed by the nurse when the muscle is stimulated? a. The muscle has slow contraction speed. b. The muscle is fatigued easily. c. The muscle has a profuse capillary supply. d. The muscle has no fascia.

B

Body weight should be used in combination with other measurements to establish if a person is underweight or overweight. Obesity is indicated by: a. female body fat of 20% and 30%. b. body mass index (BMI) of 30 to 40. c. relative body weight of 70% to 100%. d. abdominal fat/ hip ratio of 0.8 to 1.0.

B

Chronic inflammation as a result of excess adipose tissue is implicated in the etiology of which of the following health problems? a. Osteoporosis b. Type 2 diabetes c. Rheumatoid arthritis d. Systemic lupus erythematosus (SLE)

B

If a patient had a problem with the hypothalamus, which of the following hormones would be affected? a. ACTH b. Oxytocin c. ADH d. TSH

B

If a patients posterior pituitary is removed, which hormone would the nurse expect to decrease? a. PRF b. ADH c. ACTH d. GH

B

The adipocytes in adipose tissue not only serve as a storage sites, they also: a. produce linoleic fatty acid. b. synthesize triglycerides. c. increase glucagon release. d. degrade fat-soluble vitamins.

B

The cause of Paget disease is: b. Unknown at present c. Excess vitamin A d. Osteogenic sarcoma

B

Which principle should the nurse remember while planning care for a patient with spinal shock? Spinal shock is characterized by: a. Loss of voluntary motor function with preservation of reflexes b. Cessation of spinal cord function below the lesion c. Loss of spinal cord function at the level of the lesion only d. Temporary loss of spinal cord function above the lesion

B

While checking the lab results for a patient with Graves disease, the nurse would check the T3 level to be abnormally: a. Low b. High c. Variable d. Absent

B

While planning care for a patient from general anesthesia, which principle should the nurse remember? A side effect of some general anesthetic agents is _____ diabetes insipidus. a. Neurogenic b. Nephrogenic c. Psychogenic d. Allogenic

B

While planning care for a patient with hypothyroidism, which principle should the nurse remember? The basal metabolic rate is unusually _____ with hypothyroidism. a. High b. Low c. Steady d. Variable

B

a 19-year-old male college student reports to his primary care provider that he cannot stay awake in class regardless of how much sleep he gets. a drug that stimulates which of the following areas would best treat his problem? a. corpora quadrigemina b. reticular activating system c. cerebellum d. hypothalamus

B

a 40-year-old male suffers from head trauma that affects cranial nerve I. which of the following symptoms would the nurse expect? a. visual disturbances b. loss of sense of smell c. loss of ability to taste d. hearing disturbances

B

a neurologist is teaching about the region responsible for motor aspects of speech. which area is the neurologist discussing? a. wernicke area b. broca area c. primary speech area d. insula

B

a nurse recalls characteristics of upper motor neurons include: a. directly innervating muscles b. influencing and modifying spinal reflex arcs c. cell bodies located in the gray matter of the spinal cord d. dendrite processes extending out of the CNS

B

if a neuron's membrane potential is held as close to the threshold potential by excitatory postsynaptic potentials (EPSPs), the neuron is said to be: a. hyperpolarized b. facilitated c. integrated d. inhibited

B

when a nurse is teaching about the transverse fiber tract that connects the two cerebral hemispheres, what term should the nurse use? a. peduncle b. corpus callosum c. basal ganglia d. pons

B

when a presynaptic neuron is stimulated in a patient's body by an electrical current, neurotransmitters are released from the: a. synapse b. synaptic bouton c. synaptic cleft d. receptor

B

. Which nutrient would the nurse encourage the patient to consume for thyroid hormone synthesis? a. Zinc b. Sodium c. Iodine d. Calcium

C

. Which of the following should the nurse include when discussing the different types of cartilaginous joints? a. Sutures and gomphosis b. Syndesmosis and gomphosis c. Symphysis and synchondrosis d. Gomphosis and synchondrosis

C

. Which statement indicates the nurse has a good understanding of joints? The joints between the teeth and the maxilla or mandible are examples of a: a. Symphysis b. Diarthrosis c. Gomphosis d. Syndesmosis

C

17. Which of the following variables determine the extent of teratogenic drug effects? Select all that apply. A) Maternal health history B) Molecular weight of the drug C) Stage of pregnancy when the drug was taken D) Duration of drug exposure E) Fetal blood type

B, C, D

. A 13-year-old male who uses insulin to control his type 1 diabetes experiences hunger, lightheadedness, tachycardia, pallor, headache, and confusion during gym class. The most probable cause of these symptoms is: a. Hyperglycemia resulting from incorrect insulin administration b. Dawn phenomenon caused by eating a snack before gym class c. Hypoglycemia caused by increased exercise d. Somogyi effect caused by insulin sensitivity

C

. When a patient asks what the fuel for skeletal muscle contraction is, how should the nurse respond? The main energy source or fuel for skeletal muscle contraction is: a. Calcium b. Lactic acid c. Adenosine triphosphate (ATP) d. Actin

C

. When a patient wants to know what most commonly causes hypoparathyroidism, how should the nurse reply? It is most commonly caused by: a. Pituitary hyposecretion b. Parathyroid adenoma c. Parathyroid gland injury d. Hypothalamic inactivity

C

A 45-year-old female presents to the emergency room (ER) reporting excessive vomiting. A CT scan of the brain reveals a mass in the: a. Skull fractures b. Thalamus c. Medulla oblongata d. Frontal lobe

C

11. Contact with poison ivy has resulted in intense pruritus, erythema, and weeping on a patients forearm. Which of the following processes resulted in the patients signs and symptoms? A) IgE-mediated mast cell degranulation B) Formation of antigen-antibody complexes C) Cytokine release by sensitized T cells D) Formation of antibodies against cell surface antigens

C

13. A patient with a diagnosis of cirrhosis has experienced an acute rejection of a donor liver. Which of the following cells is central to the rejection of the patients transplanted organ? A) Natural killer cells B) Mast cells C) T cells D) Neutrophils

C

13. The parents of a newborn infant are relieved that their baby was born healthy, with the exception of a cleft lip that will be surgically corrected in 10 or 12 weeks. Which of the nurses following statements to the parents best conveys the probable cause of the infants cleft lip? A) Though you are both healthy, you likely both carry the gene for a cleft lip. B) Provided one of you had the gene for a cleft lip, your baby likely faced a 50/50 chance of having one. C) Your childs cleft lip likely results from the interplay between environment and genes. D) A cleft lip can sometimes result from taking prescription drugs, even when theyre taken as ordered.

C

15. A 41-year-old woman has made the recent decision to start a family, and is eager to undergo testing to mitigate the possibility of having a child with Down syndrome. Which of the following tests is most likely to provide the data the woman seeks? A) Genetic testing of the woman B) Genetic testing of the woman and the father C) Prenatal blood tests D) Ultrasonography

C

16. A 70-year-old female patient has had her mobility and independence significantly reduced by rheumatoid arthritis. Which of the following processes likely contributed to the development of her health problem? A) Delayed-type hypersensitivity (DTH) reaction B) Proliferation of cytotoxic T cells C) Failure of normal self-tolerance D) Deletion of autoreactive B cells

C

3. Autosomal recessive disorders are characterized by: A) age of onset later in life. B) abnormal protein structure. C) inborn errors of metabolism. D) one in two risk of a carrier child.

C

4. When a male child inherits an X-linked disorder from his heterozygous carrier mother, A) his sons will be carriers. B) his father has the disorder. C) some of his sisters will be carriers. D) his daughters will have the disorder.

C

A 12-year-old female is newly diagnosed with type 1 DM. When the parents ask what causes this, what is the nurses best response? a. A familial, autosomal dominant gene defect b. Obesity and lack of exercise c. Immune destruction of the pancreas d. Hyperglycemia from eating too many sweets

C

A 15-year-old male was struck by a motor vehicle and suffered a traumatic brain injury. Paramedics found him unconscious at the scene of the accident. During the ambulance ride, he regained consciousness and was able to maintain a conversation with the medical staff. Upon arrival to hospital, he was alert and oriented. Physical exam reveals confusion and impaired responsiveness. What is the probable nature of his brain injury based on this history? a. Mild concussion b. Subdural hematoma c. Extradural (epidural) hematoma d. Mild diffuse axonal injury

C

A 20-year-old female suffered from spinal cord injury that resulted from a motor vehicle accident. She had spinal shock lasting 15 days and is now experiencing an uncompensated cardiovascular response to sympathetic stimulation. What does the nurse suspect caused this condition? a. Toxic accumulation of free radicals below the level of the injury b. Pain stimulation above the level of the spinal cord lesion c. A distended bladder or rectum d. An abnormal vagal response

C

A 25-year-old male presents to his primary care provider reporting changes in facial features. CT scan reveals a mass on the anterior pituitary, and lab tests reveal severely elevated growth hormone (GH). Which of the following would the nurse also expect to find? a. Decreased IGF-1 b. Hypotension c. Sexual dysfunction d. Height increases

C

A 30-year-old female had a seizure that started with her fingers and progressive1y spread up her arm and then extended to her leg, with no loss of consciousness. How should the nurse chart this? a. Myoclonic seizure b. Tonic-clonic seizure c. Focal motor seizure d. Atonic seizure

C

A 30-year-old male was diagnosed with hypothyroidism. Synthesis of which of the following would decrease in this patient? a. Corticosteroid B globulin b. Sex hormone-binding globulin c. Thyroid-binding globulin d. Albumin

C

A 30-year-old male was diagnosed with thyroid carcinoma. The lab tests the nurse would most likely find are _____ T3 and T4 levels. a. High b. Low c. Normal d. Variable

C

A 30-year-old white male recently suffered a cerebrovascular accident. Which of the following is the most likely factor that contributed to his stroke? a. Age b. Gender c. Diabetes d. Race

C

A 32-year-old male was injured in a motor vehicle accident and confined to bed for 3 weeks. During this time, the size and strength of muscle fibers decreased, a condition referred to as: a. Myodysplasia b. Ischemic atrophy c. Disuse atrophy d. Deconditioning hypoplasia

C

A 32-year-old obese male begins a jogging routine. A week after beginning, he fractures his leg. This is referred to as a what type of fracture? a. Comminuted b. Greenstick c. Fatigue d. Compound

C

A 33-year-old male is brought to the ER for treatment of injuries received in a motor vehicle accident. An MRI reveals an injury of the cervical cord. Cord swelling in this region may be life threatening because: a. Increased ICP may occur. b. Reflexes will be disrupted. c. Diaphragm function may be impaired. d. Bladder emptying will not occur.

C

A 35-year-old female with Graves disease is admitted to a medical-surgical unit. Which of the following symptoms would the nurse expect to find before treatment? a. Weight gain, cold intolerance b. Slow heart rate, rash c. Skin hot and moist, rapid heart rate d. Constipation, confusion

C

A 40-year-old male undergoes surgery for a PTH-secreting tumor in which the parathyroid is removed. Which of the following would the nurse expect following surgery? a. Increased serum calcium b. Decreased bone formation c. Decreased calcium reabsorption in the kidney d. Increased calcitonin

C

A 45-year-old female with Graves disease underwent surgical removal of her thyroid gland. During the postoperative period, her serum calcium was low. The most probable reason for her low serum calcium is: a. Hyperparathyroidism secondary to Graves disease b. Myxedema secondary to surgery c. Hypoparathyroidism caused by surgical injury to the parathyroid glands d. Hypothyroidism resulting from lack of thyroid replacement

C

A 46-year-old male presents with severe pain, redness, and tenderness in the right big toe. He was diagnosed with gouty arthritis. He is at risk for developing: a. Cholelithiasis b. Myocarditis c. Renal stones d. Liver failure

C

A 46-year-old male presents with severe pain, redness, and tenderness in the right big toe. He was diagnosed with gouty arthritis. The symptoms he experienced are caused by the crystallization of _____ within the synovial fluid. a. Purines b. Pyrimidines c. Uric acid d. Acetic acid

C

A 50-year-old male patient is deficient in ADH production. Which of the following assessment findings would the nurse expect to find? a. Increased blood volume b. Increased urine osmolality c. Increased urine volume d. Increased arterial vasoconstriction

C

A 50-year-old male presents with low back pain. He denies trauma and says he just woke up and it was hurting. An MRI reveals that the vertebra at L5 slid forward relative to those above and below it. Which of the following conditions will be documented on the chart? a. Degenerative disk disease b. Spondylolysis c. Spondylolisthesis d. Spinal stenosis

C

A 54-year-old male was recently diagnosed with rheumatoid arthritis (RA). Which of the following is the expected treatment of choice? a. Nonsteroidal anti-inflammatory drugs (NSAIDs) b. Gold salts c. Methotrexate d. Hydroxychloroquine

C

A 56-year-old male was admitted to the hospital with a diagnosis of osteomalacia. History reveals that he takes anticonvulsants, underwent small bowel resection 3 years earlier, and suffers from chronic pancreatitis. What is the common link between these three factors and the development of osteomalacia? a. Impaired phosphate absorption b. Increased calcium excretion c. Vitamin D deficiency d. Impaired vitamin C metabolism

C

A 60-year-old female with a recent history of head trauma and a long-term history of hypertension presents to the ER for changes in mental status. MRI reveals that she had a hemorrhagic stroke. What does the nurse suspect caused this type of stroke? a. Rheumatic heart disease b. Thrombi c. Aneurysm d. Hypotension

C

A 65-year-old female loses her balance while walking in the woods, causing her to fall and hit her head. She loses consciousness and is in a coma for 5 days. She is diagnosed as having diffuse brain injury. Which of the following would most likely occur in this patient? a. Complete loss of vision b. Arrhythmia c. Blunted affect d. Meningitis infection

C

A 65-year-old male suffers a subarachnoid hemorrhage secondary to uncontrolled hypertension. He appears drowsy and confused with pronounced focal neurologic deficits. This condition is grade: a. I b. II c. III d. IV

C

A 70-year-old female has brittle bones secondary to osteoporosis. Her primary care provider prescribes calcitonin to: a. Activate vitamin D b. Stimulate osteoclastic activity c. Inhibit calcium resorption from bones d. Promote thyroid hormone release

C

A 70-year-old female is being closely monitored in the neurological critical care unit for a severe closed head injury. After 48 hours, her condition begins to deteriorate. Her pupils are small and sluggish, pulse pressure is widening, and she is bradycardic. These clinical findings are evidence of what stage of intracranial hypertension? a. Stage 1 b. Stage 2 c. Stage 3 d. Stage 4

C

A 70-year-old female presents with a hip fracture secondary to osteoporosis. This condition is caused by an increase in bone: a. Density b. Formation c. Resorption d. Mineralization

C

A 70-year-old female with osteoporosis fractures her leg at a location of preexisting abnormality. She reports that the fracture occurred following a minor fall. Which of the following best describes the fracture? a. Fatigue fracture b. Stress fracture c. Pathologic fracture d. Greenstick fracture

C

A 72-year-old male demonstrates left-sided weakness of upper and lower extremities. The symptoms lasted 4 hours and resolved with no evidence of infarction. The patient most likely experienced a(n): a. Stroke in evolution b. Arteriovenous malformation c. Transient ischemic attack d. Cerebral hemorrhage

C

A 76-year-old female was diagnosed with osteoporosis by radiologic exam. She is at high risk for: a. Bone infections b. Joint injuries c. Pathologic bone fractures d. Ssteomalacia

C

A clinician is preparing to teach about bone matrix. Which information should be included? The component of the bone matrix that gives bone tensile strength is: a. Calcium b. Phosphate c. Collagen d. Magnesium

C

A compensatory alteration in the diameter of cerebral blood vessels in response to increased intracranial pressure is called: a. Herniation b. Vasodilation c. Autoregulation d. Amyotrophy

C

A major contributing process in CVAs is the development of atheromatous plaques in cerebral circulation. These most commonly form: a. In the larger veins b. Near capillary sphincters c. In cerebral arteries d. In the venous sinuses

C

A malignant tumor of striated muscle tissue origin is called: a. Myelogenic tumor b. Giant cell tumor c. Rhabdomyosarcoma d. Rhabdomyoma

C

A neurologists explains that arousal is mediated by the: a. Cerebral cortex b. Medulla oblongata c. Reticular activating system d. Cingulate gyrus

C

A nurse checks lab results as both Cushing syndrome and Addison disease can manifest with elevated levels of: a. ADH b. Cortisol c. Adrenocorticotropic hormone (ACTH) d. Aldosterone

C

A nurse is caring for a patient with SIADH. What severe complication should the nurse assess for? a. Stroke b. Diabetes insipidus c. Neurologic damage d. Renal failure

C

A nurse is discussing the different phases of bone remodeling. Which information should be included? Phase 2 of the bone remodeling cycle involves bone: a. Formation b. Activation c. Resorption d. Fracturing

C

A public health nurse has noted a significant increase in the number of school-aged children who are obese. Which of the following factors is most significant predictor of childhood obesity? a. Low socioeconomic status b. Low self-esteem c. Having obese parents d. Living in a rural or inner-city neighborhood

C

A student asks the instructor which of the following is the most potent naturally occurring glucocorticoid. How should the instructor respond? a. Aldosterone b. Testosterone c. Cortisol d. Prolactin

C

An aide asks the nurse what activates tyrosine. What is the nurses best response? a. GH b. PRL c. Insulin d. Estrogen

C

An initial assessment finding associated with acute spinal cord injury is _____ the injury. a. Pain below the level of b. Loss of autonomic reflexes above c. Loss of voluntary control below d. Hyperactive spinal reflexes below

C

Protein contains nitrogen. A negative nitrogen balance represents: a. more protein consumed than excreted. b. a reduced need for nitrogen as protein. c. more nitrogen excreted than consumed. d. less use of nitrogen for protein synthesis.

C

Protein-calorie malnutrition with loss of lean tissues and muscle mass results in: a. respiratory muscle stimulation. b. excessive blood cell production. c. diarrhea. d. increased cardiac contractility.

C

Similarities between girls or women with anorexia nervosa and bulimia nervosa include: a. periodontal disease. b. low estrogen level. c. electrolyte imbalances. d. enlarged parotid gland.

C

Six weeks ago a female patient suffered a T6 spinal cord injury. She then developed a blood pressure of 200/120, a severe headache, blurred vision, and bradycardia. What does the nurse suspect the patient is experiencing? a. Extreme spinal shock b. Acute anxiety c. Autonomic hyperreflexia d. Parasympathetic areflexia

C

The epimysium, perimysium, and endomysium are made of: a. Bone b. Epithelium c. Fascia d. Cartilage

C

The most common cause of toxic myopathy is: a. Infection b. A tumor c. Alcoholism d. Osteoporosis

C

The nurse is assessing the patient with a pen light. The integrity of which cranial nerve is being evaluated? a. Olfactory b. Vagus c. Oculomotor d. Trigemina

C

When a patient asks what the somatic nervous system controls, how should the nurse respond? a. the heart b. the spinal cord c. skeletal muscle d. smooth muscle organs

C

When a patient asks, What is the cause of multiple sclerosis? how should the nurse reply? The cause of MS seems to be an interaction between: a. Vascular and metabolic factors b. Bacterial infection and the inflammatory response c. Autoimmunity and genetic susceptibility d. Neurotransmitters and inherited genes

C

When a student asks which type of nerves transmit nerve impulses at the fastest rate, what is the best response by the nurse? a. Large, non-myelinated b. Small, myelinated c. Large, myelinated d. Small, non-myelinated

C

When the nurse is using the term sarcopenia, what is the nurse describing? It is the: a. Absence of muscle cells b. Fatigue of muscle cells c. Age-related loss in skeletal muscle d. Muscles that are unable to contract

C

Which of the following characteristics distinguishes kwashiorkor from marasmus? a. Impairment of immune function b. Lack of dietary fat intake c. High intake of carbohydrates d. Impaired pigment synthesis

C

Which of the following measures should a school nurse prioritize in the treatment and prevention of childhood obesity? a. Group cognitive therapy b. Use of selective serotonin-reuptake inhibitors (SSRIs) c. Education on exercise and nutrition d. High-protein, low-carbohydrate diet

C

Which of the following patients is most likely to be in positive nitrogen balance? a. A patient who is receiving treatment for sepsis b. A patient whose diagnosis of pneumonia is causing a fever c. A woman who has been admitted to the hospital in early labor d. A patient who sustained extensive burns in a recent industrial accident

C

While planning care for a patient with muscle problems, which information should the nurse remember? _____ ions directly control the contraction of muscles. a. Sodium b. Potassium c. Calcium d. Magnesium

C

a 32-year-old female suffers from severe brain damage following a motor vehicle accident. after rehabilitation she notices that her though processes and goal-oriented behavior are impaired. which are does the nurse suspect is damaged? a. thalamus b. limbic c. prefrontal d. occipital

C

a cell was isolated from the CNS. a researcher revealed that its main function was to clear cellular debris. what type of cell is the researcher studying? a. astrocyte b. ependymal cell c. microglia d. schwann cell

C

a neurologist is teaching about sensory pathways. which information should the neurologist include? sensory pathways in the spinal cord include the: a. corticospinal tract b. pyramids c. spinothalamic tract d. anterior column

C

a neurologist is teaching about the location of the primary visual cortex in the brain. which area is the neurologist discussing? a. frontal lobe b. temporal lobe c. occipital lobe d. parietal lobe

C

a nurse is teaching about the area of the spinal cord that contains cell bodies involved in the autonomic nervous system. which of the following area is the nurse discussing? a. anterior horn b. ventral horn c. lateral horn d. dorsal horn

C

an experiment looking at an isolated neuron revealed a sensory nerve with one process containing a dendritic portion extending away from the CNS and an axon extending toward the CNS. which of the following classifications would this neuron fall into? a. bipolar b. multipolar c. pseudounipolar d. interpolar

C

the __________ ensures collateral blood flow from blood vessels supplying the brain. a. carotid arteries b. basal artery c. circle of willis d. vertebral arteries

C

. An endocrinologist isolated a new hormone and found it to be a water-soluble amine. Which of the following is most like this new hormone? a. Growth hormone (GH) b. Luteinizing hormone (LH) c. Antidiuretic hormone (ADH) d. Epinephrine

D

. The incidence of fractures of the pelvis is highest in: a. Preadolescent boys b. Adolescent boys c. Adolescent girls d. Older adults

D

. When a patient has a peculiar sensation that immediately precedes a seizure, what term should the nurse use to describe this sensation? a. Prodroma b. Agnosia c. Spasm d. Aura

D

. When insulin binds to its receptors on muscle cells, an increase in glucose uptake by the muscle cells occurs. This is an example of a _____ effect by a hormone. a. Pharmacologic b. Permissive c. Biphasic d. Direct

D

. When the endocrinologist asks the staff how the releasing hormones that are made in the hypothalamus travel to the anterior pituitary, how should the staff reply? Via the: a. Vessels of the zona fasciculata b. Chromophils c. Median eminence d. Hypophysial portal system

D

. Which layer forms the fascicles in skeletal muscle? a. Tendon sheath b. Endomysium c. Epimysium d. Perimysium

D

. Which of the following alterations would the nurse expect to find in a patient with untreated Cushing disease or syndrome? a. Bradycardia b. Tachypnea c. Hyperkalemia d. Hypertension

D

. Which statement by a patient indicates teaching was successful regarding myasthenia gravis? Myasthenia gravis results from: a. Viral infection of skeletal muscle b. Atrophy of motor neurons in the spinal cord c. Demyelination of skeletal motor neurons d. Autoimmune injury at the neuromuscular junction

D

10. HIV-positive persons that display manifestations of laboratory category 3 or clinical category C are considered to have: A) zero viral load. B) seroconversion. C) complete remission. D) AIDS-defining illnesses.

D

12. A patient with a long history of hay fever has recently begun a series of immunotherapy (allergy shots). How will this treatment potentially achieve a therapeutic effect? A) By blocking cytokine release from sensitized mast cells B) By preventing mast cells from becoming sensitized C) By causing T cells to be sequestered in the thymus for longer periods D) By stimulating production of IgG to combine with antigens

D

12. Which of the following statements is true of autosomal recessive disorders? A) Onset is typically late in childhood or early in adulthood. B) Symptomatology is less uniform than with autosomal dominant disorders. C) Mitochondrial DNA is normally the site of genetic alteration. D) Effects are typically the result of alterations in enzyme function.

D

15. A patient has developed pericarditis after developing acute glomerulonephritis, a development that may be attributable to the presence of similar epitopes on group A, b- hemolytic streptococci and the antigens in the patients heart tissue. Which of the following has most likely accounted for this patients autoimmune response? A) Breakdown of T-cell anergy B) Release of sequestered antigens C) Superantigens D) Molecular mimicry

D

19. A patient has been admitted to the hospital for the treatment of HIV infection, which has recently progressed to overt AIDS. Which of the following nursing actions should the nurse prioritize when providing care for this patient? A) Frequent neurologic vital signs and thorough skin care B) Hemodynamic monitoring and physical therapy C) Careful monitoring of fluid balance and neurologic status D) Astute infection control and respiratory assessments

D

2. In addition to having a 50% chance of inheriting an autosomal dominant disorder from an affected parent, such a disorder is characterized by: A) aneuploidy of genes in all cells. B) deficiencies in enzyme synthesis. C) affected X transmission to daughters. D) varied gene penetration and expression.

D

8. A teratogenic environmental agent can cause birth defects when: A) inherited as a recessive trait. B) intense exposure occurs at birth. C) disjunction occurs during meiosis. D) retained during early pregnancy.

D

A 13-year-old female is admitted to the hospital for evaluation and treatment of an osteosarcoma in her left distal femur. Which statement best describes osteosarcoma? a. Myelogenic, develops in red bone marrow only b. Benign, develops in spongy bone tissue c. Collagenic, originates in the periosteum d. Osteogenic, most often develops in the bone marrow

D

A 19-year-old female with type 1 DM was admitted to the hospital with the following lab values: serum glucose 500 mg/dl (high), urine glucose and ketones 4+ (high), and arterial pH 7.20 (low). Her parents state that she has been sick with the flu for a week. Which of the following statements best explains her acidotic state? a. Increased insulin levels promote protein breakdown and ketone formation. b. Her uncontrolled diabetes has led to renal failure. c. Low serum insulin promotes lipid storage and a corresponding release of ketones. d. Insulin deficiency promotes lipid metabolism and ketone formation.

D

A 20-year-old male is brought to the emergency room (ER) for treatment of injuries received in a motor vehicle accident. A spinal cord injury is suspected. What two regions should the nurse assess as they are most likely to be damaged? a. Cervical and thoracic regions b. Thoracic and lumbar regions c. Lumbar and sacral regions d. Cervical and lumbar regions

D

A 20-year-old male was at the supermarket when he fell to the ground. Bystanders reported that he lost consciousness and his body tensed up then relaxed, then tensed and relaxed several times. He most likely was experiencing a(n): a. Partial seizure b. Absence seizure c. Myoclonic seizure d. Tonic-clonic seizure

D

A 25-year-old female with Graves disease is admitted to a medical-surgical unit. Palpation of her neck would most likely reveal: a. A normal-sized thyroid b. A small discrete thyroid nodule c. Multiple discrete thyroid nodules d. Diffuse thyroid enlargement

D

A 25-year-old male presents with fatigue, constipation, and sexual dysfunction. Tests reveal all pituitary hormones are normal and no masses are present. The nurse suspects the most likely cause of his symptoms is a dysfunction in the: a. Anterior pituitary b. Posterior pituitary c. Pars intermedia d. Pituitary stalk

D

A 35-year-old female took corticosteroid therapy for several months. Which of the following a. Renal toxicity b. Episodes of hypoglycemia c. Hypotension d. Type 2 DM

D

A 36-year-old male complains of pain and weakness in the elbow. He reports that he is a warehouse worker and lifts boxes daily. MRI reveals inflammation of the tendon where it attaches to bone. This condition is called: a. Periostitis b. Muscle strain c. Bursitis d. Epicondylopathy

D

A 44-year-old patient with pulmonary tuberculosis (lung infection) is evaluated for SIADH. Which of the following assessment findings would be expected in this patient? a. Peripheral edema b. Tachycardia c. Low blood pressure d. Concentrated urine

D

A 45-year-old female has elevated thyroxine production. Which of the following would accompany this condition? a. Increased thyroid-releasing hormone (TRH) b. Increased anterior pituitary stimulation c. Decreased T4 d. Decreased thyroid-stimulating hormone (TSH)

D

A 50-year-old male suffers a severe head injury when his motorcycle hits a tree. His breathing becomes deep and rapid but with normal pattern. What term should the nurse use for this condition? a. Gasping b. Ataxic breathing c. Apneusis d. Central neurogenic hyperventilation

D

A 75-year-old female suffers a stroke and now has difficulty writing and production of language. This condition is most likely caused by occlusion of the: a. Anterior communicating artery b. Posterior communicating artery c. Circle of Willis d. Middle cerebral artery

D

A nurse is preparing to teach about myofibrils. Which information should the nurse include? The structure of the contractile subunit of the myofibril is called the: a. Sarcoplasmic reticulum b. Actin c. Motor endplate d. Sarcomere

D

A nurse is preparing to teach staff about the most common type of traumatic brain injury. Which type of traumatic brain injury should the nurse discuss? a. Penetrating trauma b. Diffuse axonal injury c. Focal brain injury d. Concussion

D

A nurse is teaching staff about protein hormones. Which information should the nurse include? One of the protein hormones is: a. Thyroxine (T4) b. Aldosterone c. Testosterone d. Insulin

D

A nurse is teaching the staff about oxytocin. Which information should the nurse include? Target cells for oxytocin are located in the: a. Renal tubules b. Thymus c. Liver d. Uterus

D

A nurse recalls insulin has an effect on which of the following groups of electrolytes? a. Sodium, chloride, phosphate b. Calcium, magnesium, potassium c. Hydrogen, bicarbonate, chloride d. Potassium, magnesium, phosphate

D

A nurse recalls that when a muscle acts as the prime mover, it is called the: a. Flexor b. Recruiter c. Antagonist d. Agonist

D

A nurse thinks a patient may be experiencing dementia. Which assessment finding will most help support this diagnosis? a. Violent behavior b. Hyperactivity c. Depression d. Loss of recent and remote memory

D

A patient wants to know what can cause ACTH to be released. How should the nurse respond? a. High serum levels of cortisol b. Hypotension c. Hypoglycemia d. Stress

D

A teenage boy sustains a severe closed head injury following an all-terrain vehicle (ATV) accident. He is in a state of deep sleep that requires vigorous stimulation to elicit eye opening. How should the nurse document this in the chart? a. Confusion b. Coma c. Obtundation d. Stupor

D

Which of the following statements best conveys the endocrine function of adipose tissue? a. Adipose tissue antagonizes the effects of insulin on cell membranes. b. Adipose tissue produces ghrelin, which stimulates both appetite and eating. c. Adipose tissue produces and secretes cholecystokinin (CCK), which stimulates the hypothalamic feeding center. d. Adipose tissue produces leptin, which mediates body weight.

D

Which of the following would increase a patients risk for thrombotic stroke? a. Hyperthyroidism b. Hypertension c. Anemia d. Dehydration

D

_____ are most at risk of spinal cord injury from minor trauma. a. Infants b. Men c. Women d. The elderly

D

a 20-year-old male was brought to the emergency room for severe burns. he requested something for the excruciating pain he was experiencing. blocking which of the following neurotransmitters would reduce his pain? a. enkephalin b. dopamine c. acetylcholine d. substance p

D

a neurologist is teaching the staff about motor neurons. which information should be included? motor neurons are structurally classified as _____________ neurons. a. unipolar b. pseudounipolar c. bipolar d. multipolar

D

a nurse is preparing to teach about functions to maintain homeostasis and instinctive behavioral patterns. which area of the brain is the nurse discussing? a. thalamus b. medulla c. cerebellum d. hypothalamus

D

a patient is looking at a picture of the brain and points to the convolutions on the surface of the cerebrum. the nurse should tell the patient these are called: a. sulci b. fissures c. reticular formations d. gyri

D

a patient presents with altered respiratory patterns following head trauma. based upon the symptoms, which of the following areas does the nurse suspect is injures? a. cerebrum b. cerebellum c. midbrain d. reticular formation

D

7. The predominant antibody of a typical primary immune response is: a. IgG b. IgM c. IgA d. IgE ANS: B

IgM is the largest immunoglobulin and is the first antibody produced during the initial, or primary, response to antigen. IgM, not IgG, is the largest immunoglobulin and is the first antibody produced during the initial, or primary, response to antigen. IgM, not IgA, is the largest immunoglobulin and is the first antibody produced during the initial, or primary, response to antigen. IgM, not IgE, is the largest immunoglobulin and is the first antibody produced during the initial, or primary, response to antigen. Cognitive Level: Comprehension 147

26. A 42-year-old female is diagnosed with chronic renal failure, and the nurse is discussing dietary treatment. Which information indicates the nurse understands dietary regimen? Treatment includes restricting: a. Fats b. Complex carbohydrates c. Proteins d. Sugars ANS: C

Low-protein diets are recommended. Low-protein diets, not reduced fats, are recommended. Low-protein diets, not reduced carbohydrates, are recommended. Low-protein diets, not reduced sugars, are recommended. Cognitive Level: Comprehension 760


Kaugnay na mga set ng pag-aaral

Maternal Child Nursing Care Chapter 16 Nursing Care of the Family During Labor and Birth

View Set

COMP129 - Chapter 7: Internet Blueprint

View Set

Ap Euro Ch. 19 Revolutions in Politics 1775 - 1815

View Set